Anda di halaman 1dari 148

...

...
.( . . )


....
5341 6 51 4153 3- 51 4153
:
5 .

4 .
4 .
4 .
3 .
1 .
6 .
) ( .7
.8
==========================================
.
====================================
...






, :
:
, :

, , :

:
:
1. Dental calculus contributes to periodontal diseases by:
a. Mechanical irritation
b. Harbors microorganisms ***

2. Child lives in an area with water fluoridation if 0.2 ppm. What is the most appropriate
management:
A. give daily oral tablets 0.5 mg
B. give daily oral tablets 1 mg...
C. give daily fluoride mouthwash
D. perform pits and fissure sealants ***
.( )

if take 0.2 ppm and child age was:
6 months 3 yrs. 0.25 mg
3-6 yrs. 0.5 mg
6 yrs. up to at least 16 yrs. 1.0 mg

3. pulp hyperemia in a deciduous tooth is caused by:
a. Pulp necrosis
b. Congestion of blood vessels ***
c. Fibrosis
d. Pulp ischemia

4. In selecting color shade for composite resin:
a. Use bright light
b. Use dry tooth with rubber dam
c. Use dry shade guide
d. None of the above ***

5. Patient has adrenal insufficiency come to your clinic and start developing signs of
adrenal crisis you would administer:
a. 40-60 . .. Corticosteroid (oral tab)
b. 100-200 ... corticosteroids (oral tab) ***
C. . (IV)
D. . (IV)

6. Child has missing 2 canines and left D, which space maintainer is better:
a. lingual arch ***
b. band and loop
c. distal shoe
I answered the question depend on patient at 7 yrs. & lower jaw and in the question
does not mention the exact age..
7. 6 years old child have 74 and 84 extracted best space maintainer is:
a. Lingual arch. ***
b. Bilateral band and loop.
c. Bilateral distal shoe.
d. No need for space maintainer
74 = Left deciduous lower 1
st
molar
84 = Right deciduous lower 1
st
molar
3
1.


2. ... D E
3.
====================

crown and loop
band and loop
distal shoe
:
1. Lingual arc \ arch

2. Nance
( )
3. Partial denture
E, D

===================
7 6

33-33

8. Time for HIV to show symptoms:
a. 1-5 years
b. more than 9 years ***
c. no definite timing
d. none of the above
http://aids.gov/hiv-aids-basics/hiv-aids-101/signs-and-symptoms/
http://211bigbend.net/hivaids-symptoms.html
http://www.health24.com/Medical/HIV-AIDS/Symptoms-and-diseases-
associated/Symptoms-and-phases-of-HIV-infection-Aids-20120721


9. Adult patient with buccal fistula apical to a recently treated RCT in tooth 44
upon insertion of a gutta percha cone in the fistula to track it. Radiograph shows the tip
of GP cone touching the side of 44 root. The cause of this fistula is:
1. Lateral Periodontal abscess***
2. Untreated accessory canal
3. Periradicular cyst.

10. Upper class I PD with porcelain teeth against full natural teeth. After 3 years, bilateral
posterior teeth in mandible were extracted. the choice for a lower bilateral free end
saddle PD would be:
a. porcelain teeth
b. porcelain teeth with gold occlusal Surface ***
c. acrylic teeth with gold occlusal surface
d. hardened acrylic teeth

11. Pt. has RPD with distal extension opposing to natural, he now with lost lower
posterior teeth if he needs RPD and upper teeth with porcelain type of teeth in lower:
1. Porcelain
2. Acrylic
3. Porcelain with gold occlusally ***
4. Acrylic reinforced
12. Systemic factors can cause periodontal conditions:
OR
Systemic factors can modify periodontal conditions:
a. both correct ***
b. both false
c. First correct second false
d. First false second correct

13. Best material for canal weeping is:
a. gutta percha
b. Calcium Hydroxide***
c. Formocresol

14. Restorative material that bond chemically to teeth
a. Glass ionomer***

15. When cementing gold inlay, what should you do before complete setting of cement:
1. Adjust occlusal height
2. Remove excess cement
3. Adapt peripheries with burnisher ***

16. To assess range of movement of mandibular condyles. all these techniques are useful
EXCEPT:
1. Conventional transcranial ***
2. Conventional tomography
3. Computerized tomography
4. Arthrography

17. Standard of infection control advice sequence of infected dental instrument be:
a. initial cleaning, inspection, cleaning, sterilization, storage ***
b. Inspection, cleaning, sterilization, storage.
c. Cleaning, sterilization, storage.
d. Sterilization and storage.
First Aid 2008, Page 335
Sequences of instrument processing:
Cleaning, Packing, sterilization, storage.


18. 3 days after delivery of upper and lower complete dentures comes with difficulty in
swallowing and ulcers in post dam area of upper denture:
1. under extended post dam
2. Over extended post dam ***
3. High post dam
4. Occlusal interference

19. The most superior for pulp testing is:
1. Cold water
2. Ice
3. Endo ice spray ***
4. Ethyl chloride

20. After pouring an irreversible hydrocolloid impression after waiting for more than 15
mins. The stone cast looks chalky on the surface:
OR
After pouring a master cast for alginate impression have chalky occlusal surface
appearance why:
1. Dehydration of alginate impression ***
2. Formation of exudates on surface
3. Slurry water
4. 1 or 2

21. Old pt. has upper and lower complete dentures of 8 years old. Comes to clinic with
small discrete slightly elevated white lesions on the alveolar ridge of lower jaw. No
symptoms. you would:
1. Excise lesions immediately
2. Ask pt. to remove lower denture for 2 weeks then reevaluate***
3. Make new dentures
22. Pt. with multiple sebaceous glands on back of neck and palms. has osteomas in
mandible and multiple unerupted teeth:
1. Hyperparathyroidism
2. cliedocranial dysistosis
3. Gardner's syndrome ***
First Aid 2008, Page 312

23. To prevent fracture in tooth with bicuspidization:
1. Composite resin connecting 2 parts
2. Full coverage crown ***
3. Orthodontic fixation

24. A child having acute lymphocytic leukemia (ALL) comes to clinic for extraction of a
primary tooth. active neutrophil count (ANL) was 1700:
1. Defer dental treatment
2. Give antibiotic coverage and extract
3. Proceed as normal pt. ***
4. Give platelet transfusion

25. 10 year old child received a trauma to central incisor few hours ago. He has pain and
numbness in gingiva around tooth. there's no mobility or displacement of tooth:
a. give oral analgesics and follow up***
b. RCT
c. extract tooth
d. splinting followed by pulpectomy

26. 30 year old received a trauma to central. which caused root fracture between middle
and apical thirds:
1. Extract coronal part
2. Do RCT for entire tooth
3. Do RCT for coronal part ***
4. Splint both parts of fractured root

27. In extensive amalgam restoration. for each cusp we use how many pins:
a. 0
b. 1***
c. 2
d.3

28. Nicotinic stomatitis:
1. acanthosis with keratin***
2. Prickle cell like shape bases

29. Testing pulp vitality of a tooth with PFM crown using:
OR
The best way To make pulp vitality test of a tooth with PFM crown:
a. cold
b. heat and cold
c. cold under rubber dam ***
d. electric test

30. Patient had bulimia and had lesion in palatal surface in upper teeth with recurrent
vomiting. What is the type of lesion :
a. attrition
b. abrasion
c. erosion***

31. Young adolescent complaining of rapidly growing unilateral mass in body of mandible,
which is painful, causing paresthesia. Radiographically, it shows large ill-defined
radiolucent lesion:
a. ostoma
b. fibrous dysplasia
c. osteomyelitis
d. osteosarcoma***

32. Secondary dentine occur due to:
a. occlusal trauma
b. recurrent caries
c. attrition dentine
d. all of the above***

33. Difference between Gracey and universal curette:
a. Section of gracey is hemi circular and in universal triangular.
b. Gracey has one cutting edge while universal has two.
c. Gracey Used for cutting in specific area while universal is in any area.
d. Universal 90 not offset, gracey 60 offset.
e. A and d
f. A, b and c
g. B, C and D. ***

34. Medications used for GIT disturbance:
a. H1 receptor blockers
b. H2 receptor blockers***

35. 15 year old pt. with right mandibular slowly growing mass. in radiograph it shows a
well-defined radiopaque lesion surrounded by radiolucent periphery:
a. osteoma
b. cementoblastoma***
c. ossifying fibroma
d. osteosarcoma

36. A fracture 3 days after delivery between the abutment and free end saddle. You
would fix it by:
OR
How can u repair fractured rest (in the place where it passes over the marginal ridge of
the tooth) in removable partial denture:
a. Spot welding
b. Electrical Soldering ***
c. Laser welding
d. Industrial brazing.


37. After extraction of tooth 3 & 8, you need to make sutures. what to use to hold the two
segments of wound:
a. Allis Forceps
b. Adison forceps ***
c. Curved hemostat

38. What would you do right after taking symptoms from pt.:
a. Start cavity preparation
b. Oral hygiene instructions
c. X-rays
d. Start examination***

39. Young patient with ulceration in his mouth and bad breath:
a. Acute generalized periodontitis
b. Herpetic gingivostomatitis
c. ANUG ***

40. We use low speed hand piece (5000 15000) in preparation of a cavity with deep
carious lesion in a primary tooth to:
a. Less vibration
b. Less dust formation
c. Prevent pulp exposure***

41. Your dental assistant asks you how to disinfect the dental operatory after treatment
of HBV pt. :
a. Formaldehyde
b. Ethylene oxide gas
c. Ethyl alcohol (= Ethanol)
d. Sodium hypochlorite
e. 100% Dettol ***
HBV= Hepatitis B Virus
)(
mixture of 70% ethanol or isopropanol diluted in water
08 % ethanol + 5% isopropanol


http://en.wikipedia.org/wiki/Disinfectant

42. Pt. Comes to your clinic after 34 hours after placing a new amalgam filling opposing a
gold inlay with sensitivity on biting due to galvanic shock:
a. Put varnish
b. Put separating medium
c. Change restoration material ***
d. Extract

43. Techniques of local anesthesia depend on:
a. Size of nerve
b. Type of bone ***
c. Location of nerve
d. Type of anesthetic solution

44. Lateral pterygoid muscle has:
a. One point of origin
b. Two***
c. Three

45. Patient comes to your clinic with anterior open bit following a trauma to the chin:
a. Unilateral condyle fracture
b. Bilateral condylar fracture***
c. Le fore 1 fracture

46. A child received a trauma to face upon radiographic examination you see a fracture in
the angle of mandible (or neck of condyle) there is no dislocation and pt. can open and
close his mouth with normal occlusion:
a. Intermaxillary fixation with wire for 3 weeks
b. Intermaxillary fixation with elastic for 6 weeks
c. Open reduction and plates
d. No treatment***

47. A question about advantages of partial coverage veneer. select correct answers:
More than one
a. All margin are visible to allow for doctor examination
b. Allow for better oral hygiene
c. Permit pulp testing
D. A & B & C ***
the most accurate is B &c
http://www.slideshare.net/indiandentalacademy/vamc-partial-veneer-crowns
http://books.google.com.sa/books?id=ZGvegIdicUoC&pg=PA585&lpg=PA585&dq=advanta
ges+of+partial+veneer&source=bl&ots=EvRmJlyvBe&sig=lh6shDuaFNuSpWKiQra5sfQPFdI
&hl=en&sa=X&ei=i2sWU_fcE4GNtAaQ4YHwCA&ved=0CD8Q6AEwBw#v=onepage&q=adva
ntages%20of%20partial%20veneer&f=false

48. A question about sequences of management of cleft lip and palate

49. Three weeks after delivery of a unilateral distal extension mandibular removable
partial denture, a Pt complained of
A sensitive abutment tooth, clinical examination reveals sensitivity to percussion of the
tooth, the most likely cause is:
a. Defective occlusion. ***
b. Exposed dentine at the bottom of the occlusal rest seats.
c. Galvanic action between the framework and an amalgam restoration in the abutment
tooth.

50. Caries propagation in adult pt. is slower than young because:
a. More sclerotic dentin***

51. The PH of the calcium hydroxide is:
a. 7.2
b. 12 *** (Exactly 12.5)
c. 19
d. 5.5

52. Calcium hydroxide is used in deep cavity because it is:
a. Simulate formation of 2
nd
dentine. ***
b. Not irritant to the pulp.
c. For thermal isolation

53. Success of pit & fissure sealants is affected mainly by:
a. increased time of etching
b. contamination of oral saliva***
c. salivary flow rate
d. proper fissure sealant

54. The following are multilocular radiolucency in x-ray EXCEPT:
a. Ameloblastoma.
b. Odontogenic keratocyst.
c. Adenomatoid Odontogenic cyst. ***
d. Myxoma
55. Pt with new denture complain of too tight dentures in morning that loosen at night.
This is due to:
a. Poor post dam
b. Relief of denture ****
c. Reduced elasticity of cheeks

56. When doing a study. Participant should get:
a. Written informed consent in native language****
b. Oral consent
c. No need for consent

57. Pt. calls you with avulsed tooth an hour ago. Best transfer medium:
OR
Patient with avulsed tooth they call u what to do, u told them to come immediately to
replant the tooth, what is the best medium:
a. Saliva Under tongue
b. Water
c. Cold milk***
d. put inside pt. mouth
http://www.aapd.org/media/Policies_Guidelines/RS_TraumaFlowSheet.pdf
)(.


58. All of the following are oral features of acquired immunodeficiency syndrome AIDS
EXCEPT:
a. Candidiasis.
b. Erythema multiform. ****
c. Hairy leukoplakia.
d. Rapidly progressing periodontitis.

59. Restoration of a cavity extending to cementum:
a. Veneer
b. Composite
c. Glass ionomer****
d. Amalgam


60. A patient 14 years with avulsed incisors 11 and 21 we can use a splint for:
OR
Splint in avulsed tooth:
OR
When do reimplantation for avulsed teeth fixed it to:
a. 1 2 weeks. ***
b. 2 3 weeks.
c. 3 4 weeks.
d. 4 5 weeks.

61. Food low cariogenic potential the following should be characteristic:
1. Low buffering capacity
2. pH higher than 3
3. Contain mineral ***
4. Contain protein

62. In hairy tongue, which taste buds increase in length:
OR
Pt. with hairy tongue he get injury which make bud to elongate ... what is it:
a. Filli form. ***
b. Fungi form.
c. Foliate.
d. Circumvallates.

63. Patient with pain on the upper right area, and the patient can not tell the tooth causes
the pain, what is the least reliable way to do test pulp:
a. Cold test.
b. Hot test.
c. Electric test *****
d. Stimulation the dentine. (Cavity test)
Electrical test is least reliable way
cavity test is more reliable way (Best reliable way)

64. After scaling and root planning, healing occurs by:
a. Long junctional epithelium. *** = (created)
b. New attachment
c. New bone and connective tissue formation
d. New attached periodontal ligament fibers.

65. Pt. has a lesion in tongue which suffering from scar & fever, the lesion when removed
leave bleeding area under it, diagnosis is:
a. Leukoplakia
b. Candida ***
c. ulcer.

66. In resection the tip of root in Apicoectomy, the cut should be:
OR
Resection of root end surgery must be:
a. Perpendicular to the long axis of tooth. ***
b. Parallel to long axis
c. Acute angle.
d. Obtuse angle.
The cut in Apicoectomy should be perpendicular to long axis or 45 degree
latest studies suggested to cut 3mm of the apex perpendicular to axis, retrograde
preparation of the canal using ultra sound tips and retrograde obturation with MTA

67. What is the function of primer:
a. penetrate into collagen framework and copolymerize with the resin ***

68. A dentine ' Primer':
1. Etches the dentin
2. Raises the surface-free energy (wet) dentine ***
3. Removes the smear layer
4. Bonds composite
69. Bacteria causes Pericoronitis:
a. Anaerobic streptococci (streptococcus melleri group) ***
Happen mostly by styphalococcus & streptococcus.

70. Radiolucent structure occupied by a radiopaque structure that forms a mass of
disorganized arrangement of odontogenic tissue:
a. Complex odontoma. ***
b. Calcifying Epithelial Odontogenic Cyst.
c. Compound odontoma .

71. Duct of submandibular gland is:
a. Wharton. ***
b. Bartholin.
c. Barvenous.
d- Stensen. (Duct of parotid gland).

72. A patient that had a class II amalgam restoration, next day he returns complaining of
discomfort at the site of the restoration, radiographically an overhanging amalgam is
present. This is due to:
a. lack of matrix usage
b. Improper wedging. ***
c. No burnishing for amalgam

73. Patient 5 years old with denture has a severe gag reflex, upon history he says he had
the same symptoms in the first few days of the denture delivery and it went all alone:
A. patient has severe gag reflex
B. patient has underlying systemic condition
c. Denture is overextended. ***
74. Studies show that Complete remeniralization of surface of an accidentally etched
enamel:
A. after hours
b. after weeks.
C. after months. ***
d. Never occur.
75. A headgear appliance is used for :
A. anchorage.
B. traction.
C. both anchorage and traction. ***
D. neither anchorage or traction
Dental Decks

76. The Seibert`s classification represented mostly in defect of residual ridge
cornogigivally at class Kennedy classification:
a. I
b. II
c. III ****
Siebert II has classified residual ridge deformities into 3 categories:
Class I defects-faciolingual loss of tissue width with normal ridge height
Class II defects-loss of ridge height with normal ridge width
Class III defects-a combination of loss in both dimensions
The high incidence (91%) of residual ridge deformity following Ant. tooth loss
the majority of these are Class III defects, Because patients with Class II and III defects
are frequently dissatisfied with the esthetics of their FPDs
preprosthetic surgery to augment the residual ridge should be carefully considered

77. The Seibert`s classification of edentulous ridge regarding the apico-coronal loss is
class:
a. I
b. II ***
c. III
d. IV
http://books.google.com.eg/books?id=J0S6tsl3GocC&pg=PA284

78. Child has unilateral posterior crossbite during eruption of lateral incisor due to:
A. abnormal chewing habit
b. abnormal swallowing habit ***
C. congenital

79. The ideal amount of dentin required between an amalgam restoration and the pulp
for insulation is :
A. 0.5 mm.
B. 1.0 mm
C. 2.0 mm ***
D. 3.0 mm

80. Excessive fluoride levels in drinking water are associated with fluorosis. Fluoride levels
in excess of begin to pose a risk for fluorosis:
a. one part per million.
B. two parts per million.
c. Three parts per million. *** (3ppm)
d. Four parts per million.

81. To select shade of porcelain:
a. One light.
B. Before preparation.
c. Wet tooth. ***
d. Shade guide must be wet.

82. The principle muscle responsible for the opening of the mouth is:
a. Mylohyoid
B. Anterior temporal
c. Posterior temporal
d. Anterior belly of digastric ***

Seibert`s classification of alveolar ridge defect:
-Class I defect: Buccolingual loss of tissue with normal ridge height
in apico-coronal dimension

-Class II defect: Apico-coronal loss of tissue with normal ridge width in
buccolingual dimension

-Class III defect: Combined buccolingual and apico-coronal loss of
tissue resulting in loss of normal ridge height and width

83. Indirect retainers in RPD mostly needed:
A. class I ***
B. class II
C. class III
D. class IV

84. When primary molars are prepared for stainless steel crowns should the depth for
reduction of the proximal surface be similar to the depth of the buccal and lingual
surfaces:
a. Yes, reduction of all wall is similar for best retention
b. No, proximal reduction is greater to allow the crown to pass the contact area ***
c. No, the buccal surfaces has the greatest reduction to remove the cervical bulge
d. Yes, all undercuts are uniformly removed so that the steel crown can be seated
e. No, because of lateral constriction, the lingual surface needs greatest reduction

85. Caries detection method depend on tooth fluorescence:
a. fluorescent detector
b. Diognodent ***

86. Caries detection dye composed main of:
a. 5% acid fuschin
b. 5% basic fuchsin
C. propylene glycol. ***

87. Patient will make endo surgery, the Dr. give her block and still the tooth was not
anesthetized , why:
A. anesthesia spread so far in nerve
B. anesthesia spread with inflammatory fluid ***
C. inflammatory fluid make circulation cycle

88. When injecting without vasoconstrictor, the maximum safe dose of 2% lignocaine
solution for 70Kg adult is:
A. 2.2ml **
B. 22ml

89. Whats the most accurate factors that decide or confirm the outcome disease present
in high population country :
a. Etiological factors
b. Risk factors ***
C. confounding factors
Etiological factors: The study of causes or origins or the branch of medicine that deals
with the causes or origins of disease.
A risk factor is any attribute, characteristic or exposure of an individual that increases
the likelihood of developing a disease or injury. Some examples of the more important
risk factors are underweight, unsafe sex, high blood pressure, tobacco and alcohol
consumption and unsafe water sanitation and hygiene.
Epidemiological studies: search for the causes of diseases, based on associations with
various risk factors that are measured in the study. In addition to the exposures that the
study is investigating, there may be other factors that is associated with the exposure
and independently affects the risk of developing the disease.
Confounding factors (variables): is distorting factors if the prevalence of these other
factors differ between groups being compared, they will distort the observed
association between the disease and exposure under study.
Hypothetical Example of Confounding factor:
a study of coffee drinking and lung cancer. If coffee drinkers were also more likely to be
cigarette smokers, and the study measured coffee drinking but not smoking, the results
may seem to show that coffee drinking increases the risk of lung cancer, which may not
be true. However, if a confounding factor (in this example, smoking) is recognized,
adjustments can be made in the study design or data analysis so that the factor does not
confound the study results.

90. An old patient and has skeletal disorder by using x-ray found enlargement in .
Whats the type of x ray should be more accurate:
a. Cephalometric ***
b. Reverse town
c. Panoramic x-ray

91. How to repair short post dam in the retention denture:
A. relining ***
B. by soft wax this area

92. The most common advantage of wrought wire clasps than the casted clasp
a. Flexibility ***
b. Less irritation to abutment teeth

93. Child came to u with gray discoloration of the deciduous incisor also on radiographic
exam. There is dilation of follicle of the permanent successor what will u do:
1. Extract the deciduous tooth. ***
2. Endodontic treatment for infected tooth.
3. Observe over time.

94. Detection of condylar hyperplasia:
a. OPG
b. CT scan
c. Photon Emission ***
SPECT (single photon emission computed tomography) has been used with quantitative
assessments of one mandibular condyle to clivus or lumbar spine, but we have
compared one condyle with the other, which is more sensitive and accurate in detecting
abnormal activity.
http://www.sciencedirect.com/science/article/pii/S0266435699902097

95. Pt. come complaint of pain and discomfort in upper second or first molar upon
examination reveal that 15
This tooth is abutment to -fixed partial denture and on radiograph all thing is normal ...
discomfort is likely because:
A. fracture of connector
B. vertical root fracture
C. mobility of tooth
D. loose of retainer on abutment ***

96. 3 months baby had black-blue discolored rapid growing swelling, the x-ray shows
unilocular radiolucency and displaced tooth bud, is it:
OR
child 2 yrs. old came to your clinic with his parents ,he has bony lesion bluish-black in
color, the most probable diagnosis is:
a. Melanotic neuroectodermal tumor. ***
b. Giant cell granuloma.
c. Ameloblastic fibroma.
d. Aneurysmal bone cyst

97. Pt. come with deep carious lesion upon radiographic examination show well-
circumscribed radio-opaque mass 16 With radiolucent boundary and no other symptom
is:
a. odontogenic tumor
B. condensing ostitis ***
C. benign cementoblastoma

98. What is used in fungal infection:
a. Nystatin. ***
b. fluconazole
c. Amphotericin B
All used for treatment of fungal infection, but Nystatin can be used as initial treatment
http://web.squ.edu.om/med-
Lib/MED_CD/E_CDs/Essential%20of%20Oral%20Medicine/docs/ch18.pdf

99. Systemic antifungal drug
A. Nystatin
B. fluconazole ***
C. amphotericin
www.ncbi.nlm.nih.gov%2Fm%2Fpubmed%2F7498014%2F&h=aAQHWN-hS

100. Treatment of Grade II furcation involvement:
a. Scaling, root planning, bone grafts with guided tissue regeneration GTR. ***

101. Decrease the polymerization shrinkage of composite by :
a. Incremental placement with increase time of curing.
b. Incremental placement with high intensity light cure. ***

102. Which of these is used for gingival contouring (gingivectomy):
a. PK.2
b. PK.4
c. Bard Parker. *** (surgical blades no. 11 & 12)

103. In children pulp damage is less frequent than in adults due to:
a. Minor subluxation does not cut the blood supply. ***
b. More hemoglobin content in children.
c. Less nutritional deficiencies.

104. Fracture of tooth in maxilla:
1. 45%
2. 25%
3. 75% ***
Dental fractures are commonly observed with other oral injuries.
Approximately 82% of traumatized teeth are maxillary teeth.
Fractures to the maxillary teeth are distributed among the central incisors (64%), lateral
incisors (15%), canines (3%).

105. Blade of PDL instrument should be:
A. perpendicular to long access
B. parallel to long access
C. perpendicular to shank ***

106. Patient has complete denture and has pain and erythema ( fissured ) at the
angle of mouth :
A. angular cheilitis ***
B. denture sore mouth

107. When dentist leave the canal open without dressing this will lead to:
A. decrease the pain
B. contamination of RCT system ***
C. healing of periapical tissue

108. Pt. has ceramic crown need endodontic surgery best approach:
A. semilunar ***
B. parasemilunar
C. submarginal
D. sulcular
109. Female patient in 50's has facial swelling related to badly carious lower molar,
need RCT and has allergy to penicillin. What to do:
a. Give erythromycin then RCT ***
b. Start RCT then give erythromycin
c. 3 & 4 was about amoxicillin so surely wrong.

110. What is the name of the retractor which retract the flap and the cheek
together when doing surgery:
1. farabeuf retractor ***
2. longneck retractor

111. Migration of 1
st
permanent molar following the premature loss of the
deciduous 2nd molar is usually:
A. Mesial with mesio buccal cusp rotating lingually ***
B. Mesial with mesio buccal cusp rotating buccally
C. Mesial with buccal tilt of the crown
D. Not found

112. Patient with gingivectomy surgery. After surgery, xenograft was placed with
bioresorbable sutures placed. Which dressing is placed over it?
a. Eugenol dressing
b. Non-eugenol based***
c. Antibiotic dressing

113. Rarefaction (rarefurcation)occurs in which areas:
a. Areas denuded from bone***
b. Areas of fracture
Rarefaction: is a decreased density of bone such as a decrease in weight per unit of
volume
Systemic diseases causing generalized jaw rarefaction
Osteoporosis - Rickets/Osteomalacia -Sickle Cell Anemia Cushing syndrome
Hypophosphatasia - Hypophosphatemia

114. Spedding principle:
a. Used for selection of stainless steel crowns ***
B. Used for selection of restorations
C. Used for selection of shade
D. Used for selection of sealant

115. Ortho brackets that have suffer from deformation:
a. Ceramic brackets
b. Metal brackets
c. Self-ligating
d. Plastic ***
http://www.aapd.org/assets/1/25/Fong1-02.pdf

116. Patient with maxillary complete denture and natural mandibular anterior
teeth. After years, comes to you to do distal extension for posterior mandibular teeth.
You will see:
a. Bone resorption on maxillary anterior region***
b. Severe bone loss of mandibular posterior teeth

Dental decks

117. Patient with gold crown suffers pain and sensitivity on chewing only. What is
the cause:
a. Thermal conductivity of gold
b. Occlusal trauma***
http://www.onedollardentist.com/dentalcrownprocedure.html

118. Fluoride toxicity for 60 kg adult patient:
OR
What's The lethal dose of fluoride for 60 Kg pt. :
a. 50-60 mg/kg
b. 32-56 mg/kg ***
c. 40-70 mg/kg
The lethal dose for most adult humans is estimated at 5 to 10 g (which is equivalent to
32 to 64 mg/kg elemental fluoride/kg body weight)

119. Reduction of cusps for (tipping cusps) in complex amalgam restorations:
OR
With Complex amalgam how many reduction of cusp:
1. 1-2
2. 1.5 - 2
3. 2-3 mm ***
4. 5mm
http://www.uiowa.edu/~op2l/CARHandout.htm

120. Patient with old mandibular denture suffers now pain and paraesthesia in
lower lip, Cause is:
a. Pressure on mandibular nerve*** (Mental N. branch)
b. Vitamin b deficiency
http://books.google.com.sa/books?id=525T7rw_M4IC&pg=PA744&lpg=PA744&dq=caus
es+of+paresthesia+denture+wearers&source=bl&ots=ulj5XY7vS2&sig=OxaPxuGDYx5WH
RnugpzAvs4mahw&hl=ar&sa=X&ei=X2zYUquxNMSI0AWo5YDoCA&ved=0CEoQ6AEwAg#
v=onepage&q=causes%20of%20paresthesia%20denture%20wearers&f=false

121. Irreversible pulp is:
a. Normal
b. Necrotic
c. Vital
d. Vital but not normal***

122. Loss of taste in the anterior tongue is related to
a. Facial nerve (VII) ****
b. Hypoglossal nerve
c. Glossopharyngeal
d. Vagus
Due to chorda tympani nerve branch of Facial n, that give taste to Ant. 2/3 of the tongue
accompany with lingual n. branch of Mandible N.

123. Diagnosis for incipient proximal caries in primary teeth is by:
a. Tactile
b. Visual examination **
c. Radiograph
If fiberoptic option present its better than bitewing radiograph

124. Palatal gingiva supplied by:
OR
What innervates the anterior palatal area:
a. Grater palatine
b. Nasopalatine ***
c. infraorbital
d. superior anterior alveolar
125. Nerve for anterior gingiva of maxillary teeth:
a. Nasopalatine
b. Greater palatine
c. Posterior superior alveolar nerve

( . )


1. None of the above
2.Infra orbital n
3. Ant. superior alveolar n
Churchill's Pocketbook Clinical Dentistry 3Ed 2007, Page 97-98-99
Nasopalatine n: supply palatal mucoperiosteum of ant. Hard palate related to upper
canine & incisor
greater palatine n: supply mucoperiosteum up to canine region Anterior to this, the
innervation is derived from the Nasopalatine n.
Post. Superior alveolar n: supply upper molars & tissue related to it (gingiva-buccal
mucosa of molars and Max.sinus)
Infraorbital N: supply upper lip, cheek, side of nose and lower eyelid , buccal gingiva and
sulcus from midline to premolar region
incisor-canine & premolars (Ant. & middle superior alveolar nerve arise from the infra
orbital nerve in the infra orbital canal.
http://www.one-dent.com/foundations_of_periodontics/nerve-supply-to-the-
periodontium/

126. Radiograph for disk perforation:
a. MRI
b. Arthrography***
c. CT
Arthrogram: X-ray sensitive dye is injected into the TMJ joint capsule and x-rays of the
TMJ are taken
This test to determine disc dislocation and disc degeneration or perforation. Sometimes
in cases of mild disc dislocations, this test may be therapeutic in itself as the fluid dye
allows the disc to "float" back into place.
http://www.oralsurgerysandiego.com/tmj.htm

127. Space maintainer to replace premature loss of second primary teeth without
eruption of first permanent molar:
a. Distal shoe ***
b. Band and loop
c. Lingual arch
Distal shoe appliance used when 2
nd
primary molar loss and 1
st
permanent molar has not
erupted
http://www.slideshare.net/makkahguys/space-maintainer

128. Gates Glidden:
a. Has numbers 6-9
b. For coronal preparation ***
c. Breaks if stuck
D. for root preparation
http://quizlet.com/12701481/endodontics-flash-cards/

129. What part of RPD that provides occlusal support and prevents movement from
tissue:
a. Direct retainer
b. Rest ***
Churchill's Pocketbook Clinical Dentistry 3Ed 2007,Page 317-318
Rest: provide support and transfer load to the tooth.
Also, required to minimize rotational axe.
PRD requires support usually provided by an occlusal rest
http://www.slideshare.net/ammar905/removable-partial-denture

130. On intracanal instruments taper 2 means:
a. 0.02 increase in diameter between each instrument
b. Amount of increase in diameter ***

131. Function appliance:
a. bionator***
b. Head gear
c. reverse activated headgear
d. Face shell
e. Posterior Bite Plane
First AID NBDE II
Functional appliances can be categorized as:
A. Tooth-borne appliance:
1.Bionator: Advances the mandible to an edge-to-edge position to
Stimulate mandibular growth for correction of class II malocclusion.
2. Herbst: Maxillary and mandibular framework splinted together via
Pin and tube device that holds the mandible forward
B. Tissue-borne appliance:
The Frankel functional appliance is the only tissue-borne appliance.
It alters both mandibular posture and contour of facial soft tissue

132. Pt with wide cleft lip and palate lip adhesion or nasoalveolar molding planned:
1. Few week after birth
2. First third month*****
3. Third to sixth
4. 6 9
http://www.ncbi.nlm.nih.gov/pmc/articles/PMC2884751/

133. Patient complains from cold the best test is:
a. Electric test
b. Cold test***
c. Percussion test
d. Periodontal probe
http://www.hindawi.com/journals/ijd/2009/365785/

134. Age at which crown formation of 3
rd
molar is finished:
a. 10
b. 13****
c. 17
d. 19
crown completed formation of Upper & Lower 3
rd
molar : 12-16 yrs.
http://www.angle.org/doi/pdf/10.1043/0003-
3219(1962)032%3C0270:TMFAID%3E2.0.CO%3B2

135. Characteristics of permanent maxillary central incisor:
a. Wider mesiodistally than buccolingually***
b. Cingulum in middle third
c. Mesial side is more rounded than distal

Wheeler's Dental Anatomy Physiology & Occlusion 9Ed 2010, Page 99
a. The maxillary central incisor is the widest mesiodistally of any of the anterior teeth
b. The mesial outline of the crown is only slightly convex with the crest of curvature. The
crown is looks wedge-shaped or triangular
c. below the cervical line a smooth convexity is Cingulum
http://books.google.com.eg/books?id=pJokhKIHRKQC&pg=PA111&lpg=PA111&dq=Char
acteristics+of+permanent+maxillary+central+incisor&source=bl&ots=WF2qlbOphe&sig=
BZZgv5gtGVF1DNQLgYPk9_edofI&hl=ar&sa=X&ei=OOvXUpHFFKbqywOruoL4Bg&ved=0C
DAQ6AEwAA#v=onepage&q=Characteristics%20of%20permanent%20maxillary%20centr
al%20incisor&f=false

136. After extraction, the dentist recommended immediate denture. purpose:
a. To preserve bone resorption **
http://www.slideshare.net/shabeelpn/immediate-denture

137. After plaque accumulate on the tooth and colonizes what happens:
a. Proliferation
b. Maturation ***
Dental Decks 2012, Page 1021

138. Why does the technician use a movable die on the master cast
a. To allow expansion
b. To prevent expansion
c. To help carving of wax and finishing of crown ***

139. The dentist needs local anesthesia of 4%with 1:200,000 what color coded
cartridge should he ask the assistant to give him:
a. Red
b. Blue
c. Green
d. Yellow ***
Lidocaine 2% with Epinephrine 1:100,000 Red
Lidocaine 2% with Epinephrine 1:50,000 Green
Lidocaine Plain Light Blue
Mepivacaine 2% with Levonordefrin 1:20,000 Brown
Mepivacaine 3% Plain Tan
Prilocaine 4% with Epinephrine 1:200,000 Yellow
Prilocaine 4% Plain Black
Bupivacaine 0.5% with Epinephrine1:200,000 Blue
Articaine 4% with Epinephrine 1:100,00 Gold
Articaine 4% with Epinephrine 1:200,000 Silver

140. Distance Between patient And Cephalometric:
OR
the distance of cephalometec x-ray source from the patient:
OR
When take cephalometric x-ray we should stand:
a. 3 feet
b. 4 feet
c. 5 feet *** (150 cm)
d. 6 feet (with angle of 90-135 degree)
http://www.answers.com/topic/cephalometric-radiograph

141. Longest & sharpest cusp of teeth:
1. Lingual of deciduous. *** (Mesiolingual of 1
st
Molar)
2. buccal of premolar
3. Lingual of premolar.
Wheeler's Dental Anatomy Physiology & Occlusion 9Ed 2010, Page 58
In Primary teeth:
Mesiolingual cusp of Maxillary 1
st
molar is the largest and longest and sharpest.
In Permanent teeth:
Max. 1st Molar: Mesiolingual cusp is the longest and sharpest and wider buccolingually
than mesiodistally
Man. 1st Molar: Mesiobuccal cusp is the widest cusp.
and wider mesiodistally than buccolingually.

142. Delayed eruption of upper permanent incisor in child 8 yrs. on radiograph
show mixed radiopaque with radiolucent area:
A. compound odntoma***
B. complex odontoma
C. calcifying epithelial odontogenic tumor
D. adenomatoid odontogenic tumor
Compound odontomas: the common in all odontogenic tumor commonly occur in the
incisor-canine region of the (anterior maxilla.)
Complex odontomas: are frequently located in the premolar and molar region of
mandible then anterior maxilla
http://www.aapd.org/assets/1/25/deOlivera-23-02.pdf
Calcifying epithelial odontogenic tumor: rare only type 2 occur in ant maxilla as a soft
tissue mass
http://en.wikipedia.org/wiki/Calcifying_epithelial_odontogenic_tumor
Adenomatoid odontogenic tumor: located in the anterior maxilla and in the anterior
mandible. impacted tooth (usually the canine) sometimes misdiagnosed as a cyst.
http://en.wikipedia.org/wiki/Adenomatoid_odontogenic_tumor

143. A crown margin can be extended subgingivally when required:
A. for esthetics.
B. to increase retention.
C. to reach sound tooth structure
D. all of the above. ***

144. in deep carious lesion, a pin point pulp exposure occur with excessive salivary
flow but dentist use a rubber dam in this condition the treatment will be:
a. pulpotomy
b. pulpectomy
c. direct pulp capping ***
d. indirect pulp capping

145. What's the artery that supply the upper lip:
A. superior labial artery ***
It is an artery that is a branch of the facial artery that supplies the upper lip

146. Patient with hypertension over 10 years ago, in mouth we will find:
A. fungal infection
B. mucosal changes ***
C. cellulitis
http://www.ncbi.nlm.nih.gov/pmc/articles/PMC3424937/

147. how many weeks for complete tissue regeneration:
a. 8 - 12 weeks ***

148. Fluoride containing toothpaste for children:
A. recommended ***
B. useful
C. toxic
D. not advisable
http://www.ada.org/2684.aspx

149. Primary goal in treatment plan:
A. restore all carious lesion
B. eliminate iatrogenic plaque in all patients
c. detect incipient caries and high risk patient
D. complete elimination of plaque from all patients ***

150. External resorption of pulp found:
OR
What condition of pulp in case of external resorption:
OR
External resorption effect on pulp:
A. Normal
B. Reversible pulpitis
C. Irreversible pulpitis
D. Necrosis ***
External resorption is usually traumatic resorption in permanent teeth and usually
associated with necrotic pulp tissues ((((Non-vital tooth)))
151. Internal resorption of pulp:
a. normal
B. reversible pulpitis
C. Irreversible pulpitis ***
D. necrosis
http://www.endoexperience.com/documents/ecir.pdf

152. Female come with radiographic short gutta percha, the dentist remove the old
gutta percha and decided to re-do RCT then he can't find canal again, why:
A. perforation
B. ledge ***
C. lateral canal

153. In short crown preparation last thing to do to increase retention and stability
A. use zinc phosphate cement
B. retentive groove ***
C. decrease occlusal carving
d. complete bevel shoulder
http://www.medicinaoral.com/odo/volumenes/v4i4/jcedv4i4p230.pdf

154. Which is the sharpest and longest cusp:
A. buccal cusp of primary tooth
B. lingual cusp of primary tooth***
C. buccal cusp of permanent tooth
D. lingual cusp of permanent tooth
Wheeler's Dental Anatomy Physiology & Occlusion 9Ed 2010, Page 63
The mesiolingual cusp is long and sharp at the tip, more so than any of the other cusps
on deciduous mandible 1
st
molar.

155. Force of removable appliance:
OR
Function of removable appliance:
A. torque
B. tipping ***
C. shearing
D. rotation

156. Pedo pt. with extremely ve behavior, to restrain the extremity:
A. use mouth prope.
B. Belt.
C. Board. ***

157. Whats the mainly factors that effect in healing:
A. intraossoius healing
B. connective tissue healing
C. inflammatory cells ***
D. inflammatory fluids

158. The lowest level of fluoride in drinking water which will cause enamel mottling
is:
A. 0.5ppm.
B. 1.0ppm.
C. 3.0ppm. ***
D. 5.0ppm

159. Child have a dog bite and his father take him to dental clinic, dog bite him in
right side and take tetanus injection, whats your opinion in treatment :
A. wash his face by sodium peroxide
B. leave it without any thing and healing occur
C. make a suture ***
D. put iodine
E. Irrigate the side with NaocL
Dog bites on the face may be sutured to prevent visible scars
http://pets.webmd.com/dogs/dog-bites


160. The cell from which of the following tissues should reach the healing area first
for the new attachment to occur:
a. Gingival epithelium
b. Gingival C.T
c. Periodontal ligament (PL) ***
d. Alveolar bone
Guided tissue regeneration "GTR": is based on the assumption that only the periodontal
ligament cells have the potential for regeneration of the attachment apparatus (new
attachment)

161. How can be standardized for intracanal instrument
A. length of the blade
B. width of tip of the blade ***
C. length of the hand
D. size of the hand

162. Patient has missing #45 need restoration:
a. Maryland bridge
b. FPD
C. implant ***

45 = Lower right 2
nd
premolar

163. Which deciduous tooth cause crowding in lower anterior region if early
extracted:
a. Primary mandibular first molar ***
b. Primary mandibular second molar
C. The lower lateral primary incisor

164. Best tooth responsible for crowding of lower arch:
a. D
b. E***
c. Upper 6
d. Lower 6
165. Most teeth responsible for crowding is:
a. Lower E ***

166. Early loss cause crowding in anterior teeth due to:
a. primary 1
st
molar ***
b. primary 2
nd
molar
c. primary canine


167. Flux used for
A. prevent oxygen from contacting alloy
b. dissolve oxide
c. a and b****

168. Patient has upper #6 RCT with small MOD caries, best treatment:
a. MOD gold inlay
b. MOD gold onlay
C. Gold crown ***
Onlay to protect the cusp

169. Whats the best material under composite:
A. Ca(oH)2 ***
b. calcium hydrate
C. zinc oxide and eugenol
D. zinc phosphate

170. Patient diabetic, upper #5 have amalgam MOD, and have mobility grade II,
calculus quit, and want to make artificial for missing tooth:
a. FPD with tooth supported
B. extraction and make RPD ******
C. make occlusal rest and make RPD
D. remove it and make implant
E. resin bounded tooth supported

171. What is the active appliance:
A. headgear
B. reverse pull head gear
C. posterior bite plate*****
Active appliances employ force to the teeth to change their position
Most active appliances are fixed. Examples of active appliances include:
Rapid maxillary expansion appliance (palatal expander)
Helix
Bite plate
d
e
Pin and tube
Ribbon arch
Edgewise
Beg light wire
http://www.aboutbraces.com.au/types/orthodontic-appliances/

172. Patient had xerostomia need restoration:
a. FPD
b. RPD
C. implant ***
http://www.beachdentistry.com/dry-mouth-how-to-cope-with-it/

173. Cyst acc to WHO recent classification become tumor:
A. adenamatoid odontogenic cyst
b. Keratocyst ***
C. Radicular
D. Glandular cyst
In 2005 the world health organization (WHO) classified
The odontogenic keratocyst" as the keratocystic odontoginic tumor
Textbook of Oral and Maxillofacial Surgery
http://books.google.com.eg/books?id=s6uuSx7gsbkC&pg=PA573

174. Pt. treatment radiotherapy from 10 months to make impression of removable
partial denture what is type of material
A. plaster of Paris
b. Elastomers
C. compound
d. ZOE
e. alginate ***
F. agar-agar

175. Must in concern:
A. metaplasia
B. hyperplasia
C. dysplasia ***
Metaplasia: an initial change from normal cells to a different cell type (such as chronic
irritation of cigarette smoke causing ciliated pseudo stratified epithelium to be replaced
by squamous epithelium more able to withstand the insult).
Dysplasia: an increasing degree of disordered growth or maturation of the tissue (often
thought to precede neoplasia) such as cervical dysplasia as a result of human
papillomavirus infection. Dysplasia is still a reversible process. However, once the
transformation to neoplasia has been made, the process is not reversible

176. GI cement used under large composite restoration for:
A. compressive strength
B. prevent micro leakage ***
c. increase resins
D. increase elasticity of restoration


177. pt. came to your clinic with pain in his mouth but he can not localize which the
jaw, which test is useful:
a. Thermal test
b. Percussion test
C. anesthetic test ***
d. Cavity test

178. Over extended GP should remove using:
A. ultrasonic vibrating.
b. Dissolving agent. ***
c. Rotary or round bur
d. Surgery

179. soft tissue protrusion in:
a- class I
b- Class II
c- Class III
d- Class II mod I ***
180. The bacteria that affects cardiac valves
A. streptococcus viridans
B. staphylococcus aureus ***
C. staphylococcus epidermidis
D. Streprtococcus sanguinis
Infective endocarditis is the inflammation of heart valves and chambers
181. Bacteria in infective endocarditis:
OR
Infective Endocarditis bacteria:
A. streptococcus Virden's******
B. streptococcus. Salivaris
Styphalococcous aureus is the most common
streptococcus viridans is present 50-60 consider 2ns most common
www.en.wikipedia.org/wiki/Infective_endocarditis

182. Muscle for control cleft palate:
A. elevator palatine*****
B. tensor palatine
http://emedicine.medscape.com/article/1280866-overview

183. Minimum duration for antibiotic in days: (Antibiotic course)
A. 3 ***
b. 5
c. 7
d. 10
http://www.ncbi.nlm.nih.gov/pubmed/19931818

184. which of these luting materials if mixing slow give best working consistency &
most irritation to vital pulp :
1. ZOE
2. Zinc phosphate ***
3. GIC
4. Resin

185. Antibiotic for empiric treatment in end stage renal deficiency is:
A. Flagyl
B. Doxycycline ***
C. Clindamycin
D. Penicillin

186. In order to treat ANUG all causative microorganisms must be eliminated.
ANUG if left untreated will lead to ANUP.
1. Both correct ***
2. First correct second false
3. First false second correct
4. Both false

187. Your treatment was scaling for facial should be angle:
a. 40-60
b. 60-70
c. 70-80 ***
d. 80-90

188. A child with fracture of tooth at the apical third of the root, what your first
decision would be
A. Wait and recall after one month and observe for any necrotic or radiolucency ***
B. Root canal treatment
C. Extraction
D. Apiectomy

189. Patient with pain on the upper right area, and the patient cannot tell the tooth
causes the pain, what is the best reliable way to do test pulp:
1. Thermal test.
2. Electric test.
3. Stimulation the dentine.
4. Anesthesia test
5. Cavity test ***

190. Patient with pain on the upper right area, and the patient can not tell the
tooth causes the pain, what is the least reliable way to do test pulp:
1. Cold test.
2. Hot test.
3. Electric test. ***
4. Stimulation the dentine.

191. Class V composite resin restorations can be polished
A. 24 hours after application
B. Immediately after application ***
C. 3 to 4 days
D. 3 to 4 weeks
E. Not at all
192. bilateral infection in submental , submandibular and sublingual triangles with
difficulty in breathing and swallowing is
1. Chronic periapical abscess
2. Ludwig angina*****

193. The dentine bond strength of 5th generation adhesives:
1. 5
2. 10
3. 15
4. 30 ***
In 5th generation:
Dentin bond strength: 35 MPa
Enamel bond Strength 25 MPa

194. Teeth with RCT and you want to use post, which post is the least cause to
tooth fracture :
a. Ready-made post.
b. Casted post.
c. Fiber post. ***
d. Prefabricated post.

195. The most ideal film thickness cement:
a. Zinc phosphate ***
b. Zinc polycarboxilate
c. GIC
d. Resin cement.

196. Mesiobuccal root of maxillary first molars MOST COMMONLY have:
A. One canal with one foreman ***
B. One or two canals with one foreman
C. Two canals with one foreman
D. Two canals with two foremen
Dental Decks Part II 2011-20012, Endodontic section, Page 4 (168)


197. Sjogren syndrome is characterized by:
A. Dryness of the mouth
B. Dryness of the eyes
C. Rheumatoid arthritis
D. All of the above ***

198. Marsupialization is a technique used in the treatment of:
A. Pericoronitis
B. Cysts ***
C. Abscesses

199. The most common characteristic symptom of malignant tumors occurring in
lower jaw is:
A. Pain
B. Bleeding
C. Paresthesia ***

200. How would you diagnose a periapical abscess:
A. Pain on percussion ***
B. Pain when eating hot food
C. Pain when eating cold food
D. The thickness of periodontal ligament on X-Ray


201. Female teeth eruption before the male by:
a. 5 ***
b. 7
c. 9
d. 11 months

202. The most common injures in child is:
A. tooth
B. root
C. intrusion of the tooth inside the socket well ***

203. A rigid part of the partial denture casting that unites the rests and another
part of the prosthesis to the opposite side of the arch is called:
a. Minor connecter.
b. Major connecter. ***
c. Retainer.
d. Rest.

204. the primary choose for antibiotic in dental clinic is:
A. penicillin v***
B. amoxicillin ****
C. cephalosporin
D. amoxicillin with clavolanic acid

205. Focal infection theory...:
A. generalized infection
B. local infection ****
C. external trauma
D. autoimmune

206. Flat alveolar ridge we use:
A. 0 degree teeth ***
B. 20 degree teeth
C. 30 degree teeth
D. 40 degree teeth
207. The posterior part of the tongue supply by:
a. Glossopharyngeal ***

208. Facial nerve supply which muscle:
1. Masseter
2. Temporalis
3. Buccinators ***

209. Fracture containing E+D+cementum:
A. crown root fracture ****
B. complicated crown fracture
C. root fracture

210. The needle holder used in suturing of lower third molar:
A. Curved hemostat. ***
B. Allis forceps.
C. Adson forceps.
D. Regular tweezers.

211. In case of fracture of the ramus of the mandible, to evaluate if fracture
favorable or unfavorable:
1. Panoramic radiograph
2. 30 degree oblique radiograph ***
3. Ap radiograph
4. submentovertix radiograph

212. Orthognathic surgery, you plan to use 2 mm screw the drill size:
A. 1 mm
B. 1.5 mm***
C. 2 mm
D. 2.5 mm
http://www.synthes.com/MediaBin/International%20DATA/036.000.413.pdf

213. Whats the ratio:



Oral cancer Control
Smoker 90 10
Non smoker 40 60



a. 6
b. 2.35
c. 3.43
d. 13.5 ***
Odds ratio: 90 *60 / 10*40= 5400/400 = 13.5
Explanation:
In statistics, the odds ratio is one of three main ways to quantify how strongly the
presence or absence of property A is associated with the presence or absence of
property B in a given population. Wikipedia

214. Patient comes with pain but he can't localize the tooth what is the best test:
A. thermal test ***
B. percussion test
C. cavity test
D. anesthesia test

215. Which one is flexible:
a. K File
b. Reamers (K-flex file) ***
C. hedstorm (H file)
d. barbed broach
NiTi protaper k file is the most flexible file


216. The contraindication of functional appliance in class II Pt. :
OR
The contraindication of functional appliance in class II division (I):
a. Protracted up central incisors
B. retracted upper central incisor
c. Protracted lower central incisor ***
D. retracted lower central incisor
functional
appliance

217. Rarefaction of bone diagnosis is by:
A. pain in the cold
B. pain by percussion
C. cortical bone resorption ***
D. radicular bone resorption

218. In case of fracture of the ramus of the mandible, to evaluate if fracture
favorable or unfavorable :
A. panoramic radiograph
B. 30 degree oblique radiograph ****
c. Ap radiograph
D. submentovertix radiograph
Simple fractures of the mandibular angle and ramus enlarge Routine diagnosis of this
type of fracture should include x-rays taken in two planes at 90 to each other. The
minimum requirement is a PA view and a panoramic view. CT or digital volume
tomography (DVT) imaging may be used as an alternative
https://www2.aofoundation.org/wps/portal/!ut/p/c1/04_SB8K8xLLM9MSSzPy8xBz9CP0os3hng7BAR
ydDRwN3Q1dDA08XN59Qz8AAQwMDA6B8JJK8haGFgYFnqKezn7GTH1DahIBuP4_83FT9gtyIcgBttnJy/d
l2/d1/L2dJQSEvUUt3QS9ZQnB3LzZfQzBWUUFCMUEwRzFFMTBJREZMVUlRUDEwMDA!/?segment=Ma
ndible&bone=CMF&showPage=diagnosis
Mandible - Diagnosis - AO Surgery Reference
www2.aofoundation.org
http://www.slideshare.net/tongmd/mandibular-fractures-5798481


219. What the pH the enamel make complete remineralization:
a. 3.5
b. 4.5
c. 5.5
d. 6.5 ***
When the pH returns to pH 5.5 or above, the saliva which is supersaturated with calcium
and phosphate, forces mineral back into the tooth.7 Fluoride adsorbs to the surface of
the partially demineralized crystals and attracts calcium ions. This new surface veneer
takes up fluoride preferentially from the solution around the crystals and excludes
carbonate
It is called Stefan curve

220. Enamel is not or less mineralized to:
a. formative cells dead or disappear after eruption *
b. Large content of organic particles
c. Small content of organic particles
d. Large amount of inorganic mineral

221. Muscle that control in cleft palate:
A. tensor
B. elevator ***

222. Amalgam restoration when restore a cusp it should be shorter than normal:
A. 2mm
B. 1.5mm ***
C. 3mm
D. 4 mm
223. Best diagnosis of root fracture involved in place of fracture during healing:
A. interproximal connective tissue
B. interproximal bone
C. interproximal connective tissue and bone ***
D. inflammation tissue

224. The sequence in deep carious lesion close to pulp are
a. GI base, varnish, caoh2
b. Varnish, GI base, caoh2
c. caoh2, GI base, varnish***
d. caoh2, varnish, GI base.

225. Post should set passively in root canal and crown should set with slight
resistance:
a. Both statement are false. ***
b. Both statement are true.
c. First statement is true and second is false.
d. First statement is false and second statement is true

226. Fastest route of absorption is:
A. inhalation ***
B. intravenous (IV)
C. oral

227. Onlay restoration:
A. larger than amalgam restoration
B. smaller than amalgam
C. same as amalgam
D. depend on caries extension ***

228. Muscle that pulls lower jaw upward and backward:
a. Temporalis ***
b. Masseter
c. Lateral ptyregoid

229. A reline for a complete denture is contraindicated when:
a. There is extreme over closure of the vertical dimension. ***
b. Centric occlusion and centric relation do not coincide.
c. The denture contains a broken tooth.
d. There is resorption of the ridge.

230. Pt 45 yrs. female with upper PRD denture come complaining from
unstability but when dr. put her finger on palatal...dentur stable from buccal extention
and teeth without caries but clasp not good tr.is:
a. ...relining... ***
B. rebase.
c. New denture


231. Fractured tooth to alveolar crest, what is the best way to produce ferrule
effect:
A. restore with amalgam core sub-gingival.
B. crown lengthening
C. extrusion with orthodontics ***
232. The percentage of total dentine surface dentinal tubules make in 0.5mm away
from pulp is:
A. 20%
B. 50% ***



233. Pt. has fluoride toxicity, what should dentist do:
a. make pt. gaging and give enough calcium (milk) to inhibit fluorite work ***
Also giving aluminum Hydroxide will form & inhibit fluorite activity
234. Which muscle that elevate the mandible jaw:
a. All of below ***
b. masseter
c. temporalis
d. medial pterygoid

235. A Simple orthodontic case that could treated within 7 9 yrs should be:
a. Simple crowded
b. Protrude
c. Cross bite ***

236. The mandible originate from which Arch in embryo:
a. 1
st
Arch ***
b. 2
nd
Arch
c. 3
rd
Arch

237. How many duration of GTR (barrier membrane):
A. 1 Week
B. 1 month
C. 3 month
D. 6 month ***

238. What is the main content of para nasal fluid:
a. Highly glucose content ***

239. The growth spurt between boys & girls is:
OR
Difference in puperty between boy and girl:
a. 3 months
b. 6 months
c. 1 year
d. 2 years ***
Girls = 11-15
Boys= 12-16

240. Varnish in class I cavity in permanent lower molar is used to:
A. protect the pulp
B. prevent discoloration ***

241. Adult 21 yrs. do exo to 3
rd
molar and return after 2 days with swelling in right
& left of the jaw:
a.Ostyiomlitis
b. Ludwig's angina ***

242. Old patient has pain in lower premolar by radiograph you found that the canal
disappear at mid root this is due to:
A. calcification of canal
b. Two canal are present ***
Because calcification start from coronal to apical so if the canal patent coronal it's not a
calcification but usually the canal divide in to two canals buccal and lingual which do not
appear in 2D x-Ray

243. Patient come to you after few days from putting upper and lower complete
dentures and suffering from redness and pain, he is Asthma patient, what is the patient
sense:
A. Pt. used antihistamine ***


244. The bone graft took from the same person called:
a. Autograft ***

245. Pt came 2 u with coloration bluish (or green) and black in the gingival
margins, he said that he has gasteriointensinal problem. this is caused because of:
A. mercury
B. lead
C. bismuth. ***
D. arson.

246. Pt. come to you suffering of palatal cleft in which age will that happen:
a. in 8
th
weeks ***

247. First thing to be checked during metallic crown try in:
A. rocking
B. fitting of inner surface ***
C. margin.

248. Child 6 year have abnormal enamel dentin and pulp in A quadrant you
diagnosis is:
a. hypoplasia
B. regional odontplasia ***
c. Detogensis imperficta
d. Amelogensis imperficta

249. Trauma caused fracture of the root at junction between middle and cervical
third:
1. Do endo for coronal part only
2. RCT for both
3. Leave
4. Splint the two parts together ***

250. After final inlay cementation and before complete setting of cement we
should:
A. remove occlusal interferences
B. burnishing of peripheries of restoration for more adaptation. ***
C. lowering occlusal surface

251. Galvanic shock:
a. Put separating medium.
b. Wait. ***
C. put varnish.

252. Pt. has high mastication forces has caries on posterior teeth and he want only
esthetic restoration:
a. Composite with beveled margins.
b. Composite without beveled margins. ***
c. Light cured GI.
d. Zinc phosphate cement.

253. Three weeks after insertion of FPD marked discomfort to heat and cold occurs
there are no other symptoms. The most likely cause is:
A. gingival recession
B. unseating of FPD will cause pain on chewing
C. deflective occlusal contact ***
D. torsional forces on the abutment tooth
E. incomplete coverage of cut surfaces of prepared abutment teeth

254. Nerve responsible for pain:
A. Myelinated
B. Unmyelinated ***
First Aid, Page 89
Small, Unmyelinated nerve fiber (Pain, temperature, autonomics) are more sensitive to
local anesthesia than larger, myelinated nerve fiber

255. Which not compatible to the pulp:
A. GIC
B. Zinc phosphate cement ***
C. Zinc polycarboxylate cement
GIC for vital tooth and ZnP for non-vital tooth

256. The retention Pin in an amalgam restoration should be placed:
A. Parallel to the outer wall***
B. Parallel to the long axis of tooth

257. The retention of a pin:
A. increase as the diameter of the pin increase. ***
b. Increase as the diameter of the pin decrease.
c. Decrease as the diameter of the pin increase.
d. The retention of a pin has nothing to do with the diameter of the pin. ** In 5th
version

258. Cell life cycle phases:
a. 2
b. 3
c. 4
d. 5***

259. Osteogenesis imperfecta also provides an illustration of variable gene
expression The cardinal signs of this disease are:
1. Multiple fractures ***
2. Blue sclera
3. Dentinogenesis imperfecta
4. Otosclerosis
http://www.nature.com/gim/journal/v11/n6/full/gim200954a.html
http://books.google.com.sa/books?id=YmQ3GGGjDhMC&pg=PA755&lpg=PA754&ots=5
LpJwzDiJp&focus=viewport&dq=cardinal+feature+of+osteogenesis+imperfecta&output=
html_text

260. Patient has pain and he can't differentiate which the jaw in pain:
A. electrical test
B. thermal test
C. anesthesia ***
D. cavity test

261. The shape of open access in the 1
st
mandibular molar and its base should be:
A. Triangular and the base toward mesial ***

262. Patient during operation has hypotension and loss conscious:
A. upright position and his leg in the floor
B. supine position and his leg in the floor
C. flat position and his leg at the same of his heart
D. flat and leg above the level of his heart ***

263. 10 yrs. old child has bilateral mandibular swelling:
OR
Child has bilateral swelling in mandibular, In x-ray multi-radiolucent area:
OR
Patient 10 yrs. old with bilateral submandibular (multilocular) swelling which push
lower 8,7 anteriorly:
OR
9 yrs. old patient with bilateral swelling in the mandible (multiloucular):
A. cherubism ***
B. ameloblastoma
b. osteomyelitis
c. OKC


264. Relation between enamel and cementum:
A. cementum is slightly above (Overlap) enamel ***
B. enamel is slightly above cementum
C. Enamel and cementum have sharp contact
D. no junction between enamel and cementum

There exists a normal variation in the relationship of the cementum and the enamel at the
cementoenamel junction. In about 6065% of teeth, the cementum overlaps the enamel
at the CEJ, while in about 30% of teeth, the cementum and enamel abut each other with
no overlap. In only 510% of teeth, there is a space between the enamel and the
cementum at which the underlying dentin is exposed.

265. Which one of the following are not used in water fluoridation:
1. SnF2 ***
2. 1.23% APF
3. CaSiF2
4. 8% Stannous fluoride

266. In casting the substructure for a metal ceramic restoration it is necessary to
use:
1- Gypsum-bonded investment
2- Phosphate- bonded investment ***
3- Gas-air torch
4- Crucible, which has not been used for other gold alloy
a. 1 and 3
b. 1 and 4
c. 2 only ***
d. 2 and 3
e. 2 and 4

267. Growth of mandible, In the area of condyle:
A. deposition, intramembranous
B. deposition, endochondral ***
C. instion, intramembranous
D. instion, endochondral

268. Child has red vestibule and drain sinus in mandibular carries molar:
A. pulp polyp
B. pulp pathosis ***
C. pulp stone
D. pulp calcified

269. Use of chin cup applies:
A. pseudo class 3
b. class3 with long lower face ***

270. What is the purpose of leveling the curve of Spee:
A. correct open bite
B. correct deep bite ***
C. correct angulation of the teeth
D. change arch diameter

271. Isolated miller upper canine has grad II recession:
A. full flap with full flap
B. full flap with strip flap
C. connective tissue graft with full flap
d. CT graft with strip flap ***

272. Cartilage when increased in growth we differentiate between it and bone by :
A. growth intensity
B. deposition ***

273. The primary goal of post is:
A. retention ***
B. indicated when soft tissue compromised

274. Improper use of dental floss:
A. lingual region
B. inter papillary trauma ***
C. buccal region

275. PT with upper complete denture complaining from poor retention & after
examination we found bubbles in posterior area of upper denture, WHY?
a. Underpostdamming ***
B. Over extension
C. Under extension
D. over post damming

276. Skeletal bone of skull develop from:
1. endochondral ossification
2. intramembranous ossification
3. a +b ***

277. the purpose of splint in MPDS:
A. release joint tension
B. release muscle tension ***
MPDS = Myofacial pain Dysfunction Syndrome

278. Pt came to clinic for construction of max complete denture during intraoral
exam u found severe anterior undercut and posteriorly lateral to max tuberosity what
should dentist do:
a. reduction of max anterior undercut and relief of post one
b. reduction of ant and post undercut ***
c. reduction of post and relief of ant
d. relief of both
Dental decks
279. Pt came to you need upper and lower complete denture after examination
you noticed that he has an anterior undercut + undercut in post. area near to tuberosity,
what you will do:
A. remove undercut from anterior area
B. remove undercut from post. area ***
C. remove under from post. Area & make reline in denture for anterior area
D. remove under from ant. Area & make reline in denture for post area

280. 21 year old pt. has submandibular space infection swelling in 36 area and 38 is
missing on radiograph he has radiolucency extend in 36, 37, 38 area with septal
tubercula and root resorption:
a. Kerato cyst
b. Ameloblastoma ***
c. Dentigerous cyst

Ameloblastoma = Unilocular swelling mostly

281. At age, 4 years a child took antibiotic. Which teeth are affected:
A. central and lateral incisor and 6th
B. central and lateral incisors only ***

282. What is the best material for direct pulp capping in pedo:
A. zinc oxide
B. formocresol ***
C. caoh
d. GIC
Ca(OH)2 is contra indicated in primary teeth. Direct Pulp capping indicated only if there
is small mechanical exposure.

283. Most desirable finished surface composite resin can be provided by:
A. white stone

Bilateral undercuts in the tuberosity area (or
anywhere) should also alert you for possible
surgery.
________________________
When there are multiple undercuts, posterior
undercuts are the (first) to be removed because
they can't be easily negotiated.
____________________
There is usually a maxillary anterior undercut.
This anterior undercut can be avoided by using
an anterior-posterior path of insertion of the
denture
B. hand instrument.
C. carbide finishing bur. ***
D. diamond finishing bur
E. celluloid matrix band

284. Patient has pain on chewing and cold will remove immediate after remove the
cause and has crack in mesial surface in the lower molar and no carries:
1. irreversible pulp and make RCT 1
2. Reversible pulp and make crown ***
3. Normal pulp and make mod amalgam
4. in x ray show radiolucent area in mesial root

285. Muscles of the tongue are:
a. 17 ***
b. 18
c. 19
d. 20

286. The technique to remove alginate impression:
A. gradually removal
B. snap motion ***
287. class I malocclusion pt. with high over bite and diastema he came to your clinic
to correct the diastema what's to do:
A. correct the over bite first ***
B. correct the diastema

288. Main use of dental floss:
a. Remove calculus.
b. Remove over hang.
c. Remove dental plaque. ***
d. Remove food debris

289. A child in early mixed dentition with transverse discrepancy and unilateral
posterior crossbite what do this child need:
A. headgear
B. skeletal expansion ***
C. dental expansion
D. wait until eruption of all permanent teeth

290. Imaging showing disk position and morphology and TMJ bone:
a. MRI ***
b. CT
c. Arthrography
D. plain radiograph.
E. plain tomography

291. Phosphoric acid is applied for:
1. 10-20 seconds ***
2. 30-45 seconds
3. 20-30 seconds
4. none

292. Pt need exo, he takes Antidepressant, amount of epinephrine on anesthesia:
a. 0.1 ***
b. 0.02
c. 0.4
d. 0.8
e. 10


293. Best treatment of choice for carious exposure of primary molar in 3year old
child who complain of toothache during and after food taking:
A. direct pulp capping with caoh
B. direct pulp capping with ZOE paste
C. formocresol pulpotomy ***
D. Caoh pulpotomy

294. Complete re-epithelization after surgery:
a. 1-5 days
b. 10-15 days***
c. 17-21 days
295. Fluorescent detect caries:
a. DIAGNODENT (a laser used for detecting caries)

296. Resorbable membrane need:
a. 4-12

297. Non resorbable:
a. 3-6

298. Best implant material is:
a. Titanium ***

299. Endo fail Restoration coz (sealer and GP) pass over apical foramen:
a. improper apical preparation ***
b. improper sealer
c. use spreader to apex
d. small master GP

300. serial extraction (hotz) is:
a. CD 4 ***

301. Which part of periodontal instrument is parallel to long axis of the tooth
A. hand
B. blade
C. shank ***
D. cutting edge

302. Patient has swelling in submandibular area increase with eating what is your
diagnosis:
A. Ranula
B. sialothisis ***
C. mucocele

303. Which type of anesthesia has the slowest onset:
a. lidocaine
b. mepivacaine***
c. bupivacaine & Ropivacaine (Longest)
d. Chloroprocaine (Safest & fastest onset)


304. Soft palate falls abruptly facilitate recording post dam, falls gradually make
recording post dam difficult:
A. both statement true
B. both false ***
c. First true and second false
d. First false and second true

305.
Smear layer how to prevent pulpal damage:
a. Prevent passage of irritants through dentinal tubules ***
b. Decrease the effect of acid etching on the pulp

306. Lymph node become peak at:
A. birth
B. early childhood
C. late childhood ***
D. early adult

307. Dentist after finishing preparation for full veneered crown but a groove on
buccal surface as a final step why:
a. Indicator for seating of crown
b. Anti-rotational in cementation ***
C. to prevent mesiodistal movement

308. COPD pt. for extraction of multiple teeth need:
a. 2.5\oxygen ***
B. 4 l
C. 6 l
D. no need if it is only COPD
(COPD) = Chronic Obstructive Pulmonary Disease

309. To do test of caries activity:
A. change of pH in saliva
B. detect incipient caries
C. Amount of microorganism and its growth ***

310. Decrease the effect of acid etching on the pulp The three length of files and
reamers that you work by them:
a. 20-26-29
b. 21-25-32
c. 20-25-32
d. 21-25-31 ***


311. Before an accurate face-bow transfer record can be made, it is necessary to
determine:
a. The axial center of opening-closing rotation peoples ***
b. The inclination of each condoyle
c. The physiologic rest position
d. Centric relation
e. All of the above

312. Chromium cobalt the constituent responsible for corrosion:
A. cobalt ****
B. chromium
Responsible for corrosion is cobalt
Resist corrosion is chromium

313. Odontogenic infection can cause least complication:
A. pulmonary abscess
B. peritonitis ***
C. cavernous sinus thrombosis
D. prosthetic valve infection
Dental Secrets Page 260

314. Splinting of several teeth together as abutments for a FPD is done to
a. Distribute occlusal load ***
b- Facilitate plaque control
c- Improve retention of the prosthesis
d- Preserve remaining alveolar support
E- Assure optimum design of embrasure

315. Pt. was bad oral hygiene and u want to make posterior bridge what the bes
type of pontic for cleaning and keep good oral hygiene:
a. Hygenic pontic ***

316. When tooth paste is used the child is advised:
A. not swallow ***
B. swallow a small amount
C. do not rinse
D. none

317. Small caries confined to enamel:
A. preventive measure ***
B. restoration
C. observation

318. Tongue develop from:
1. Mandibular branch and toburculum Kampar ***
2. First branchial arch

319. What composite should ideally be used for a class V:
OR
Best restoration for class V (5) & has glassy finish is:
1. microfill because it is polished better ***
2. microfill because it is stronger
3. Hybrid because it is polished better
4. Hybrid because it is stronger

320. Periodontium of diabetic patients affected by:
A. macrophages
B. neutrophils ***

321. Posterior palatal bar or strip used with:
a. Kennedy class1
b. Kennedy class2
c. Kennedy class3 ***
d. Kennedy class4


322. 2 canals in the same root most common in:
A. lower premolar
B. lower 6 in distal ***

323. Pedo, has trauma in 11, half an hour ago, with slight apical exposure, open
apex, treatment is:
A. pulpotomy with formocresol
B. apexification
c. DPC *** (Direct pulp capping)


324. Apical periodontal cyst arise from:
a. Hertwig sheath
b. Epithelial cell rest of malassez. ***

325. Formation of periodontal cyst due to:
a. Nasolacrimal cyst.
B. Hertiwigs. ***
c. Epithelial rest of malassaz
d. Peals of serres.

326. Which is the most Likely cause of periodontal cyst:
a. Cell Rest of Malassez
b. Cell rest of serss.
c. Cell of Hertwig sheath. ***

327. Endomethasone is a root canal sealer that:
a. Dissolve in fluid so it weaken the root canal filling.
b. Very toxic contain formaldehyde.
c. Contain corticosteroids.
D. All the above. ***

328. Endomethasone as root canal sealer:
A. have high solubility which dissolve and cause root canal failure
B. is highly toxic
C. contains cortisone
D. none of the above ***
Endomethasone
Root canal sealer of the Zinc Oxide type with Paraformaldehyde and corticosteroids
FORMULA Hydrocortisone acetate 1.0 g Excipients: thymol iodide, barium sulphate, zinc
oxide, Magnesium stearate q.s.ad. 100.0 g

329. The action of the sealant:
A. open enamel tubule to enter floride
B. make low surface tension to make retention ***
C. clean the enamel surface
D. make chick white appearance

330. Pt. with plasma enzyme should take:
a. Procaine with hcl which 1,200.00 epinephrine
b. Procaine hcl without epinephrine
c. Prolicaine hcl with 1.200.00 epinephrine ***

331. Path of insertion of RPD:
A. tilt right & tilt left
B. tilt anterior
C. tilt posterior
D. perpendicular to occlusal plane ***

332. What are the recommended numbers of implants for complete edentouls
patients:
A. maxilla 1 mandibular 1
B. maxilla 3 mandibular 2 *** for complete denture
C. maxilla 6 mandibular 4
D. maxilla 8 mandibular 6 *** for fixed prosthesis recommended (d)

333. What's the endo instrument that have bevel tip:
A. peeso Remer
b. Gates Glidden
c. Protaper ***
d. K file
The ProUltra ENDO 1-5 ultrasonic instruments.
http://www.endoruddle.com/FAQ?name=FAQRetxInstr

334. A 6o yrs. old female extracted several teeth and wear immediate denture for
six month without making a new one. There is folding of the mucosa in layers this is
called
A. flabby ridge ***
B. papillary hyperplasia (happen due to over growth)
C. epulis fissuratum (always associated with ertheyma and ulceration)

335. Pt. with multilocular radiolucency in mandible angle and multiple bone
fractures
a. Myloma
b. Osteomyelitis
C. hyperparathyroidism ***

336. Hyperplasia of nerve fiber:
A. calcitonin...
B. prostaglandin + serotonin ***
C. lysosomes enzymes
D. substance p....
337. object of demineralization of allogeneic DFBG in cold hcl:
a. Collagen fibers expose ***
b. HIV other pathogens killed
c. Deactivate proteins

338. Soaking frizzed direct bone graft in cold HCL for:
1. expose collagen fibers ***
2. Kill HIV virus
339. Basal cranial bone formed from:
A. Frontal & Occipital
B. Occipital & Sphenoid ***
C Frontal & Sphenoid

340. Pt. has impacted canine how can measured the distance to down it to suitable
place:
1. ..x ray only
2. .. x ray with clinical ***

341. The rows show truth the column show test result
a. Cell A has true positive sample. ***
B. Cell A has true negative sample.
C. Cell A has false positive sample.
D. Cell A has false negative sample.

A true positive
B true negative
C false negative
D true negative
342. Removing of dentine in dangerous zone to cementum is:
1. Perforation. (Apical perforation).
2. Ledge.
3. Stripping ****
4. Zipping.

343. 12 Child the dentist said to his parent to extract the 46 and 36 teeth because
this teeth can't treatment then what to do after extraction:
OR
12 years old female, has badly decayed 36, 46 non-restorableafter extraction you do:
A. partial denture
b. FPD
C. leave without any treatment
D. interim partial denture ***
http://www.cda-adc.ca/jcda/vol-73/issue-5/425.pdf

344. After root fracture. Best tissue healing:
A. interproximal connective tissue
B. interproximal bone
C. interproximal bone & CT ***
D. inflamed tissue

345. Patient has discrepancy of tooth color and different size, to do plan to treat
we need:
A. diagnostic cast ***
B. panoramic x-ray
C. cephalometric

346. Vazikonin Akinosi technique:
a. 27 gauge with long needle
b. 30 gauge with long needle
c. 25 gauge with long needle ***
http://books.google.com.sa/books?id=uM3DAeTH4mgC&pg=PA685&lpg=PA685&dq=ne
edle+used+in+akinosi&source=bl&ots=MW-
y441hZ7&sig=j5Lzxsox7a2PfuOwT8ugAIFXCN0&hl=en&sa=X&ei=2RUaU4LJIMjPhAe_84G
gDA&ved=0CEwQ6AEwDQ#v=onepage&q=needle%20used%20in%20akinosi&f=false

347. The most common cause of failure of the IDN Inferior Dental Nerve block is:
A. Injecting too low
B. Injecting too high

348. Most important sealer criteria to be success:
a. high viscosity
B. high retention
C. high strength
D. can add colorant
e. High resilience ***

349. The narrowest canal found in a three root maxillary first molar is the
a. Mesio-buccal canal. ***
b. Disto-buccal canal.
c. Palatal canal.
d. Disto-palatal canal.
e. Mesio-palatal canal.

350. Most canal predictable to perforation during post preparation is:
a. Lower (mb) canal ***

351. The placement of a retentive pin in the proximal regions of posterior teeth
would MOST likely result in periodontal ligament perforation in the:
A. mesial of a mandibular first premolar.
B. distal of a mandibular first premolar.
C. distal of a mandibular first molar.
D. mesial of a mandibular first molar ***
352. Second canal most frequently seen in
a. Mesiobuccal of upper molar ***

353. Perforation during endo space preparation what is the most surface of distal
root of lower molar will have tendency of perforation:
a. Mesial surface. ***
b. Distal surface.
c. Buccal surface.
d. Lingual surface.

354. Which of following disease associated decrease in caries:
a. Sjogren's syndrome (increase caries)
b. Cystic fibrosis (decrease caries)
c. Cerebral palsy
d. Down syndrome *** ( decrease caries more)

355. During perio surgery for a Pt. With chronic periodontitis

356. Streptococcus activity detected by:
a. Fermentation. ***
B. Catalase.

357. Thickness of luting cement:
A. 100 micrometer.
B. 40 micrometer. ***
C. 1mm.

358. Patient with Class II Kennedy PD, good oral hygiene and low caries index you
would use:
a. Circumferential clasp
b. Back action clasp
c. Cast clasp
d. Ring clasp ***

359. Patient who has un-modified class II Kennedy classification, with good
periodontal condition and no carious lesion the best clasp to use on the other side teeth
side:
A. reciprocal clasp (aker's clasp). ***
B. ring clasp
C. embrasure clasp
D. gingival approaching clasp

360. Relining which one is contraindicated:
a. resorbed ridge
b. increase vertical dimension ***

361. Cement appear in radiographic like caries cannot distinguish from it:
A. calcium hydroxide include hydroxyl group ***dental decks
B. zinc phosphate
C. glass ionomer
E. zinc polycarboxlate
F. none of above

362. Pt with wide cleft lip and palate lip adhesion or nasoalveolar molding planned:
a. Few week after birth
b. First third month ***
c. Third to sixth
d. 6 9

363. The protocol for dental trauma involves all of the following, except:
a. Ideally, the tooth should be repositioned to its original position.
b. EPT and thermal testing are unreliable following trauma since physical trauma can
severe or damage nerve supply without altering pulpal blood supply/vitality.
c. If the root is completely formed on a tooth that has been intruded, a pulpectomy
should be performed within 1-3weeks after the injury. ***
d. The tooth should be splinted for 2 to 4 months if it sustains a root fracture

364. Which of the following not appear in cleft lip and palate
A. enamel hypoplasia
B. missing teeth
C. supernumerary teeth
D. developmental cyst ***

365. Cone 20 its tip is:
a. 0.20 ***
b. 0.02
c. 0.2
d. 2
Cohen's Pathways of the Pulp 10Ed 2011

366. Patient had anaphylactic shock due to penicillin injection, what's the most
important in the emergency treatment to do:
A. 200 mg hydrocortisone intravenous
B. 0.5 mg epinephrine of 1/10000 intra venous
C. adrenaline of 1/1000 intra muscular (IM) ***

367. 3 year old pt., water fluoridation 0.2ppm what is the preventive treatment
A. 0.25 mg fluoride tablet***
B. 1 mg fluoride tablet
c. Fluoridated mouthwash
d. Sealant
368. For children considered to be at high risk of caries and who live in areas with
water supplies containing less than 0.3 ppm:
A. 0.25 mg F per day age 6 months to 3 yrs.
B. 0.5 mg f per day from 3-6 yrs.
C. 1 mg per day more than 6 yrs.
d. All of above***

369. The best way of radiograph shows displacement of mandibular condyle:
OR
pt. trauma leads to disk movement best radiograph to see disk movement:
OR
Displacement of TMJ due to trauma what is the best radiograph to show displacement:
a. Reverse towne ***
b. Oblique horizontal 30
c. AP

370. The blood intrapulpal pressure by Cm Hg is:
a. 10 ***
b. 15
c. 18

371. Which of the following would be ONE possible indication for indirect pulp
capping:
A. Where any further excavation of dentine would result in pulp exposure. ***
B. Removal of caries has exposed the pulp
C. When carious lesion has just penetrated DEJ

372. What is the correct sequence of events:
a. Differentiation of odontoblast, elongation of enamel epithelium, dentine formation
then enamel formation. *****
b. Differentiation of odontoblast, dentine formation then enamel formation, elongation
of enamel epithelium.
c. Elongation of enamel epithelium, differentiation of odontoblast, dentine formation
then enamel formation. ***
http://dentallecnotes.blogspot.com/2011/07/06.html

373. Amount of vertical bone loss around dental implant in first year:
a. 0.5
b. 1-1.5 ***
c. 4
http://www.quintpub.com/userhome/prd/prd_28_4_Cappiello_4.pdf

374. Type of osteointegration b/w dental implant & the bone is:
A. tissue
B. bone ***
C. titanium
http://en.wikipedia.org/wiki/Osseointegration

375. Type of microorganism present in Chronic sinusitis:
A. mixed anaerobic & aerobic ***
B. primary anaerobic
C. primary aerobic
d. 80% aerobic and 20% anaerobic
http://en.wikipedia.org/wiki/Sinusitis
also isolated are Staphylococcus aureus

376. Microorganism in responsible of caries & cavity excavation is:
A. klebsila
b. Strept. + Lactobacillus ***
c. Strept + actinomyces
http://en.wikipedia.org/wiki/Dental_caries

377. For Rebasing a denture:
a. Self-Cure
b. Heat Cure ***

378. Contraindication of gingivoectomy all the following Except:
a. Gingival enlargement ***

379. Case + x-ray pic. & the main feature is
1. Diabetes pt.
2. Mouth eaten appearance
What is your diagnosis?
a. Acute osteomyelitis ***

380. Case + x-ray pic. & the main feature is:
1. Wear of teeth
2. Blue sclera
3. More bone fracture
What is your diagnosis?
a. Osteogensis imperficta ***

381. One characteristic feature of PD, minor connector is :
a. connect with major connector by 90 degree ***
B. connect with major connector by acute angle
C. contact tooth surface on either side of embrasure ahold be ABRUPTLY
Minor Connector contacts tooth surface on either side of embrasure ahold be ABRUPTLY.
Angle at junction between Major connector and denture base should be <=90 degree

382. Mild yellow brown tetracycline staining:
OR
Child has Tetracycline discolored in permanent tooth what is the proper treatment:
1. Composite veneer ***
2. Home bleaching
3. Pumice micro abrasion
4. Porcelain veneer

383. Pt. presented to you with pulp polyp It is:
A. hypertrophic
B. hyperplasia ***

384. What of this surfaces ONE not contributed to Denture surfaces:
A. impression surface
B. polished surface
C. vestibule surface ***
d. Occlusal surface

385. Pt with internal resorption, what dentist should do:
A. complete extirpation of the pulp ***
b. Enlargement of canal to area of resorption

386. How you will do Differentiate between vital & necrotic pulp:
A. percussion
B. palpation
C. electric ***

387. Patient feel severe pain upper mouth pain is radiated to eye and ear, after you
check no caries when you pressure on maxillary premolar he feels pain. In X-ray no
change what diagnosis:
a. Acute apical periodontitis
B. maxillary sinusitis. ***
C. canine space infection
D. dento alveolar infection

388. Pt. come with pain in Max.Premolars area when examination no caries or
pocket but patient still feel pain with percussion:
a. Acute periodontitis
b. Gingivitis
c. Maxillary sinusitis. ***
http://www.dentistrytoday.com/oral-medicine/1529

389. Case with dry mouth, eye ocular dryness. Diagnosis is:
a. Sjgren's syndrome

390. Most cement irritation to the pulp is :-
a. Zinc phosphate

391. When we can't do orthodontic:
a. 15
b. 25
c. 50
d. Unlimited ***

392. Patient have caries reach the cementum which material best to restore this
caries:
A. glassionomer ***
b. Veneer
C. amalgam
D. composite

393. For re-endo treatment of failure canal , how you will remove the gutta (firstly):
A. barbed broche
B. solvent material ***
3. k- file
4. New hedstrom

394. What the influence of xylitol
a. It cause caries
b. Safe to the teeth***
c. Increase saliva
d. Decrease saliva

395. Xylitol gum used for gum:
a. Prevent caries ***
B. decrease salivation

396. Xylitol:
a. Reduce caries ***
http://www.ncbi.nlm.nih.gov/pubmed/12693818

397. Tug back is referred to:
A. resistance of material to withdrawal ***
A. Retention of GP inside the canal ***
A. the retention of the Gutta Percha at the apical portion of the canal ***


398. Large amount of topical fluoride does not cause fluorosis, Why:
A. the teeth already calcified ***

399. Type of brushing bristles works as active part:
a. Stillman's technique
B. bass technique ***
c. Roll technique
400. What is the best brushing tech.:
A. modified bass ***
B. stillman's
C. modified stillman
D. modified charter

401. Bacteria cause osteomyelitis:
a. S.aureus ***
402. Organisms that associated with osteomyelitis :-
a. Streptococcus
b. Staphylococcus ***
403. Rapid maxillary expansion activation:
A. one week
B. 2 week
C. everyday ***
D. every 2 day

404. At the beginning of the Operation day in the clinic, you should start the
water/air spray for Three minutes in order to get rid of which type of microorganisms:
OR
At the start for our clinic day, why do we spray for 3 minutes:
A. streptococcus mutans.
B. streptococcus salivaris. ***
c. spirochete

405. Instrument that has +ve positive Rank angle:
A. protaper ***
B. profile ( -ve)
C. k files
d. K reamer

406. 3yrs old child receive trauma to max incisor which lead to grayish
discoloration and in radiograph there is periapical radiolucency what is the ttt of choice:
A. pulpotomy
B. apexification and filling with ZOE***
407. Prosthetic pt. have upper complete denture oppose lower 6anterior teeth
what's show on the ridge:
A. flabby ridge in the lower
B. bone resorption in the upper anterior ***

408. We want to construct upper denture with palatal strap, which act as indirect
retainer what's the type of Kennedy class:
A. class 1 ***
B. class 2
C. class 3
D. class 4

409. Child pt. has swelling which grow rapidly associated with pain and numbness
on radiograph we see radiolucency with pathosis of radiopacity give sunray appearance
what's your diagnosis:
A. osteofibroma
B. osteosarcoma***

410. Tertiary dentine:
A. calcified dentine
B. reactive***

411. Aesthetic temporary restoration for rampant caries in adult:
a. Zn Phosphate
b. ZOE
C. composite
D. silicate***

412. 2 days after conservative ttt (deep composite restoration) pt. came with pain
the diagnosis:
A. reversible pulpitis***
B. irreversible
C. liqufication necrosis

413. When we want to give child fluoride ask about:
a. Age and amount of water fluoridation ***

414. Child pt. complaining of burning sensation, malaise increase in temperature
and sore throat:
A. acute herpetic gingivostomatitis ***
B. aphthous ulcer
C. herpengina

415. badly decayed lower 6, what's important thing about restoration:
A. adequate core material
B. ferrule effect***

416. Disinfection of GP inside dental clinic:
a. Naocl 5.2%***
b. H2o2 70%

417. pt. with gingival inflame, the gingival appear edematous and erythematous
what's the best brushing tech:
A. stillmans
B. modified stillman
C. modified charter
D. modified bass ***
http://www.juniordentist.com/types-of-tooth-brushing-techniques.html


418. The first sign of gingival inflammation:
a. Increase the vascularity ***

419. pt. with phenytoin u did gingivectomy u will wait for how much time to heal:
A. 3 days
b. 5-7 days ***
C. 3 months
Complete surface epithalization after gingivoectomy take from 7-14 days
http://www.slideshare.net/PARTHPMT/gingivectomy

420. You do gingivectomy to pt. take phenytoin when surface epithalization occur:
A. 3 days
b. 2-4 weeks ***
c. 22-24 days
D. 28 days
421. pt. take phenytoin after gingivectomy surface epithialization occur:
Patient take (phenytoin) and make gingivectomy, How many days for epithalization:
a. Within 5-14 days ***

422. Low pain of anesthetic injection in all of following Except:
1. Stretch of the muscles
2. Needle gauge larger than 25 ***
3. Slow injection
4. Topical

423. Material used for one visit apexification:
a. MTA ***
b. Caoh+ CMCP
C. formocresol

424. Bonding agent EBA consist of:
OR
What does Enamel bonding agent (EBA) consist of:
a. Unfilled resin ***
B. primer and bonding agent
c. A mixture of resins in an acetone or ethanol solvent
d. A wetting agent and resins
e. A mixture of priming and bonding agents

425. Lingual nerve:
A. part of trigeminal nerve ***
B. supply the submandibular gland
c. A and B
The gland receives its blood supply from the facial and lingual arteries
http://en.wikipedia.org/wiki/Submandibular_gland#Innervation
http://en.wikipedia.org/wiki/Lingual_nerve

426. Lingual nerve branch of:
a. Trigeminal N
b. Mandible N
c. Facial N***
http://en.wikipedia.org/wiki/Lingual_nerve


427. Which bone form nasal septum:
a. Vomer bone ***
428. Instruments commonly used for closed treatment of nasal fractures are:
a. Asch forceps
b. Walsham forceps ***
c. Boies fracture elevator

429. Pt. come with broken nose, what forceps should use in this situation:
a. Walsham forceps ***
b. Asch forceps
c. Boies fracture elevator
Walsham's forceps used to reduction of fracture nasal forceps
To reduce nasal complex fractures.
Walsham's forceps & Asche's forceps are used for reduction of fractured segments.
These fractures should be repaired within 7-10 days.
Dental Pulse vol.1

Simple nasal bone fractures can be repositioned with
Walsham's forceps. In addition, the nasal septum needs
To be straightened with Asche's forceps and the lower
Portion of the septum relocated in its groove on the nasal
Floor.
Textbook of general & oral surgery

430. Pt on dialysis when do the extraction:
A. One day after dialysis ***

431. Common complication of RCF:
A. improper obturation ***
B. over extended GP
C. fracture instrument ***
http://www.mouthandteeth.com/treatments/root-canal-complications.htm
The most common root canal complication is caused by the accidental breaking

432. Pt. has denture one yr. ago, 15 days he came complaining from pain around
the tooth on examination no caries :
A. flexible connecter***
b. Unfit surface

433. Maryland bridge:
a. Resin bonded bridge ***
b. Cantilever bridge
c. Conventional bridge

434. Best community based caries prevention method:
a. Water fluoridation. ***
b. Sealants

435. Clicking at posterior sides in denture:
A. inappropriate occlusal plan
B. high vertical dimension***
http://www.cram.com/flashcards/rp-final-common-denture-complaints-and-their-
causes-1304165

436. Redness on residual ridge at denture wearing patient:
OR
Pt. with complete denture 2 yrs. ago, come with small red lesion, he has bad oral
hygiene and wear denture all time at night also diagnosis is:
A. denture stomatitis ***
B. epulis fissuratum

437. Impression came to technician with blood, what is the suitable disinfectant:
a. Glutaraldehyde. ***
Five laboratories reported that some alginates resulted in casts with poor surface
properties when immersed in hypochlorite (0.1 and 1%), sodium dichloroisocyanurate,
and 2% glutaraldehyde solutions. This paper highlights that there is no universally
recognized impression disinfection/sterilization protocol. It is recommended that all
impressions should at least undergo a disinfecting procedure by immersion in 1%
sodium hypochlorite for a minimum of 10 minutes
http://www.nature.com/bdj/journal/v180/n10/full/4809092a.html

438. Muscle Retract (retrudes) the mandible:
a. Temporalis M ***
Muscles elevation of the mandible: Masseter, temporalis and medial pterygoid
Muscles depression of the mandible: geniohyoid, mylohyoid, digastric, lateral pterygoid
and infrahyoid
Muscles protrusion of the mandible: Lateral pterygoid, medial pterygoid assists,
masseter
Muscle retraction (retruded) of the mandible: Temporalis
Muscle do lateral movement of the mandible: Lateral pterygoid, medial pterygoid

439. Newborn with mass on the upper anterior region, what is ur diagnosis:
a. Congenital epulis of the newborn.
b. Lamina dura cyst
c. Bohns nodules*** (Dental lamina cyst of new born)
Epsteins pearls may be found on the mid-palatal raphe of the hard palate.
Which occur along the mid palatine raphe at junction of soft and hard palate
Bohns nodules remnants of salivary glands are located on the buccal or lingual mucosa,
or on the hard palate, away from the raphe.
(Dental lamina cyst of newborn) small nodules or cyst in the gingival due to proliferation
of rest of serres. Buccal and lingual aspect of dental ridge is remnant of mucous gland
located on the crests of the alveolar ridges.
Congenital Epulis of the newborn similar in appearance to a dental lamina cyst
but usually located in the maxillary anterior region. Although some recede
spontaneously, a usually large congenital epulis may cause feeding problems and
require excision. Recurrence is unlikely.
http://www.nypartnersinoralhealth.com/tableofcontents/oralpathology.html

Bohns nodules
////////////////////////////////////////////////

440. The most tooth prevent crowding in lower arch is:
a. Primary lower first molar ***
b. Primary upper first molar
c. Permanent first lower molar
d. Primary upper second molar

441. Dose of carbamazepine:
A. 1200 mg *** (used as ttt for trigeminal Neuralgia)
Initial: On the 1
st
day, 100 mg b.i.d. for a total daily dose of 200 mg. may increase to 200 mg/day
Maintenance: Control of pain can be in most patients with 400 to 800 mg daily.
However, 1200 mg daily. At least once every 3 months
http://www.globalrph.com/anticonvulsants.htm#carbamazepine

442. Patient has lefort I, bleeding mainly from:
A. maxillary artery ***
B. infraorbital artery
If other options have posterior superior alveolar artery of the maxillary should be the
right answer
http://www.ncbi.nlm.nih.gov/pubmed/19816309

443. Patient has periodontal disease which in lower #6 has furcation grade III,
mobility grade II, moderate bone loss, short root , whats the prognosis:
A. fair
B. moderate
C. poor***
444. Patient have moderate bone loss and grade II mobility and grade II furcation
ttt will be:
a. Fair ***
B. Weak
c. Excellent
d. Moderate
Questionable prognosis: One or more of the following:
Advanced bone loss, Grade II and III furcation involvements Tooth mobility,
Inaccessible areas, Presence of systemic/environmental factors

445. Zinc phosphate and zinc polycarboxilate cement have:
a. Zinc oxide powder ***

446. The type of caliper used for vertical dimension measurement of complete
denture is:
A. bolye's caliper
b. Willis caliper ***

447. Dental plaque composed primarily from:
A. food debris
B. bacteria ***

448. Role of saliva in dental plaque:
a. buffering + decrease cone of bacterial acids ***

449. The radioactive therapy to treat the hot tumor:
A. yttrium-89
B. yttrium-90 ***
C. yttrium-91

450. 30 months child had trauma of upper (A) and x-ray show intrusion of it in the
dental sac of the future successor what is ur ttt plan:
A. Surgical extraction of (A) *** (A= any tooth).
B. wait spontaneous eruption
C. pulpotomy
D. orthodontic ttt

451. Pt. on warfarin about to do endo surgery for him which of the following is
more important:
A. PT ***
B. PTT
Heparin: PTT, affects the intrinsic pathway and fibrinogen levels; safe in
pregnancy.
Warfarin: PT, affects the extrinsic pathway, and vitamin K; teratogenic.
452. Patient need extraction and he is on warfarin:
a. PTT 2-2.5 INR
b. PTT 1-1.5 INR
c. PT 1-1.5 INR
d. PT 2-2.5 INR ***
Pt. on Warfarin always look for PT / INR
Pt. on Heparin always look to PTT

453. Pt. with TB infection with positive sputum and need dental ttt what's ur
choice:
A. make ttt wearing mask ***
B. postpone ttt

454. The ideal time for beginning of tooth brushing of pediatrics is:
A. after complete eruption of deciduous teeth
B. after 2 years ***
C. after 4 years
D. just before primary school
No answer is right because the best time when the first tooth erupt from 6 months

455. Child had trauma in the upper central and become intruded with loss of
superficial layer of epithelium this is
1. Concussion and subluxation
2. Laceration and subluxation
3. Laceration and luxation ***

456. Peg shaped lateral in which stage:
A. morphodifferentiation***
B. initiation
C. histodifferentiation

457. Neonatal teeth:
A. before birth
b. 0-30 days ***
C. 6 month
Natal teeth: before birth

458. The most accurate diagnosis test:
a. Thermal test
b. EpT
c. Cavity test *** but most invasive.

459. In case of resorption of alveolar ridge, the percussion when do crystal (crestal)
incision posteriorly not to damage:
A. lingual nerve***
B. buccal nerve
C. mylohyoid nerve
D. alveolar nerve

460. Dental implants are successfully with minimum failure:
A. premaxilla area in the upper arch.
b. Posterior area of the maxillary arch. (Max. failure)
c. Mandible between the mental foramens. ***
d. Buccal shelf of the mandible.

461. Patient suffering from a submandibular gland abscess, dentist made a stab
incision and is fixing a rubber drain to evacuate the pus, the drain is sutured to:
A. Intra-oral between the myeloid muscles.
B. Extra orally from the most fluctuant point. ***
C. Extra orally under the chin.
D. From angle of the mandible

462. For lower premolars, the purpose of inclining the handpiece lingually is to:
A. Avoid buccal pulp horn***
B. Avoid lingual pulp horn
C. Remove unsupported enamel
D. Conserve lingual dentine

463. For amalgam Restoration of weakened cusp you should:
A. Reduce cusp by 2mm on a flat base for more resistance
B. Reduce cusp by 2mm following the outline of the cusp***
C. Reduce 2mm for retention form

464. Before filling a class V abrasion cavity with GIC you should:
A. Clean with pumice, rubber cup, water and weak acid ***
B. Dry the cavity thoroughly before doing anything
C. Acid itch cavity then dry thoroughly

465. The fundamental rule in the endodontic emergencies is :
A. control pain by inflammatory non-steroid. ***
B. diagnosis is certain.

466. Smoking is dangerous during which period of pregnancy:
A. 3-8 week ***
B. 3
rd
trimester

467. Best measurement of periodontitis by:
a. Pocket depth.
b. Bleeding.
c. Attachment level. ***

468. Tooth responsible for providing space for lower jaw and prevent crowding:
a. Upper 1st primary molar
b. Upper 2nd primary molar
c. Lower 1st primary molar
d. Lower 2nd primary molar ***
Also primary canine in lower especially


469. Ugly duckling corrector:
OR
In ugly duckling stage, which tooth corrected this stage:
A. canine ***
b. lateral incisor
c. premolars

470. Pt. has complete denture upper and lower Ant. is porcelain teeth, Ant
posterior is acrylic the ridge:
A. flabby ridge. ***
B. knife-edge ridge

471. Teenager boy with occlusal wear the best ttt. is:
A. remove the occlusal.
B. teeth capping. ***
C. restoration.

472. Nerve supply to tongue & may anaesthetized during nerve block:
a. V
b.Vll (Facial N.) ***
C. lX
d. Xll
Dental decks 1904
An injection into parotid capsule during nerve block injection MAY CAUSE Bell`s palsy
facial expretion and The sensory portion of facial N supplies the taste to the anterior
two-thirds of the tongue.
If needle is positioned too posteriorly, anesthetic may be put into parotid gland
(dangerous systemic effects), or paralyse Cranial Nerve VII (7), resulting in Bell's Palsy-
like symptoms. Also, if the needle is placed too medially the medial pterygoid muscle
can be injected, resulting in trismus.
The sphenomandibular ligament is most often damaged in an inferior alveolar nerve
block
http://en.wikipedia.org/.../Inferior_alveolar_nerve...

473. ANB in Class II:
A. increase***
B. decrease
C. same ANB is angle between SNA and SNB
ANB is angle between SNA and SNB SNA is angle between horizontal line from sella to
nasion and vertical line from nasion to PointA (Subspinale) inner most point one of
conitour of premaxilla between anterior nasal spine and incisor tooth. its normal value
is 82 SNB is angle between horizontal line from sella to nasion and vertical line from
nasion to point B (Supramentale): innermost point on contour of mandible between
incisor tooth and bony chin. . its normal value is 80normal value of ANB is 2
degreenormal value of ANB is 2 degree

474. What medical condition should prevent the dentist from practicing dentistry:
A. Diabetes.
B. Hypertension.
C. Influenza. ***
D. Headache
475. :

476. Pt. after delivery of upper complete denture and lower metallic partial
denture with only remaining anterior teeth he was excellent, after 1 year he came to u
with pain around the abutment after u checked everything is normal ,what is the
problem:
A. Rough fitting surface
B. Sharp metallic framework
C. Loose denture***

477. The hardest tooth to be anesthetized:
A. Upper premolar
B. Lower premolar
C. Upper molar
D. Lower molar***

478. Blood supply of palatal mucosa in anterior region:
A. Greater palatine artery
B. Incisive foreman ***
C. Infra orbital

479. Part of removable or fixed prosthesis that connect the abutment to other part:
1-major connector (unit of partial denture connect parts of prosthesis located in one
side of arch with those on opposite side)
2- Minor connector (connecting unit between major connector or base and other part of
prosthesis)

480. Pt. has supernumerary teeth and exophthalmos and conjunctival.what is the
diagnosis:
A. crouzons syndrome.

481. Pt. with chronic renal failure what is developed:
A. Hyperthyroidism
B. Hyperparathyroidism***

482. The factor that may predict the outcome of disease and prognosis:
A. Risking factors ***
B. Prognostic factor

483. The subgingival scaler to be safed it should be:
A. universal
B. The head should be 90% with shank
C. gracey curette***

484. Which statement is right about caries:
a. reversible
b. Irreversible ***

485. Standard treatment for completely edentulous pt. nowadays is:
a. Conventional CD
b. CD with attachment
c. CD supported by 2 implant
d. CD supported by 4 implant ***

486. Fluoride in water fluoridation should be:
a. 1ppm ***
Ideal water fluoridation is: 0.7-1.2 ppm
www.ada.org
Fluoride is effective in preventing & reversing the early signs of dental caries (decay).

487. Premature loss of primary teeth to effect the occlusion should be:
A. 1-2 yrs.
B. 3-4 yrs.
C. 5-6 yrs. ***
D. 7-8 yrs.
The space of prematurely lost teeth usually change in the following:
6 months period, sometime a decrease in space may occur within days or weeks.

488. Centric relation is:
A. Teeth to teeth
B. Bone to bone***
C. bone to teeth
Centric relation is the most retruded relationship of the mandible to maxilla when the
condyle in the most posterior unstrained position in glenoid fossa from which lateral
movement can be made at any given degree of jaw separation.
Centric occlusion the occlusion of teeth when the mandible is in centric relation to
maxilla (maximum teeth intercuspation)
489. Cross-section of 10 K-file is:
a. Square

490. After trauma to 11 there is small pink spot appear, it is:
a. Internal resorption***
b. external resorption
491. After trauma, a tooth becomes yellowish in color, this is due to:
a. Necrotic pulp.
b. Irreversible pulpitis.
c. Pulp is partially or completely obliterated. ***
d. Hemorrhage in the pulp.
Tooth discoloration after trauma is a common problem faced by adults and children but children are
reported to be the worst sufferers.

492. Discoloration of endo treated teeth:
a. Hemorrhage after trauma.
b. Incomplete remove GP from the pulp chamber.
c. Incomplete removal of pulp tissue. ***
If pulp remnants remain inside the pulp chamber after endodontic treatment, this can
cause a coronal discoloration due to a gradual disintegration and flow into the dentinal
tubules. When filling materials and sealer remnants or medicaments containing
tetracycline that are not completely removed from the pulp chamber after endodontic
treatment are in contact with dentin over time, they can penetrate into the dentin
tubules and discolor the tooth
http://munin.uit.no/bitst.../handle/10037/4210/thesis.pdf

493. 2 yrs. child had trauma to upper central it becomes intruded what to do:
a. Extraction
b. Observe ***
494. Some test of the autoclave for testing the Efficacy of Autoclaves:
Quality control is essential to ensure that potentially infectious agents are destroyed by
adequate sterilization regimes.
The ways for testing Autoclave efficacy:
1. Browne's tubes are glass tubes that contain heat sensitive dyes. These change color
after sufficient time at the desired temperature.
2. Bowie Dick tape is applied to articles being autoclaved. If the process has been
satisfactory, dark brown stripes will appear across the tape. Pale brown stripes are
suggestive of poor heat penetration, and an unsatisfactory sterilization process.
These methods give an immediate indication of the success or otherwise of an autoclave
run but they are only suggestive of a successful sterilization.
3. Spore strips may be placed inside the autoclave at the start of its cycle. After running,
the autoclave the strip is recovered and cultured. Absence of growth after a suitable
period indicates a successful run.
The problem with this method is that it is retrospective. If a problem has arisen, then
this will be discovered only when the spores have germinated. This is probably too late
to take effective action other than to call in an engineer to prevent further problems

495. Class I malocclusion pt. with high over bite and diastema he came to your
clinic to correct the diastema what's to do:
1-correct the over bite first ***
2-correct the diastema
496. When we do CBC we checked:
OR
Dentist see in CBC:
1. RBCs, WBCs, platelet, HB***
2. RBCs, WBCs, platelet, HB, k
3. RBCs, WBCs, HCT, HCT, platelets, Hb
4. RBCs, WBCs, HCT, HCT, platelets, Hb, calcium

497. 34 yrs. old pt. has class 1 malocclusion with centric occlusion she has no
problem but while protrusive movement she had premature contact of anterior:
1. Grinding of upper anterior ***
2. Grinding of lower anterior
3. Grinding of upper molar
4. Grinding of lower molar
498. Pt. has upper complete denture and lower partial denture missing posterior
teeth:
A. combination syndrome ***


499. Which root we found 2 canals:
OR
Root with 2 canals:
A. distal root of upper molar
B. distal root of lower molar
C. mesial root of upper first molar ***
D. mesiobuccal of lower
Mesial root of upper 1
st
molar (most commonly)
Distal root of lower 1
st
molar

500. Best bone graft to be used in young pt. has mass in upper anterior region after
removal of the mass:
a. Dried freeze graft
b. Auto graft from autogenous bone grafting ***
501. Which of the following status u cannot do crown lengthening:
a. Heavy calculus
b. Enlargement of the gingiva ***
502. Focal therapy of systemic infection:
503. Blood supply for upper arch:
A. facial
b. Maxillary ***
C. Ant. nasal
504. Branch of internal carotid artery that supply facial:
A. Opthalmic artery ***
B. Facial
C. Vomer
D. Orbital
.
the internal carotid artery supplies the Ant. portion of the forehead and dorsal surface
of the nose via ophthalmic artery branches.
Black: from internal carotid artery (Via opthalmeic)
Red: from external carotid artery


505. Curve used to diagnosis marginal deterioration:
1. Mahler scale ***
506. Diagnostic aid in osseous defect:
a. pt. dental & medical history
b. Mobility & furcation involve
c. X-ray ***
507. Talon cusp most common in:
a. Max. Lateral incisor ***
B. mand. Incisor.
C. max. Incisor
508. Pt. with blue sclera, Fracture of bone before (x-ray):
A. dentinogenesis imperfecta***
B. osteogenesis imperfect ***

509. Pt. 8 yrs. old fracture upper central 9 months ago with -ve vitality test best
treatment:
A. endo
b. Pulpotomy with formocresol
c. Pulpotomy with caoh
d. Total debridement + apexification ***
We do Apexification because it's immature tooth
(Not complete root apex)

510. Old pt. need complete denture but tissue need linear removal take care of
injury to:
A. lingual nerve***
b. Hypoglossal nerve

511. During setting of lower molar in complete denture lower 6:
1. 2/3 distance to retro molar pad ***
2. 1/3 distance away of retro molar pad

512. Type of cementum coronal 2/3 of root:
1. acellular intrinsic fiber
2. acellular extrinsic fiber ****
c. Mixed
513. Need use for biopsy for aspiration:
OR
What gauge of needle of aspiration:
a. 19 gauze ***
b. 22
c. 26
Decks say its 18 gauge with 5ml syringe.








514. Hemisection bone resorption:
a. One side of alveolar bone***
b. Two side of alveolar bone
c. Three side of alveolar bone
515. We make PDL flap why:
A. to increase mucogingival width
B. to remove granulation tissue
C. to accessibility to the diseased root surface ***
516. Type of bone for best successful implant:
OR
Best Implant Success in bone:
a. type1
b. type2 ***
c. type3
D. type 4
517. Permanent dentition: (Presents)
A. long live ***
B. 22 years
C. 12 years
518. Tooth in dense common at:
1. Maxillary canine
2. Maxillary central
3. Mandibular central
4. Lower molar
519. Maxillary lateral incisor Trauma, blow in right side of face with ocular
hemorrhage and ecchymosis of buccal sulcus and can't open his mouth:
a. Lefort I
b. Lefort II
c. lefort III
d. Zygomatic fracture ***
520. 10 yrs. pt. with bluish central incisor:
A. pulpectomy
B. pulpotomy
C. direct pulp capping
D. hemorrhage in pulp ***
521. Fracture of angle of mandible which view is best to see if its favorable or
unfavorable:
A. occiptomental
B. reverse town
C. 30 degree lateral oblique ***
For mandibular fracture to detect if its favorable or not
If he asking vertically u will choose periapical view
If its horizontal u go for 30 degree oblique
If he asking of body and ramus it is 15 degree... This is from dentogist MCQS

522. Functional cusps in class 1 relation:
A. buccal upper with lingual lower
B. palatal upper w buccal lower
C. maxillary palatal cusp and mandibular buccal cusp***
https://www.google.com.sa/url?sa=t&rct=j&q=&esrc=s&source=web&cd=2&cad=rja&v
ed=0CDMQFjAB&url=http%3A%2F%2Fwww.ncbi.nlm.nih.gov%2Fpmc%2Farticles%2FPM
C3481985%2F&ei=N3zpUsuvJYXDhAedw4CgDg&usg=AFQjCNG3-
yhD9ew2bvbrtdIqfehwQdMa7Q
523. instrument used for dentine scratch and beveling angle:
A. angle former***
https://www.google.com.sa/url?sa=t&rct=j&q=&esrc=s&source=web&cd=5&cad=rja&v
ed=0CEgQFjAE&url=http%3A%2F%2Fquizlet.com%2F20897393%2Fdental-instruments-
functions-and-characteristics-flash-cards%2F&ei=-
3zpUo2DMMqBhAeHrIGgAQ&usg=AFQjCNH4ralpwmNyz-QwyGz2CwWapa8lmA

524. Fluoride application for mentally retarded pt.: Neutral sodium fluoride
varnish***
525. Pt feel pain when u put explore on tooth:
A. reversible pulpitis
B. irreversible pulpitis
C. dentine hypersensitivity ***
http://en.wikipedia.org/wiki/Dentin_hypersensitivity
526. Child with HIV what is the most oral manifestation:
A. psoriasis
b. Candidiasis ***
c. Herpes simplex
D. aphthous ulcers.
Tyldesly's Oral Medicine 5th Ed 2003
http://www.hivdent.org/_oralmanifestations_/oralmanifestations_omhah0502.htm

527. Mandibular nerve is branch of: trigeminal nerve***
528. In alginate impression what is responsible for fast up and slow down:
A. potassium alginate
b. Sodium phosphate***
c. Potassium sulfate
d. Sodium sulfate
Sodium phosphate 2%: is retarder, control setting time
(Fast or slow)
529. Maxillary nerve originate from:
a. Rotandum ***
b. Oval
C. spinosum
530. Diameter of arterioles is: 50 micrometer ***

531. IgG, IgA is:
a- active and acquired
b- Active and natural
c- Passive and acquired
D- Passive and natural ***
http://en.wikipedia.org/wiki/Antibody

532. Prophylactic: intraligamentary injection ***
533. Pt. Complain from cold we should do: cold test ***
534. Temperature damage bone: 46 C (1 min) ***
Churchill's Pocketbook Clinical Dentistry 3 Ed 2007, Page 331
Bone tissue damage occurs at temperature over 47 C
so a traumatic surgery imperative.
535. Tongue:
Blood supply
The tongue receives its blood supply primarily from lingual artery, a branch of the
external carotid artery. Veins of the tongue, the lingual veins, drain into internal jugular
vein.
The floor of mouth supply from the lingual artery.
2
nd
blood supply to the tongue from the tonsillar branch of the facial artery and the
ascending pharyngeal artery. The triangle formed by the intermediate tendon of the
digastric muscle, the posterior border of the mylohyoid muscle and the hypoglossal
nerve is sometimes called Pirogov's, Pirogoff's, or Pirogov-Belclard's triangle.
The lingual artery is a good place to stop severe hemorrhage from the tongue.
Innervation Nerves which supply the sensation and taste of the tongue differ between
the anterior and posterior parts.
Anteriorly, taste is passed along the chorda tympani, a branch of the facial nerve.
Sensation is passed along lingual nerve, a branch of the trigeminal nerve.
Posteriorly, both taste and sensation are passed along the glossopharyngeal nerve.
All intrinsic and extrinsic muscles of the tongue are supplied by the hypoglossal nerve,
with the exception of the palatoglossus. The palatoglossus is innervated by the vagus
nerve.

536. Best dental vitality test for old patient with sensitivity to electricity
537. An old patient has skeletal disorder by using X-ray found enlargement in
Whats the type of x ray should be more accurate:
a. Cephalometric ***
b. Reverse town
c. Panoramic x ray
538. SCC:
I. The great majority of oral and pharyngeal cancers are squamous cell carcinoma (SCC).
2. Oral cancer remains twice as prevalent in males as in females, and nearly twice as
many deaths occur in males
As in females. Oral cancer is closely related to increasing age, alcohol consumption and
smoking are the main risk Factors.
3. Cancers of the lip and oral cavity account for about two-thirds of all new oral and
pharyngeal cancers with
The tongue being the most common site of cancers of the oral cavity.
4. Overall the 5-year survival rate for oral and pharyngeal cancers is about 50%.
However, survival rates vary considerably depending on the cancer site, gender, and
race. Note:
The 5-year survival rates for cancer of the lip are about 90%, of the tongue. It is about
half that and is only about 20% among male African-Americans.
Women tend to have higher survival rates with the exception of cancer of the lip.
5. Erythroplasia, rather than leukoplakia, is often the first sign of cancerous change in a
lesion.

539. The antibiotic of choice in ttt of osteomyelitis:
A. penicillin
B. clindamycin***
C. erythromycin
Clindamycin is given orally after initial intravenous treatment for 1-2 weeks for
osteomyelitis and has excellent bioavailability. It is active against most gram-positive
bacteria, including staphylococci. Linezolid is active against methicillin-resistant
staphylococci and vancomycin-resistant Enterococcus. It inhibits bacterial protein
synthesis, has excellent bone penetration, and is administered intravenously or orally.

540. Child with puffy erythematous gingiva is most likely to have:
A. Herpangina
B. Atypical varicella.
C. Primary herpetic gingivostomatitis.***
D. None of the above.
E. All of the above.

541. Buccal shelf of bone is :
A. primary stress bearing area***
B. relief area
C. undercut area
542. The primary bearing area on the mandible is:
a. Alveolar crest
B. shelf area
C. retro molar pad
d. Buccal shelf of the mandible ***

543. Perforation in distal root of lower molar in which wall:
A. mesial ***
B. distal
C. buccal
D. lingual
544. To check posterior palatal seal:
1. T. burnisher (for vibrating line)
2. Pencil ***
545. Posterior palatal seal is at:
A. at line between soft and hard palate ***
B. mesial to line between soft and hard palate
C. distal to line between soft and hard palate
NBD Part II
Posterior Palatal seal (Post dam): is the post. Border of maxillary complete denture that
put pressure on the displaceable tissue near the junction of the hard and soft palate aid
in in retention of prosthesis.
546. Patient has complete denture and has pain and erythema (fissured) at the
angle of mouth:
A. angular cheilitis ***
B. denture sore mouth

547. Patient has large ulcer inside mouth and has Bull's eye and with skin lesion on
hands:
A. herpes virus
B. erythema multiform ***
c. Pemphigoid
http://kidshealth.org/parent/infections/skin/erythema_multiforme.html

548. Treatment of solitary bone cyst in young person:
A. incision and drainage
B. curettage*** (curettage & close)
C. marsupialization
http://www.ncbi.nlm.nih.gov/pubmed/20360508

Simple (Unicameral) Bone Cyst The unicameral bone cyst has specific treatments for a
that are determined based on size of the cyst, strength of the bone, medical history,
extent of the disease, activity level, symptoms an individual is experiencing, and
tolerance for specific medications, procedures, or therapies. The types of methods used
to treat this type of cyst are curettage and bone grafting, aspiration, steroid injections,
and bone marrow injections. Constant observations and activity modifications are the
most common nonsurgical treatments that will help prevent unicameral bone cyst from
occurring and reoccurring.
http://en.wikipedia.org/wiki/Bone_cyst
549. Material used in RCT:
A. mineral trioxide aggregate "MTA" ***
550. 5 years old patient with high caries incidence and lost lower D of both sides
and by x-ray the premolar are found and need space maintainer, which is the best:
A. lingual arch ***
B. band and loop
C. crown and loop
551. In pulpectomy of deciduous teeth the best root canal filling material is:
a. ZOE cement***
b. Zinc Poly Carboxylate + zinc phosphate
c. Glass ionomer cement
552. Material added to component of acrylic in complete denture to increase
strength:
A. strength
B. impact denture

553. Rest in partial denture is to:
A. increase strength
B. connect parts in one place

554. Fluoride used in-patient of special needs:
a. Na fluoride
B. stannous fluoride
C. acidulated phosphate
D. varnish fluoride***
Varnish has been regarded as a safe and easy alternative for caries control in patients
with special needs"
http://www.dentistrytoday.com/materials/1474
555. 275- Patient has in symphysis area and need maxillofacial surgery, What is
the wire:
1. 4 feet gauge
2. 4 feet 28 gauge
3. 6 feet 26 gauge ***
4. 6 fee. gauge
http://en.wikipedia.org/wiki/Mandibular_fracture#Treatment
556. Difference between dentinogenesis imperfecta and amelogenesis imperfecta:
1. hereditary factor
2. Brown color of enamel
3. Pulp chamber and root canals ***
( . )
In amelogenesis imperfecta: dentin & pulp not affect
In dentinogenesis imperfecta: no pulp cavity

557. Patient with blue sclera and colored tooth and bone defect:
A. dentenogenesis imperfecta
B. osteogenesis imperfect ***

558. Use of varnish for young boy in restoration of lower permanent molar to:
A. protect pulp from sensitivity***
b. Prevent discoloration from filling
559. Varnish used to:
A. decrease no. of dentinal tubules
b. Decrease permeability of dentinal tubules***
c. Break back toxins
Varnish sealing for dentinal tubules decrease dentin permeability of dentinal tubule.
560. 25 yrs. old pregnant female has lesion on lower anterior (or upper)
asymptomatic:
A. pyogenic granuloma ***




561. Enamel is not or less self-mineralized due to:
A. formative cells dead or disappear after eruption***
B. large content of organic particles
C. small content of inorganic particles
D. large amount of inorganic minerals
562. Impression material proper principle is:
A. long working time (in mixing) and short setting time (in a mouth) ***
B. short working time and long setting time
C. long mixing time and short working time

563. Antibiotic inhibit cell biosynthesis:
A. penicillin***
B. tetracycline
C. cyclosporine
D. sulfonamide
http://www.slideshare.net/AliaNajiha1/chapter-6-inhibitors-of-cell-wall-synthesis


564. Child patient need crown for the four anterior teeth and the permanent teeth
are good, the patient is uncooperative and you need to do all in one visit.
What is the conscious sedation?

565. You do pulp vitality test by:
a.CO2 ***
B. heat
c. Zn oxide
566. Female patient has a lesion in 46, 45 area, 45 is missing in radiograph it shows
radiolucent but there was radiopaque from the coronal portion of the tooththe lesion
extend buccal and lingual:
A. dentigerous cyst***
B. odontogenic keratocyst
C. ameloblastoma
Ameloblastoma: no teeth in it, it is radiolucent, age: 30-50
odontogenic keratocyst: it doesnt include teeth, age: 10-20
dentigerous cyst: young patients, radiolucent and radiopaque from the permanent tooth
and the lesion for sure extends buccal and lingual
567. 10 years old female in examination everything was normal but she has
unilateral cross bite and midline dont coincide:
A. tomography TMJ
B. anterio posterior x-ray
C. observe jaw function carefully
D. take impression***
568. Retentive forces in the denture:
A. function and para function forces
B. adhesion force ***
C. gravity force

569. Female first time need denture you do examination to soft and fault, you said
to pt. the denture will resist lateral movement but vertical movement cause loading
(removing) of denture what is type of vault:
OR
Old patient come need complete denture, after examination the dentist told him, lateral
forces will be good but vertical forces will break the seal:
a. U shaped palate
b. V shaped palate***
C. flat
D. paralyzed

Textbook of complete dentures, page 54

570. Female patient after 24 hours from complete denture delivery she came back
with redness and inflammation under denture, she take anti histamine:
A. sensitivity to denture material ***
B. anti-histaminic interaction
C. mouth sore throat

571. In Erupting permanent teeth present brown spot and not appear in deciduous
teeth, what is the cause:
OR
Brown spot on permanent tooth and no spot on deciduous the cause is:
1. Primary has less time of calcification
2. Placenta act as barrier for fluoridosis***
3. Patient brushing his teeth with fluoridated paste
572. Child need orthodontic treatment due to a fall on central incisor,
malocclusion:
A. class I
B. class II mod I
C. class II mod II
D. class III
573. After intrusion of primary incisor the most common effect on permanent
tooth is:
A. discoloration and enamel hypoplasia ***
B. it became yellowish to brown
C. crown displacement

574. Patient has hypercementosis and ankylosis:
a. Paget disease ***
B. cherubism
C. hyperparathyroidism

575. After replantation of 2 years the patient come back, radiograph show
ankylosis but without root resorptionso root resorption:
A. reduce greatly ***
B. occur after 5 years
C. occur after 2 years

576. Indirect retainer in removable partial denture used in:
A. class 1 ***
B. class 4
C. class 3

577. onlay restoration:
A. larger than amalgam restoration
B. smaller than amalgam
C. same as amalgam
D. depend on caries extension ***
http://books.google.com.eg/books?id=n00wduJKk90C&pg=PA293&lpg=PA293&dq=onla
y+preparation+depend+on+caries+extension&source=bl&ots=i3WUzXwsoI&sig=8hwr64
fTYCr0h9hGgBVAt3KRIuQ&hl=ar&sa=X&ei=wk_eUr3JKuqj0QXk3YDoBg&ved=0CDAQ6AE
wAA#v=onepage&q=onlay%20preparation%20depend%20on%20caries%20extension&f
=false

578. What is immediate implant:
A. put it immediately after extraction ***
B. put more than one in same time
C. put crown with implant (but temporary) ***
http://minidentalimplantsolutions.com/dentalimplantimmediate.php

579. Incipient caries in proximal detected by:
A. normal visualization
B. optic fiber light ***

580. Interproximal caries of anterior teeth is best detected by:
OR
The x-ray of choice to detect the proximal caries of the anterior teeth is:
1. Periapical X-ray film *** (specially on palatal surface)
2. Bitewing X-ray film
3. Occlusal X-ray film
4. None of the above
If anterior (periapical x-ray) if posterior (bitewing)
Bitewing for posterior teeth and distal surface of canine

581. Pt. have upper denture everything is normal( speaking , eating ) but the
upper lip showing short:
OR
Old patient with a new denture he came and said no problem in chewing or speaking or
anythingbut you noticed that upper lip is falling down and the vermilion border was
affected:
a. Vit. B deficiency
B. low vertical dimension***

582. An old patient had a complete denture, he came to you after delivery
complaining that the lower denture moves from the ridge when he just moves his
tongue, you put a (pip) paste and checked the denture and there is no any areas of
pressure or over extension, what is the problem?
A. Under extension of borders
b. Over extension of borders
c. High occlusal plane
d. Cramped tongue ***
If the tongue is cramped by the denture, lateral pressure will be exerted producing
instability when the tongue moves
http://books.google.com.eg/books?id=xuQobXlb40YC&pg=PA121&lpg=PA121&dq=%22i
f+the+tongue+is+cramped+by%22&source=bl&ots=PlybwDhYSS&sig=-
x85Zblc6wVPfKQjGUk2OPvaHZA&hl=ar&sa=X&ei=aBjeUpitO8KXhQfS3YHQDw&ved=0CC
oQ6AEwAA#v=onepage&q=%22if%20the%20tongue%20is%20cramped%20by%22&f=fal
se
583. Adjacent to PDL which bone the fiber insert to it:
A. bundle bone********
B. alveolar bone
C. cancellous bone
Bundle bone is a histologic term for the portion of the bone of the alveolar process that
surrounds teeth and into which the collagen fibers of the periodontal ligament are
embedded.
It can also be referred to as alveolar bone proper. Bundle bone is functionally
dependent in that it resorbs following tooth extraction or loss.
584. Patient has fracture in central incisor, the fracture involve enamel and dentine
only and no pulpal involvement, the patient have the fracture segment, what should
you do:
A. distract the fracture segment and restore with composite
B. do pulp therapy and put full crown
C. do pulp therapy and restore with composite
D. adhere the fracture segment in its place*****
http://www.dentaltraumaguide.org/Permanent_enamel-
dentin_fracture_Treatment.aspx
585. Anesthetic test (intraligamentary) used to localize the pain:
A. in specific tooth***
B. from upper and lower
C. from right to left in maxilla
D. in anterior teeth
http://www.dentalcare.com/en-US/dental education/continuing-
education/ce325/ce325.aspx?ModuleName=coursecontent&PartID=8&SectionID=-1

586. The most finish line used in all ceramometal crown:
A. shoulder
B. shoulder bevel ***
C. chamfer
Chamfer is used in metal crowns but in
Ceramo-metal "shoulder" or "deep chamfer" &
In all ceramic, it has "rounded shoulder" or "deep chamfer"

587. What type of pontic you will use in premolar area that will give illusion and
clearance:
a. Modified ridge lap****
b. Ovate
Modified ridge pontic: is modification for lap pontic that used on premolar region to
bring more illusion and clearance
Dental decks

588. Child want to do 4 anterior operation what type anesthesia you will use:
a. Nanoxia
b. Pes
c. HcL lidocaine***
Hydrochloride (Local Anesthetic)
Local or regional anesthesia in surgical procedures (including oral surgery)
http://www.drugs.com/monograph/lidocaine-hydrochloride-local-anesthetic.html

589. Child extract 45, 44 what will you do:
a. No ttt
b. Band and loop***
c. Interim

590. After Allograft what will you do 1
st
:
a. Clear it from infection like HIV and HBV ***
591. In full gold crown to prevent future gingival recession we but the margin at:
a. Gingival one third
B. gingival one fifth ***
592. You make ledge in the canal. You want to correct this. What is the most
complication occur in this step:
a. Creation false canal.
b. Apical zip.
c. Stripping. *** (= lateral perforation).
D. perforation. (= apical perforation).
593. What is the first sign if there is fracture in the face:
1. Fluid paranasal. ***
2. Diastic suture.
3. Overlap of bone.
4. All the above.
http://search.tb.ask.com/search/redirect.jhtml?action=pick&ct=GD&qs=&searchfor=+fi
rst+sign+of+face+fracture&cb=Y6&pg=GGmain&p2=%5EY6%5Exdm128%5ES07867%5Es
a&n=780b6203&qid=1dab7d70c65d41e5a3eb1f987c8d11da&pn=1&ss=sub&st=hp&ptb=
910F279C-C747-42CA-AB5D-
46E69F815A30&tpr=sbt&si=google_intlfromdoctopdf&redirect=mPWsrdz9heamc8iHEhl
dEQioGXfrih%2B04p8eAfoqoO0xOGSJ1UCiXuBcIiqTJdwrwUm9KczYzWSQuLCLCkrbSwoJ
mbWXNy0twOwsUDmqchk%3D&ord=2&

594. All of these are ways to give L.A with less pain except:
1. Give it slowly
2. Stretch the muscle
3. Topical anesthesia
4. Needle size over than 25 gauge ***

595. Sterilization in autoclave:
a. 20-30 min at 121c
b. 2-10 min at 134 c
c. A+B
d. None ***
.
Autoclave (121 C 15-20min) OR (134 C at 3min only)
596. Differentiate between autoclave and dry heat oven at 37.6C...Autoclave will
finish sterilization:
A. extremely before dry heat ***
B. slightly before dry heat
C. extremely after dry heat
D. Slightly after dry heat

597. Patient taking diazepam After injection there is erythema at the site of needle
puncture this due to:
1. Using large needle gauge
2. Injection in large vein
3. Prpelyne glycol in its component ***

598. Removal of dark color in dentin:
A. large excavator
b. Carbide bur with high speed ***

599. Three weeks after insertion of FPD marked discomfort to heat and cold occurs
there are no other symptoms the most likely cause is:
A. gingival recession.
B. unseating of FPD. (Will cause pain on chewing)
C. deflective occlusal contact
D. torsional forces on the abutment tooth
E. incomplete coverage of cut surfaces of prepared abutment teeth. ***
600. Amalgam restoration and there is also gold restoration in the mouth what
should the dentist do:
a. Change restoration
b. Put separating medium
c. Wait ***
D. put varnish
601. In the pulp:
a. Cell rich zone contains fibroblasts.
b. Cell free zone contains capillaries and nerve networks.
c. Odontoblastic layer contains odontoblasts.
d. All of above ***
602. Amalgam is used in extensive cavities:
a. When the cusp is supported by dentine and proper retentive preparation.
b. When cusps lost and thin supported wall. ***
c. When one cusp is lost and need to apply restoration to replace
603. Pt. has denture, after 5 years he complains of ulcer and inflammation in lower
buccal vestibule. What is the diagnosis:
a. Hypertrophic frenum. ***
b. Epulis fissurment
604. Patient presented to you after fitting the immediate denture 5 10 months,
complaining pain and over tissue in the mandibular. What is the diagnosis:
1. Epulis fissurment. ***
2. Hypertrophic frenum
605. What is the CT scan cut for zygomatic fracture:
(Oblique Parasagittal View for orbital fractures)
A. 0.5mm
B. 1-1.5mm ***
C. 15.2.5mm
D. 2.5-3.5

606. How much of minimal cortical thickness is required around neck of implant:
a. 1 ***
b. 2
c. 3
d. 4
www.ncbi.nlm.nih.gov/pubmed/19885399

607. Acute sinusitis is caused by:
A. Mixed aerobic and anaerobic bacteria
B. Streptococcus pneumonia ***
Acute sinusitis is usually precipitated by an earlier upper respiratory tract infection,
generally of viral origin, mostly caused by rhinoviruses, coronaviruses, and influenza
viruses, others caused by adenoviruses, human parainfluenza viruses, human
respiratory syncytial virus, enteroviruses other than rhinoviruses, and
metapneumovirus. If the infection is of bacterial origin, the most common three
causative agents are Streptococcus pneumoniae, Haemophilusinfluenzae, and Moraxella
catarrhalis. Until recently, Haemophilusinfluenzae was the most common bacterial
agent to cause sinus
http://en.wikipedia.org/wiki/Sinusitis
www.healthcentral.com/.../severe-sinusitis-caused-by-streptococcus-pneu

608. In a general dental practice setting, patients who are actively infectious with
Mycobacterium tuberculosis cannot be safely treated:
A. unless there is adherence to standard precautions. ***
B. even if standard precautions are used.
C. unless a preoperative antimicrobial rinse is used in addition to standard precautions
D. unless preoperative prophylactic antibiotic coverage and standard precautions are
used.
www.ada.org/2755.aspx?currentTab=2
609. Treatment of soft tissue sarcoma:
1. Chemotherapy
2. Radiotherapy
3. Surgery ***
4. Chemotherapy and radiotherapy.
1st choice is "surgery" if cannot remove all use radiotherapy, in other cases if cannot
make surgery or large in size we need to make both radiotherapy & chemotherapy
www.cancer.org
610. During maxillary 3
rd
molar extraction, the tuberosity fractured. It was firmly
attached to the tooth
What is the management:
1. Remove it with the tooth
2. Splint the tooth to 2nd molar then re-extracted after 6weeks***
3. Suture
611. Which of these canal irrigants is UNABLE to kill E.feacalis:
a- NaOH ***
b- CHX
c- MTAD
www.biopuremtad.com/documents/E_faecalis.doc

612. Cast soldered and porcelain are three types of:
A. fixed connector ***
B. movable
C. partial
D. both a and b
http://books.google.com.eg/books?id=SS1XfuLwsjsC&pg=PA204...

613. Child have dental caries in 3 or 4 surfaces of his 1st primary molar we will
replace them with:
A. preformed metal crown ***
B. porcelain crown
C. amalgam crown
D. composite crown
Indications for Use of Stainless Steel Crowns for primary molars:
Extensive caries damage involving multiple surfaces of the tooth
http://multimedia.3m.com/mws/mediawebserver?mwsId=SSSSSufSevTsZxtUnY_1P8mx
evUqevTSevTSevTSeSSSSSS--&fn=3M_ESPE_Crowns_F_EBU.pdf

614. What is the most effective technique to change behavior:
A. positive reinforcement
B. tell show do***
C. reward
615. Presence of parents in dental visit with child 3 or 4 yrs. encourage:
A. positive behavior ***
B. negative behave
C. no difference in both
D. no relation between child behavior and presence of parents.
http://www.autismspeaks.org/sites/default/files/section_5.pdf

616. Picture of RPD for patient 60 years complain began when she received the
denture pain in the retainers and all teeth also when the dentist remove the denture
there is multiple tissue ulcers under the fitting surface and pulp exposure in one of the
retainer
What is the problem with that denture:
a. Fitting surface need to be relived ***
b. Denture design haven't stress breaking action
C. no rest support

617. Defect in early development and formation of teeth:
1. Dentinogenesis imperfect
2. Amelogenesis imperfect
3. Enamel hypoplasia ***
4. oligodontia
Amelogenesis imperfecta (AI) presents with a rare abnormal formation of the enamel or
external layer of the crown of teeth. Enamel is composed mostly of mineral that is
formed and regulated by the proteins in it. Amelogenesis imperfecta is due to the
malfunction of the proteins in the enamel: ameloblastin, enamelin, tuftelin and
amelogenin.
Enamel hypoplasia is defined as an incomplete or defective formation of the organic
enamel matrix of the teeth in the embryonic stage of the tooth.
Oligodontia include six or more missing teeth, lack of development of maxillary and
mandibular alveolar bone height and reduced lower facial height. Variation in tooth
morphology is also observed along with problems in tooth development, eruption and
exfoliation. Possible causes of oligodontia include viral disease during pregnancy,
genetic predisposition, metabolic imbalances, developmental abnormalities and
environmental factors

618. Difference between acrylic teeth and porcelain teeth for complete denture:
1. Wear
2. Porcelain is a harder substance than acrylic making porcelain teeth more durable,
resist wear. Acrylic teeth are susceptible to abrasion
3. Strength
4. Porcelain teeth are more prone to chips and cracks compared with acrylic. Acrylic
teeth are less likely to break or develop fractures. Acrylic dentures may be a better
choice than porcelain if you need only a partial denture and your false teeth will meet
against your natural teeth. Porcelain causes excessive wear on natural teeth.
5. Bone Loss
6. Acrylic teeth are preferred for people who have bone loss in their jaws or unhealthy
gums, Porcelain transfers force with greater intensity than acrylic teeth.
7. Cleaning
8. Acrylic teeth resist staining, but they contain microscopic pits that can hold bacteria.
Brushing or quick soaking do not effectively remove the bacteria, although overnight
soaking does thoroughly clean the dentures. The bacteria remaining on acrylic teeth can
multiply rapidly and cause infection or bad breath.
9. Porcelain dentures must be handled carefully during cleaning. Porcelain dentures
should be cleaned over a sink filled with water or over a towel to reduce the risk of chips
if the dentures slip and fall
10. Porcelain and acrylic dentures are comparable in price, Acrylic dentures are more
quiet, similar to natural teeth, whereas porcelain teeth may cause a "clacking" sound
during chewing. Acrylic and porcelain dentures require regular checkups to ensure
proper fit, although acrylic dentures will wear sooner

619. Class I RPD need relining the best distal relining material for base:
a. Metallic
b. Acrylic ***
C. combination
D. saddle bar
620. Definition of epidemiology:
Epidemiology is the study (or the science of the study) of the patterns, causes, and
effects of health and disease conditions in defined populations. It is the cornerstone of
public health, and informs policy decisions and evidence-based practice by identifying
risk factors for disease and targets for preventive healthcare.
Epidemiologists help with study design, collection and statistical analysis of data, and
interpretation and dissemination of results (including peer review and occasional
systematic review). Epidemiology has helped develop methodology used in clinical
research, public health studies and, to a lesser extent, basic research in the biological
sciences.
http://en.wikipedia.org/wiki/Epidemiology

621. First visit for child 3 years for behaving, positively it depend on and most
affected by:
a. Home environment before visit care****
b. Service and care provide in clinic
622. Non resorbable membrane in GTR removed after:
a. 4-6 weeks ***
b. 8-12 weeks
c. 14-36 weeks
The initial membranes developed were nonresorbable (polytetrafluoroethylene [PTFE])
and, therefore, required a second, although frequently simple, procedure to remove it.
This second procedure was done after the initial stages of healing, usually 3 to 6 weeks
after the first intervention. The second procedure was a significant obstacle in the
utilization of this GTR technique, and, therefore, resorbable membranes were
developed.
Dental Decks 2011-2012 Periodontics, page 11

623. NiTi rotary manufacturer instruction for technique:
a. Step back
B. step down
C. hybrid
D. crown down ***
http://www.mecourse.com/ecourse/pages/page.asp?pid=186

624. Pt. 18 yrs. class II have badly decayed centrals need to be restore and there
minimal overlap, the best restoration:
a. Metal crown
b. Porcelain jacket
c. Veneer
d. Metal ceramic
e. Porcelain fused to metal***

625. Pt. child we put restoration after:
a. Base ***
B. caoh
C. varnish
D. just clean with water and dry***
626. Child 8 yrs. old, no caries just low carious index has missing bilateral canines
what is the cause:
OR
Bilateral missed canine in 7 yrs. child may be due to:
a. Congenitally missing ***
B. due to caries
C. early loss (exfoliation)
D. trauma
Maxillary primary canine
Eruption: 12-16 months
Exfoliation: 10-12 yrs.
Mandible primary canine
Eruption: 17-23 months
Exfoliation: 9-12 yrs.
Permanent canine erupted in Mn. 9-10 yrs. Max.11.12 yrs.

627. Fluorescent advantage of taking image for incipient caries:
a. Fluorescent system
B. diagodent
DIAGNOdent & use light fluorescence to detect incipient carious lesions.
http://link.springer.com/article/10.1007/s10266-013-0105-6#page-1
http://www.kavousa.com/US/DIAGNOdent/Classic.aspx

628. Pt. with sialolithiasis need sialography but pt. has sensitivity from iodine
injecting material what type of x-ray can we use instead:
a. Scintigraphy ***
Scintigraphy is indicated for the evaluation of patients when sialography is
contraindicated or cannot be performed
It has also been used to aid in the diagnosis of ductal obstruction, sialolithiasis, gland
aplasia, Bells palsy and Sjgrens syndrome.)
Burket's Oral Medicine
http://books.google.com.eg/books?id=Q2SP8cOZPvkC&pg=PA197

629. Best x-ray for TMJ implant:
a. Computed tomography ***
b. Arthrography
c. Panoramic
Tomography give better visualization of TMJ than plain x-ray (dental secret)
The complexity of structure and functions of (TMJ) make the diagnosis of its
diseases/disorders difficult. Remarkable progress made in the field of imaging of this
joint led us to compare four imaging modalities viz. plain radiographs, CT scan, MRI and
ultrasound. We found that MRI was most specific and sensitive for interpretation of soft
tissue and inflammatory conditions in the joint, whereas CT examination produced
excellent image for osseous morphology and pathology. Plain X-rays are useful for
destructive bony changes and sonography is a good in aid in diagnosing disc
derangement and is very economical

630. Child with red spots on different skin sites:
a. Make dental ttt normally
b. Delay ttt
C. Refer medical then delay ***
d. Make ttt then refer medical


Etiology and Epidemiology Measles is caused by an RNA virus in the Paramyxoviridae
The virus is spread primarily by airborne droplets and droplet nuclei generated by an
infected person during talking, breathing, coughing, and sneezing; by direct contact with
nasal or throat secretions
Signs and Symptoms of measles: generally begin about 7-14 days after a person is
infected, and include:
Measles Rash Skin of a patient after 3 days of measles infection.
Blotchy rash - Fever - Cough - Runny nose - Red, watery eyes (conjunctivitis) - Feeling
run down, achy (malaise) - Tiny white spots with bluish-white centers found inside the
mouth (Kopliks spots) A typical case of measles begins with mild to moderate fever,
cough, runny nose, red eyes, and sore throat.

631. Incipient caries but no cavity after clinical examination what we should do:
A. Composite
b. Preventive fluoride application ***
c. Fissure sealant
d. Amalgam
E. no ttt
Studies have shown that sealants can be placed over incipient caries, which arrests the
caries process most dentists choose to use air abrasion, a bur, or a laser to remove the
caries before the sealant is placed.

632. Acute exacerbation of chronic pulpitis:
a. Reversible pulpitis.
B. Irreversible pulpitis.
C. Acute periodontitis ***

633. Tracing with gutta percha in:
A. symptomatic periradicular periodontitis
B. acute exacerbation of chronic periapical abscess***
In primary endodontic lesions, there is an acute exacerbation of a chronic apical lesion.
Patients tend to experience varying degrees of pain. When tested, the pulp is necrotic.
Drainage can be seen through the PDL into the gingival sulcus or as a swelling in B/L
attached gingiva. A sinus tract may be present and is usually associated with deep
pocket depths. Radiographic bone loss will vary, depending on the avenue of fistulation.
A radiolucency may be present and is consistent with the origin of the lesion. It is
imperative to trace the sinus tract with gutta percha and take a radiograph to determine
the origin of the lesion. Typically, lesions will heal with endodontic therapy.

634. When give anesthesia for pt. on ttt for osteoarthritis lead to complication low-
pressure reach 90/75 low breath low pulse redness with sweating:
What complication we afraid of:
A. adrenal crisis ***
B. angina

One of the different medication used for treatment of osteoarthritis is corticosteroid if
the dentist not give steroid to pt. before the procedure he will develop adrenal crisis.
Corticosteroid Injections for Osteoarthritis Getting a shot of corticosteroids directly into
a sore joint can reduce pain and inflammation quickly and effectively.
Sign & Symptoms for Adrenal crisis includes:
Nausea Vomiting Fatigue Headache Fever- Reduced blood pressure
Electrolyte abnormalities - Low level of adrenocortical hormones
Low cortisol level Abdominal pain - Loss of appetite & Weight - Joint pain
Severe weakness - Increased heart & Breathing rate & abnormal sweating
Chills Dizziness Dehydration Confusion Coma - Slow movements

635. The different between Hedstorm file & K file:
OR
Hedstorm better than K file due to:
OR
H file more efficient in cutting than K file:
a. More positive rank angle***
b. More negative rank angle
c. Sharp tip
d. More in width

The H file has more cutting surface than the k file
The Hedstorm file has more than K-file:
hedstrom file has more flutes than reamer
hedstrom file is a type of k-files which has more cutting efficiency than other files

Definition of a rake angle is the angle between the leading edge of a cutting tool and a
perpendicular to the surface being cut.
Examples of negative rake instruments are reamers, K-files, K-Flex files, diamond burs,
most NiTi-files, and burnishing burs or regular burs run backwards.

636. Indication and contraindication of gingivectomy:
1- Elimination of supra bony pocket (pseudo Pocket)
2- Treatment of drug induced gingival hyperplasia.
3- Elimination of soft tissue creator.
4- To create clinical crown length for restorative procedure.
5. Create esthetic gingival form in case of delayed passive eruption of health teeth.
6. Gingival abscess.
7. Class I furcation involvement.
8. Elimination of gingival enlargements
9. Elimination of suprabony periodontal abscesses
Contraindication:
1- Acutely inflamed gingiva.
2- Inadequate oral hygiene.
3- Infrabony pocket.
4- Insufficient amount of keratinized tissue
5- Inadequate vestibular depth.
6- High caries rate.
7- Presence of interdental osseous creator.
8- Present of prominent oblique ridge.
9- Unacceptable appearance.

637. Fluoride tablets:
A. only swallowed
b. Chewing then swallowing ***
C. only chewing but not swallowing


638. The success of interapulpal injection depend on:
a. Type of anesthesia
b. Vasoconstrictor.
c. Back of pressure needle ***
d. Depth of penetrated needle
Advantage of the intrapulpal injection is it works well for profound anesthesia if given
under backpressure.
639. Patient with maxillary RPD with missing 16,17,18 and 25,27 and 26 is badly
decayed and a non-restorable tooth so the dentist decided to extract it what is the
change in the classification after extraction:
A. class I
B. class II modification I ***
C. class II modification II
D. Class III
ACCORDING TO KENNEDY APPLIGATE Class I, II and III RPDs Kennedy classification is
governed by the most posterior edentulous area that is being restored. For example, a
maxillary arch were missing teeth #1, 3, 7-10 and 16, the RPD would be Kennedy Class III
mod 1. It would not be Class I, because missing third molars are generally not restored
in an RPD
640. Instrument with same length and color:
a. 10 purple.
b. 30 blue.
c. 20 yellow.
d. 25 red ***
41
Because file length in endo 21, 25, 28, 31mm

641. In clinical examination there are distally food impaction in third molar area
with incipient caries, the best method to diagnose incipient and recurrent caries before
any x-rays is:
a. Visible light.
b. Transillumination fiberoptic light. ***
c. U V light
d. Digital X ray with Transillumination light

642. Simpifil type for canal enlargement by NiTi in:
a. Universal protaper
B. Reciprocal ***
c. Revers S

643. Pt. need surgery, hemodent:
A. zinc chloride
B. aluminum chloride

644. fluoride action by: bacterial clearance and mineralization
645. In maximum mouth opening:
a. Rotation of condyle around the disk. ***
b. Rotation of disk
c. One condyle forward and other with long axis

At this point, if the mouth continues to open, not only is the condylar head rotating
within the lower compartment of the TMJ, but also the entire apparatus (condylar head
and articular disc) translates. Although this had traditionally been explained as a
forward and downward sliding motion, on the anterior concave surface of the glenoid
fossa and the posterior convex surface of the articular eminence, this translation
actually amounts to a rotation around another axis. This effectively produces an
evaluate which can be termed the resultant axis of mandibular rotation, which lies in
the vicinity of the mandibular foramen, allowing for a low-tension environment for the
vasculature and innervation of the mandible

646. The muscle which do mandibular movement is:
a. Masseter M
B. temporalis
c. Middle ptyrgoid
D. lateral ptyrgoid ***
647. Pontic with illusion to tooth, all or nearly convex, allow for cleaning area:
1. Ridge lap (Concave)
2. Ovate.
3. Stationary
4. Modified ridge lap ***

648. FPD contraindication with:
A. anterior ridge with definite resorption ***

649. About electro surgery one not true:
A. used on thin tissue
B. no need anesthesia

Electro surgery
The use of electrically generated energy from high-frequency alternating currents to cut or alter tissue
within definite limits
Electrocoagulation
A technique using electrical energy to destroy the warts. Usually done for warts within the anus with a
local anesthesia, electrocoagulation is most painful form of therapy, and can cause both bleeding and
discharge from the anus
Electro desiccation
A type of electro surgery that desiccates tissue by dehydration, which employs a spark gap type of
generator to produce a highly or moderately damped alternating electrical current. It is usually used
to remove small superficial growths on the skin.

650. Pt. not anesthetized in 1
st
visit, 2
nd
visit he has trismus what you do:
A. Vaze Technique
B. Akinosi technique ***
651. More ductile and malleable:
a. Gold ***
B. titanium
c. Tungsten
652. Bone with less dimension coronoapically with type:
a. 1
b. 2
c. 3 ***
d. 4
653. Pt. has denture before 4 weeks ago, common complaining for difficult to wear
and remove the denture. During examination no caries. You will change the design, the
first design without rest seat, there are major connector with some projections from
base to hingue with lingual surface of anterior teeth .what is type of this design:
A. swing lock. ***
B. attachment design.

654. Patient came to your clinic complaining of discomfort to the base of denture
which is metal, with free end distal extension, the base only on crest, What is type of
design:
A. Gum stripper. ***
B. Flangeless
655. Pt. come with fracture tooth of upper central endodontically treated before,
you need to put post crown what is first thing to do:
a. Remove old composite and caries. ***
b. Gates Glidden for GP removal.

656. Pt. come with severe pain, no response to pulp test when you do percussion
the patient jump, diagnosis is: (No periapical change in radiograph)
1. Acute apical abscess
2. Chronic apical abscess
3. Symptomatic apical periodontitis. ***
4. Asymptomatic apical periodontitis.
Cohen's Pathways of the Pulp 10Ed 2011, Page 37

657. Patient has tooth treated endodontically but there is periapical lesion on apex
and you want to do re treatment with full and good debridement to canal:
A. Prepare tooth coronally
B. Surgery
C. Cleaning and shaping to anatomical apex

658. For retreatment of endodontically restored tooth, no complication only on
radiograph appear short, you reopen but failure to go apically, no stick sound
What is the cause of failure to go apically:
1. Perforation
2. Separate instrument ***
3. Mud
4. Ledge
659. If implant is success, the more reliable x-ray for planning the implant is:
A. periapical
B. panorama
c. CT scan ***
d- MRI

660. Indirect pulp capping for:
a. Reversible pulpitis. ***

661. Cavosurface margin for amalgam:
1. 90 Degree ***
2. Sound dentin***
3. Less stress area
662. Which of the following has mucosal change relation:
a. Hyperplasia
B. hypertrophy
C. dysplasia ***
663. Duration of nerve block anesthesia depend on:
a. Time of removal
b. Nerve bonding
c. Amount of anesthesia
D. 1+2 ***
664. Epidemic disease spread in one village, there are 1900 Person the infected
person in 2007 = 300, in 2008 = 150, the growth of disease:
1. 1600\150-1900\450 ***
2. 1900\300-1600\150
3. 1600\...................
665. Child come to clinic with his patent, he is mentally retard loss learning
capacities:
a. Cerebral palsy
b. Down's syndrome***
Churchill's PocketBooks Clinical Dentistry, Page 194
Learning disability: can be congenital (down syndrome) or acquired as result of (brain
damage pre-peri or postnatally)
physical disability: includes cerebral palsy, spina bifida, muscular dystrophy

666. Discoloration of single tooth:
A. tetracycline
B. amelogenesis
C. dentinogenesis
D. endodontic treatment ***
667. Pt. come with class IV (4) central fracture before near time, tooth with open
apex you do Ca(OH)2 pulpotomy success of treatment depend on:
a. No inflammation
b. Asymptomatic tooth
668. Small Radiolucent on X-ray for primary central:
The best treatment is pulpectomy
669. Recurrent case for orthodontic:
a. Movement of maxilla anterior downward.
b. Retroclination of mandible. ***
c. Widening of maxilla
670. bleeding on scalp, enter cranium from:
a. Subcutaneous connective tissue ***

671. Sharpening of curate you put its cutting edge at...to stone:
a. 70-90 degree***
b. 100-110
the correct is 70-80
672. Pt. with mucocele in the lip treatment is Excision ***
673. Female come with mass on left neck, slow growing before 6 years, the first
surgeon said it is a harmless sialodenitis, now CT scan show mass on submandibular
gland, your diagnosis:
A. sialodinitis
B. pleomorphic adenoma
C. adenoid cystic carcinoma. ***
http://www.google.com.sa/url?sa=t&rct=j&q=&esrc=s&source=web&cd=2&ved=0CDIQ
FjAB&url=http%3A%2F%2Fwww.cedars-sinai.edu%2FPatients%2FHealth-
Conditions%2FSalivary-Gland-Disease-and-
Tumors.aspx&ei=n07lUqryBsH50gWPjYGQDg&usg=AFQjCNFuBes_s9Cg_THN1QoHXlF6e
ndfUw
674. Pt. with swelling in submandibular gland moved during eating diagnosis by
sialogram:
A .Stone ***
http://www.google.com.sa/url?sa=t&rct=j&q=&esrc=s&source=web&cd=1&cad=rja&ve
d=0CCgQFjAA&url=http%3A%

675. Pt come with lesion between soft and hard palate. On histopathological
granulation tissue, metaplasia
What is your diagnosis:
A. metaplasia ***
676. You do crown lengthening for upper central incisor after lengthening you
should wait 6-7 months for:
a. Cementum formation
b. Periodontal maturation ***
c. Junctional epithelia formation
d. Epithelization
The tissue will take 6 months to fully mature, but the restorations can help a lot in
closing the space. To "fine tune" the tissue, some Juvederm can be placed to plump up
the tissues if you are not completely happy.

http://search.tb.ask.com/search/redirect.jhtml?action=pick&ct=GD&qs=&searchfor=+ti
ssue+healing+after+crown+lengthening&cb=Y6&pg=GGmain&p2=%5EY6%5Exdm128%5
ES07867%5Esa&qid=6fe18f00cc004ca4bef25cab0550af7e&n=780b6203&pn=1&ss=sub&
st=tab&ptb=910F279C-C747-42CA-AB5D-
46E69F815A30&tpr=sbt&si=google_intlfromdoctopdf&redirect=mPWsrdz9heamc8iHEhl
dETb9WD2HBuq1VmGk43wJBus31QhjvCARJ1L1GBSm%2B%2B0j3vpPiY%2BdeYnQqYVk
UhB0a6zGn3N3D2ECb3uGJrmOFY9N7mz%2Fy5k%2FzpgXMgN%2B%2F%2Fia&ord=6&

677. Bur used to make grooves for veneer or on composite veneer:
a. tapered round ended ***
http://search.tb.ask.com/search/redirect.jhtml?action=pick&ct=GD&qs=&searchfor=++
burs+that+used+to+do+grooves+in+veneer+prepration&cb=Y6&pg=GGmain&p2=%5EY6
%5Exdm128%5ES07867%5Esa&n=780b6203&qid=9827de7ab3524e7caf411d251901554d
&pn=1&ss=sub&st=hp&ptb=910F279C-C747-42CA-AB5D-
46E69F815A30&tpr=sbt&si=google_intlfromdoctopdf&redirect=mPWsrdz9heamc8iHEhl
dERaVlJ1927jeO%2F8t3zPndyHdFxwbQvJUCEiwNn3Hq%2BjHn7ykFG4om3guFNgChN346
pUv3bbQxEFyyIuNJavQ1iM%3D&ord=0&
678. Test for pemphigus:
a. Immunoflourosecnt test ***
nickolosky sign in the office and biopsy in laboratory
http://www.google.com.sa/url?sa=t&rct=j&q=&esrc=s&source=web&cd=2&ved=0CDU
QFjAB&url=http%3A%2F%2Fwww.nlm.nih.gov%2Fmedlineplus%2Fency%2Farticle%2F00
0882.htm&ei=X4XmUt7WMqzT7Aa90ICABg&usg=AFQjCNFKTURcr_66eF3k_j4cJIGrftncZg

http://www.google.com.sa/url?sa=t&rct=j&q=&esrc=s&source=web&cd=2&ved=0CDU
QFjAB&url=http%3A%2F%2Fwww

679. Test for diphtheria:
a. Schick test ***
http://www.google.com.sa/url?sa=t&rct=j&q=&esrc=s&source=web&cd=1&ved=0CC0Q
FjAA&url=http%3A%2F%2Fen.wikipedia.org%2Fwiki%2FSchick_test&ei=mH7mUoWBKo
eM7Qa4mYHYBA&usg=AFQjCNHMcyicsepQQsfB9fi6py47zBaqrQ

http://www.google.com.sa/url?sa=t&rct=j&q=&esrc=s&source=web&cd=1&ved=0CC0Q
FjAA&url=http%3A%2F%2Fen.

680. Why GIC make less caries rate after fall:
a. Sudden release of fissure
B. enamel take fluoride release of GIC ***

681. Detinogenesis imperfect:
A. during odontoblasts formation
B. involve dentin matrix ****

682. The forward relation of the lower jaw to the upper jaw is:
A. class II division I
B. class II division II
C. class III ***

683. If patient have history of respiratory depression and need extraction, what
antidote given:
a. Naloxone is given as a prophylactic antidote

http://books.google.com.sa/books?id=fBJbtIuhjYsC&pg=PA107&lpg=PA107&dq=antidot
es+to+prevent+respiratory+depression&source=bl&ots=bbaAsuQp-
U&sig=NzCwWG2ul9vfUtLNyhbMv3GzrSg&hl=ar&sa=X&ei=Y27lUufRGcLOhAe9xYCYBw
&ved=0CFkQ6AEwBQ#v=onepage&q=antidotes%20to%20prevent%20respiratory%20de
pression&f=false

684. Carter treatment:
OR
We found Carter how is it removed:
1. osteoplasty and Gingival curettage
2. osteomy
3. osteoectomy ***
4. gingivectomy
http://www.tard.tw/upload/publication/3-1.pdf

685. What tooth have Access opening like C:
a. Mandibular 2
nd
molar ***
.
A. C-shaped canal is characterized by:
1. Has complex internal anatomy
2. Is most commonly found in Asian populations.
3. Usually occurs in mandible 2nd molar
4. Should be referred to an Endodontist for treatment
686. Pt. has endocarditis, allergic to penicillin, what is the suitable:
1. Azithromycin
2. Clindamycin***
3. Erythromycin
Dental decks
687. Patient come to clinic wants to construct lower denture during the
examination you find wide and shallow frenum what is the suitable ttt before denture:
a. Z-plasty
b. Sub epithelial incision
c. Mucous incision
D. supraperiosteal incision of vestibuloplasty ***
http://books.google.com.sa/books?id=Jf9WZltV1BAC&pg=PA173&lpg=PA173&dq=wide
+frenum+in+edentulous&source=bl&ots=YzfiCA1CIR&sig=V4mwARjeA8H4aO_R9R0DRy
4MqRc&hl=ar&sa=X&ei=fYvlUo2DFua50QWyy4GwCg&ved=0CD8Q6AEwAg#v=onepage
&q=wide%20frenum%20in%20edentulous&f=false

688. The main purpose of gingivectomy for the patient with hyperplasia and the pt.
take cyclosporine:
a. Remove the pseudo pocket. ***
Pseduo pocket = Gingival pocket
Indications for gingivectomy is narrowed down to pocket elimination in suprabony
pocket and improve soft tissue architecture.
http://www.drbui.com/artgingivectomy.html

689. Patient come to office with complete denture acrylic in the upper jaw and
partial metallic Class l Kennedy denture in the lower constructed before one year pt.
complain was pain from 15 days in the lower anterior teeth and around of them what's
the suitable diagnosis:
a. Flexible free area ***
b. Allergy to acrylic
c. Rough surface in the denture base
d. Sharp end of the flange
Natural teeth will show bone resorption
690. Finishing line of full metal crown: chamfer ***
691. What is the main function of adrenalin during the endodontic surgery:
a. Vasoconstrictor
b. Reduce the system of lidocaine
c. Increase the duration of anesthesia
d. Hemostatic agent *****
Local vasoconstrictor in local anesthetics or gingival retraction cords. 26, 27 its also used
in endodontic microsurgery as a hemostatic agent to control the bleeding in bone cavity.

692. Lichen Planus:
Lichen planus (LP) is a disease of the skin or mucous membranes that resembles lichen.
The cause is unknown, but it is thought to be the result of an autoimmune process with
an unknown initial trigger. There is no cure, but many different medications and
procedures have been used to control the symptoms.

693. ANUG:
Acute necrotizing ulcerative gingivitis = Trench mouth
Is a common, non-contagious infection of the gums with sudden onset.
The main features are
Painful, bleeding gums, and ulceration of inter-dental papillae this disease, along with
necrotizing (ulcerative) periodontitis (NP or NUP) is classified as a necrotizing
periodontal disease, one of the seven general types of periodontitis.
The often-severe gingival pain that characterizes ANUG distinguishes it from the more
common chronic periodontitis, which is rarely painful.
The causative organisms are mostly anaerobic bacteria, particularly Fusobacteria and
Spirochete species.
Predisposing factors: poor oral hygiene, smoking, malnutrition, psychological stress and
immunosuppression when the attachments of the teeth to the bone are involved, the
term NUP is used.
Treatment of ANUG is by debridement and antibiotics (usually metronidazole) in the
acute phase, and improving oral hygiene to prevent recurrence. Although the condition
has a rapid onset and is debilitating, it usually resolves quickly and does no serious
harm. The synonym "trench mouth" arose during World War I as many soldiers
developed the disease, probably because of the poor conditions and extreme
psychological stress.

694. Pt. need to extract upper left 6 when anesthetizing the following occur to the
patient, Paleness, nausea, mental confusion, clinical examination of blood pressure
100/75 warm extremities and the patient medical history indicate, he was on ttt for
osteoarthritis:
What's the diagnosis:
a. Hypotension
b. Hypoglycemia
c. Hyperglycemia
d. Adrenal insufficiency ***
Previous tx for osteoarthritis means pt. taking cortisone, which produce such symptoms
after local anesthesia.

695. Which were excellent and detailed dentures as he said what the type of this
patient:
a. Philosophic
b. Hysterical****
c. Mental retarded
696. What is the most important teeth to prevent the severity of crowding:
a. Upper E
b. Upper D lower E
c. Lower d ****
697. The most common metal used in used in FPD:
a. Titanium
b. Platinum
c. Alloy ***
Alloys that melt between 180 and 190 C (360 and 370 F) are the most commonly used.
http://en.wikipedia.org/wiki/Solder
698. The most common advantage of wrought wire clasps than the casted clasp:
a. Flexibility***
b. Less irritation to abutment teeth
http://intranet.tdmu.edu.ua/data/kafedra/internal/stomat_ortop/classes_stud/en/sto
mat/ptn/Orthopedic%20stomatology/3/17.%20Classification%20of%20clamps,%20indic
ations%20for%20use..htm

699. Cell response about remodeling of PDL in orthodontic treatment:
OR
What is responsible in the remodeling of the periodontal ligament in orthodontic
treatment:
A. fibroblast. ***
B. osteoblast
C. osteocyte.
D. osteoclast.

Osteoclasts responsible for bone resorption are mainly derived from the macrophages.
Osteoblasts are produced by proliferations of the cells of the periodontal ligament

700. One plane labial reduction of crown:
a. Over contoured***
b. Occlusal disharmony
c. Increase retention

701. Most reason for failure of crown:
a. PDL
b. Secondary caries***
http://www.google.com.sa/url?sa=t&rct=j&q=&esrc=s&source=web&cd=1&cad=rja&ve
d=0CDAQFjAA&url=http%3A%2F%2Fwww.medicinaoral.com%2Fodo%2Fvolumenes%2F
v4i3%2Fjcedv4i3p167.pdf&ei=c-
vmUpXfCsmX7QaB1IDoCg&usg=AFQjCNFyH9mWiAfk_GEtCl_URfgMZzkvJg

702. How L.A decrease saliva flow:
A. block cholinergic receptor ***
B. decrease anxiety
http://hirnforschung.kyb.mpg.de/fileadmin/uploads/files/Methoden/Bet%C3%A4ubungsverfahren_u
nd_Chirurgie/SOP_DrugDescriptions.pdf

703. Most type bacteria in canal:
A. streptococcus
B. treponam pevellona
C. faecalis ***
the type of bacteria found within a failed root canal may differ from the normal infected
tooth. Enterococcus faecalis and/or other facultative enteric bacteria or
Pseudomonassp. are found in this situation

704. Conditioning of resected root end by:
1. Citric acid.
2. EDTA. ****
3. Phosphoric acid.
4. H2O2
.
Conditioning of resected root end use in resected root end:
1. Citric acid 50% (pH 1) for 2-3 min.
2. EDTA 15-24% (pH 7.3) for 2 min (produce the optimum root surface)
3. Tetracycline HCl (pH 1.8) for 30 sec.
Studies Comparing the effect of a 3-minute application of either EDTA (pH 7.3) or tetracycline HCl (pH
1.8) showed no significant difference in the treated tooth surfaces,37 but EDTA has been shown to be
more favorable to human PDL cell attachment.
The most usual used conditioning of resected root end (EDTA)
Cohen's Pathways of the Pulp 10Ed 2011, Page 746

705. Which of the following is not true about ultrasonic:
A. oscillates between 1500 to 3000Hz ***
B. release energy known as cavitation
C. acoustic steaming
D. implosion
Ultrasonic No. cycle
1. Magnutorestrictive 25000-40000 rpm
2. Pezoelectrive 60,000-80,000 rpm


X-Ray important informations:
Submentovertex: for diagnosis zygomatic arches
Occipitomental: Best radiograph for fracture of the middle third of face (Nasoethmoid f)
Reverse town's: for condyle & subcondylar region
Water's x-ray: view for maxillary sinus and midface fracture.
Panorama: full mouth examination
Lateral cephalometric: for orthodontic
Post. Anterior "PA" view: for vertical favourable and unfavourable fracture of the angle
Lateral Oblique 30: for horizontal favourable and unfavourable fracture of the angle
Lateral Oblique 15: for Ramus and body of the mandible
Bitewing x-ray: for caries (interproximal) in post. Teeth
MRI: for TMJ & is very good for soft tissue (therefore for TMF as disc is soft tissue)
CT scan: is for good for hard structures.
706. Relation between alveolar bon crest and CEJ
707. Function of saliva?
708. after root fracture the best type of healing is
709. which muscle not included in facial expression:
710. For eruption of permanent teeth:
Max. 61245378 ***
Man. 61234578 ***
( . )
Natal teeth: are teeth that are present at birth.
Neonatal teeth: are teeth that emerge through the gingiva during the first month of life
711. Pt. come to make new denture while he has made 3 dentures already:
712. HBV pt. injured during dental practice HOW TO DEAL
713. A cyst superior or inferior pharyngeal arch

714. Rideal-Walker test is the test for detecting activity of:
a. Sterilization by dry heat
b. Antibiotics
c. Disinfection ***
d. Sterilization by wet heat

715. Girl pt. came with her parents to your dental clinic , she was fulfaire &
shyness, What should dentist give her : (Medication )
A. diazepam
B. promethosone ***
c. Methazolomaide

716. The incisal guide should be set on the articulator at:
a. Zero degree. ***
B. 20 degree.
C. 40 degree.
d. None.

717. How we can reduce composite polymerization stresses:
A. Increase incremental & using soft light cure ***
Strategies to Reduce Shrinkage Stress in Clinical Procedures:
1- Incremental Layering Technique
2-Stress Absorbing Layers with Low Elastic Modulus Liners
3- Light Curing Procedures soft start light.

718. What of this surfaces ONE not contributed to Denture surfaces:
A. impression surface
B. polished surface
C. vestibule surface ****
d. Occlusal surface
Sir Wilford Fish described 3 surfaces of a complete denture:
Impression surface
Occlusal surface
Polished surface
719. Case + picture for broken upper maxillary denture from ant. Border to the post
dam area , the pt. suffering from severe bridge resorption:
What is the main cause of fracture?
A. decrease or increase in VOD


720. Most cement irritation to the pulp is:
A. Zinc phosphate ***

721. type of disinfection used after HB patient:

/////////////////////////////////////////////////////////////////////////////////////
//////////////////

722. AH26 means:
A. Add ZOE
B. Accelerator
C. Sealer with epoxy resin ***

723. Composite matrix band consist of:
A. AHTCH
B. bisc GMT ***
C. TRHC
724. Main cause of destruction for patient with RPD:
A. Rigidity of major connector
B. Flexibility of major connector ***
C. Shape of major connector
D. Sharp angles of major connector
Flexibility allows forces to be concentrated on individual teeth or edentulous ridges causing damage
to those areas

725. Pt. 20 yrs. fall on his maxillary central tooth was more
than 60 min, what should do to make the tooth vital inside the socket:
A. Sodium sulfate
b. Sodium chloride *** = (Normal Saline)
c. Sodium hydrate

726. 47 years old patient has pleomorphic adenoma in left said of hard palate,
what will you do:
A. chemotherapy
B. radiotherapy
C. surgical remove ***
D. leave it and disappear gradually

727. The first calcification of primary teeth appear in utero start at:
A. 6 weeks
B. 11weeks
C. 14 weeks ***
D. 16 weeks
http://en.wikipedia.org/wiki/Tooth_development

728. In Implantation of TMG, what type of X-ray used:
A. CT***
B. MRI
C. panorama
D. tomography

729. Patient has deep vein clotting and taking aspirin, He must stop the aspirin
before any surgery:
OR
Patient with deep thrombosis he want to make dental surgery and he is undertaking
aspirin how many days u told to him to stop:
OR
Patient taking aspirin how many days you tell him to stop before surgery
1. 3 days ***
2. 7 days
2. 10 days
3. 15 days

730. 20 years old female patient with good health she put removable ortho
appliance before 2 months, The patient complain from bleeding with tooth brush:
A. gingival hyperplasia
B. gingival lengthen
C. mild gingivitis
D. deep pocket ***
http://www.dentistryiq.com/topics/device/mobile/t/58686758/ortho-perio-
connection.htm?m_n=true

731. Patient has lesion firm attachment, fixed neck nodes are most to be detected
in association with:
a. ameloblastoma.
B. basal cell carcinoma.
C. fibroma.
D. squamous cell carcinoma. ***

732. What's the cause that elongate L.A. effectiveness:
A. bonding between LA and nerve
B. bonding and rate of anesthesia ***
C. fast injection


733. When you will splint primary teeth:
A. bone fracture ***
B. luxation
C. avulsion
D. laceration

734. The basic difference between K files and reamers is:
a. The number of spirals or flutes per unit length. ***
b. The geometric cross section ***
c. The depth of flutes.
d. The direction of the spirals.

735. EDTA removes:
a. Calcified Tissue ***
736. Endochondral ossification occur in:
A. mandibular body
B. maxillary suture
C. cranial suture
D. ....condrosis
737. Tooth arteriole diameter is:
1. 5 micron
2. 10 micron
3. 15-25micron
4. 45-55micron ***
738. Thyroid gland derived from 4
th
pharyngeal arch:
a. All parts
b. Follicular c cells ***
c. Inferior part
d. Superior part

739. What last tooth formed in ugly duckling stage:
A. lateral incisor
B. canine ***
c. 1st premolar
740. Naocl irrigant responsible of:
A. dissolve inorganic
B. pulp tissue and organic***
C. microorganism
Sodium hypochlorite is generally considered the best irrigation as it is bactericidal and
dissolves organic debris
741. Recognize this slide:
A. Giant cell granuloma ***


742. During which period of intra uterine life the primary tooth bud is formed:
A. 4 weeks
B. 5 weeks
C. 6 weeks **
D. 8 weeks

743. After you take elastomeric impression material you make disinfection using
spray for:
A. entire tissue surface
B. tissue surface and bottom
C. entire tissue surface, bottom and tray ***

744. Pt. has undercut posteriorly over crest removed by:
A. blade 11
B. blade 12 ***
C. blade 15

745. Part of the prosthesis that connect the abutment to other parts:
a. Major connector
b. Minor connector ***
c. rest
d. clasp

746. X-Ray show:
a. Dead pulp
b. Pulp calcification
c. Pulp damage
d. calcified canal ***

747. 56 yrs. old patient has a sharp pain in right side of face, this pain is painful and
aggressive, increase during eating and speaking not cross midline, when touching left
cheek patient feel pain:
a. bell palsy
b. trigeminal neurologia
c. psychological pain
d. . syndrome
748. When bacteria connect to each other to form biofilm (plaque),this process call:
a. Srgnolling ***
749. Child patient, loss hearing , notch incisor, copper color lesion:
a. congenital syphilis ***
750. pH of Saliva:
a. 5.5
b. 6.5 ***
c. 7.5
pH of saliva:
Alkaline 6.5
Acidosis 7.4
751. Water irrigation device:
OR
What does the water irrigation system do
A. prevent plaque formation
b. Completely remove tag's plaque
C .dilute bacterial toxin***
Water irrigation removed 99.9% of plaque biofilm from treated areas
Follicular biofilm is a layer work as connecter between plaque and tooth
http://en.m.wikipedia.org/wiki/Oral_irrigator

752. Pt. get blow in his mid-face and there continuous fluid come out of his nose,
what's the main content the paranasal fluid:
1. Highly oxidate blood
2. Highly protein
3. Blood with CFS ***
753. What is the cement that if mixed slow, give best working time:
1. Zn phosphate ***
2. ZOE
3. GI
4. Resin
754. What is the size (Volume) of Maxillary Sinus:
A. 14 ml ***
B. 20 ml
C. 9 ml
D. 30 ml
Maxillofacial Secret 2
nd
Edition
The average size of the maxillary sinus is 14.75 ML
with a range of 9.5-20 cc. On average, the width is 2.5 cm height, 3.75 cm and depth 3 cm.
The average volume of the sinus is 15ml
http://www.scientificdentalclinic.com/articles/maxillary-sinus.asp

755. How many cusp ridges:
a. 1
b. 2
c. 3
d. 4 ***
Woelfels Dental Anatomy 8Ed 2012, Page 18
http://books.google.com.sa/books?id=UNtzFQszCuQC&pg=PA18&lpg=PA18&dq=cusp+h
as+how+many+ridges&source=bl&ots=d1LNG_btKb&sig=U7xAcZM6x1j8l76_sXWqSyhGL
4Y&hl=en&sa=X&ei=YgAZU8nxHOGL7AaHk4HoCQ&ved=0CCQQ6AEwAA#v=onepage&q
=cusp%20has%20how%20many%20ridges&f=false

756. Mesiodistal width of gingival seats of class II amalgam restoration is In:
millimeters:
A. 1 mm
B. 0.5 mm
C. 1.5 mm ***
D. 2 mm
Extend to cariously involved fissures only
initial pulpal depth: 1.5
smooth curves
isthmus = 1/4 intercuspal width
gingival floor width = 1-1.5 mm axially
axial wall follows outer contour of tooth
marginal ridge width = 1.6 mm

757. Bucco lingual width of artificial posterior teeth must be:
A. same as natural teeth
B. greater than natural teeth
C. lesser than natural teeth ***
D. has no relation to natural teeth
758. When you give sedative inhalation for patient to prevent hypoxia u give:
A. 95% oxygen and 5% nitrous oxide
B. 90% oxygen and 10% nitrous oxide
C. 85% oxygen and 15% nitrous oxide
D. 100% oxygen and zero nitrous oxide ***
759. Long question For patient has fracture of condyle what is the most suitable
graft taken from:
A. anterior iliac graft
B. posterior iliac graft
C. post chondra graft ***
The Best graft usually used for condylar replacement is Post chondra graft

760. Patient come back after 24 hours of insertion of upper and low dentures with
severe pain in denture bearing area:
A. denture stomatitis
B. overextended flanges ***
C. suprocclusion
D. denture hyperplasia

761. Child come back after class 2 amalgam restoration with fracture in isthmus
area that soon placed what is the cause of that fracture :
A. improper manipulation of amalgam
B. excessive flaring of cavosurface walls
C. high occlusion ***
762. The type of retractor that used to retract the flap and check in the same time:
1. Sedten retractor
2. Meanostea retractor ***
3. Wolder retractor
4. Senn retractor

763. Main disadvantage of full mucoperiosteal flap is:
A. Bad accessibility to area of operation
B. Interdental papilla integrity
C. Healing with scar ***

764. A minimum of two points of fixation should be used to provide stable internal
fixation of mandibular symphysis fractures:
A. Apply the first plate to the inferior border of the mandible.***
Use a 1.5 mm drill bit with 6 mm stop to drill monocortically through the plate hole next
to the fracture
B. Apply the first plate to the inferior border of the mandible.
Use a 1.5 mm drill bit with 6 mm stop to drill biocortically through the plate hole next to
the fracture.
Insert a 2.0 mm screw, 6 mm in length a 2.0 mm screw, 6 mm in length
https://www2.aofoundation.org/wps/portal/!ut/p/c1/04_SB8K8xLLM9MSSzPy8xBz9CP
0os3hng7BARydDRwN3QwMDA08zTzdvvxBjIwN_I6B8JJK8haEFUD7U09nP2MkPqNSEgG
4_j_zcVP2C3IhyADJvFMw!/dl2/d1/L2dJQSEvUUt3QS9ZQnB3LzZfQzBWUUFCMUEwRzEw
MDBJNklGS05UMzIwTzI!/?showPage=redfix&bone=CMF&segment=Mandible&classifica
tion=91-
Symphysis+and+parasymphysis%2C+simple&treatment&method=ORIF&implantstype=T
wo+plates&approach&redfix_url=1285234125860

765. The 2x4 appliance is used to correct:
A. anterior cross bites and restore anterior aesthetics in the mixed dentition stage. ***

766. Bluegrass Appliance for:
A. tongue thrust ***
Also for (Thumb sucking habit breaking appliance)
767. An 8 years old boy presents with class III fracture of tooth#11, which appeared
an hour ago, the apex is not closed. Your treatment should be:
A. Direct pulp capping with Ca (OH) 2.
B. Pulpectomy follows by RCT.
C. Pulpotomy and fill with Ca (OH) 2. ***
D. Smoothening of ledges and restore with composite.
E. Restoration with Glass ionomer cement.


768. A stabilized root fracture with evidence of hyper-calcification of pulpal space
requires:
A. No further treatment. ***
B. Endotherapy with gutta percha.
C. Endotherapy with Ca (OH) 2.
D. Surgical removal of apical segment.
E. Post retained crown.

769. After the tooth completely erupted :
A. Ameloblast form primary cuticle ***
B. Ameloblast form secondary cuticle
C. Ameloblast become much longer
770. How much the suitable time of follow up for treated ameloblastoma:
A. 10 years
B. 5 years
c. Persistent ***

771. What is the pocalin plumer:
a. Bonding agent ***
b. Sealer
c. Sealant
772. Subcostal nerve supply:
a. The skin over lip ***
773. Patient in dental chair has slow and high breath what is this condition:
a. Hypoventilation
b. Hyperventilation
c. Cardiac disease ***
hypoventilating is breathing slow and shallow... not breathing enough... not blowing off
enough CO2
hyperventilating is breathing too fast and rapidly... makes you blow off too much CO2
In cardiac disease their will be shortness of breath which known as dyspnea were you find
breath faster and shallower breath
774. Child take antitatinous vaccination before 5 years now he get injured with big
wound what u will give him:
a. Toxoide ***
b. Toxiode and antitatanous
c. Antitatanous

775. What are the teeth.those mostly have ectopic eruption:
A. mandibular incisors
B. maxillary premolars
C. maxillary incisors
D. maxillary canine ***

776. For Hepatitis B only Antigen's HB is to check:
a. Immunity ***
2. Acute Infection
3. Chronic Infection

777. MAF is:
a. The first file that has binding retention at the working length ***
MAF = Master Apical file (used for recapitulation)

778. The most common cause of the discoloration of non-vital teeth:
A. stains from drinking coffee
B. hemorrhage of the pulp
C. toxins from chronic illnesses***
779. You did RCT and then the patient complains of persistent pain. Radiograph
shows that it is not properly filled--short at the apex as the question implied. What will
you do next:
A. do surgical treatment
B. analgesic and antibiotics
C. start retreatment ***

780. What is not a test for pulp vitality:
a. Ice
b. Hot Water
c. Percussion ***
d. Cavity Test
Percussion isn't for vitality test ... it is a test to know periradicular tissue condition
periodontal abscess may cause pain on percussion, yet... the tooth is vital
781. Common sites of dens invaginatus:
A. Maxillary centrals
B. Maxillary laterals ***
C. Mandibular Premolars
Dens Invaginatus= Dens in dente (tooth within a tooth)

782. GIC
A. adheres to dentin and not enamel
B. not biocompatible
C. releases Fluoride. ***

783. In dentinogenesis, what occurs:
A. Odontblast mature and become tall cuboidal cells.
B. Dentin matrix is involved ***

784. Patient came back trismus:
A. William's technique
B. Gow-gates technique
C. Akinosi technique ***

785. Patient has mentoplasty which nerve should the doctor be careful for:
A. Incisive
B. Mental ***
C. Infraorbital


786. Fibers that are embedded in the cementum?
A-Transceptal Fibers
B-Sharpe's fibers ***

787. Reversal lines in bone is by:
A-Osteoblast
B-Osteoclast ***
C-Osteocytes

788. A child has an anterior cleft, this happened when:
a. 4-6 weeks intrauterine ***

789. Patient taking systemic antifungal medication, he is most likely taking:
a. Nystatin
b. Fluconazole ***
c. Amphotericin B

790. 13 year old patient, did check with an explorer positive on a newly erupted
tooth, the rest of the teeth are non-carious?
A. Pit and Fissure sealant ***
B. Restoration with Composite
C. Restoration with Amalgam
D. Topical Fluoride

791. Patient went to your clinic with pain on a tooth and cannot tell which arch:
A. Electric Test
B. Anesthetic Test ***
C. Cavity Test

792. Which tooth tooth will most likely go to the maxillary sinus:
A. Maxillary first molar ***
B. Maxillary second molar.

793. In GV Black classification, the angle is:
a. Third number ***

794. The physiologic criterion of occlusion is:
A. Esthetics and Phonetics
B. Occlusal Stability ***
C. Effective mastication

795. Space needed in the primary teeth to accommodate the erupting permanent
teeth and incisor liability:
A. 3 mm
B. 4mm
C. 6mm ***
D. 8mm
In Mandible: 6 mm
In Maxilla: 6-7 mm

796. What causes the growth of the alveolous:
a. Eruption of teeth. ***

797. An 8 years old child has an amalgam restoration at the isthmus area. What
caused this:
A. faulty manipulation of the amalgam ***
B. extended cavosurface margin
C. extended occlusion***

798. Surgeon will make a mucoperiosteum flap, What instrument will hold the flap
and can also retract the cheeks:
A. Minnesota retractor ***
B. Seldin retractor***

799. Description about permanent and primary teeth then, Hand and foot keratosis
A. Leferve-Papillon Syndrome ***
B. Hypophosphatasia

800. RPD photo with the metal Framework. Abutment 23, 11 and 13. Patient
complains of pain and minor ulcers around the neck area of the abutments and on the
free saddle area. When patient removes it, it gives her relief. What is he cause?
A. Insufficient relief on the denture base. ***
B. retentive clasp is not rigid
C. design without stress breakers
801. Impression that can be poured more than once:
a. Alginate
b. Addition ***
c. Condensation Silicon

802. Patient wants to convert her unattached keratinized pocket to a keratinized
gingiva:
a. Apical Periosteal Flap***
b. Coronal Periosteal Flap
C. Widman Flap


803. Biopsy is least useful in the diagnosis of
A. Geographic tongue ***
B. Aphthous ulcer
C. Cysts
D. Granuloma
E. Myeloma

804. Persons who are working in glass factories they have the disease:
1. Silicosis. ***
2. Asepsis

805. Laser used in endodontic is: (Root canal preparation)
1. Co2.
2. ND (YAG) ***

806. Treatment of a periodontal abscess caused by a foreign body is
A- Gingivectomy
B- Antibiotic
C- Elimination of the pocket
D- None of the above ***
Enamel reduces permeability and increases hardness with age
807. Child 6 years old came to u with thumb suck already caused dental problem
what u will do:
1. Nothing.
2. Psychologist treatment.
3. Early habit breaking appliance. ***

808. Rubber dam is important because it:
1. Improves safety. ***
2. looks scientific.
3. Improves suction.

809. Preparation of gold crown with excepts of gingival rescission the most proper
to extent the preparation:
A. 1mm under the gingival margin
B. Make it on the fifth.
C. Make it on the third.
D. At the gingival. ***

810. Patient came to your clinic and you decide to extract his remaining teeth and
do immediate denture why will you do that denture:
1. To restore appearance ***
2. To restore speech and mastication
http://books.google.com.lb/books?id=ZGvegIdicUoC&pg=PA255&lpg=PA255&dq=compl
ete++immediate+denture+mainly+for&source=bl&ots=EvRnIkzvze&sig=2gV9WnVEEccOt
XqV0jviWZSqdqU&hl=en&sa=X&ei=7gMkU4DYM4aj0QXdtIC4AQ&redir_esc=y#v=onepa
ge&q=complete%20%20immediate%20denture%20mainly%20for&f=false


811. Child have enamel and dentine hypoplasia what will you do for his vital teeth:
1. Porcelain crown ***
2. Zinc phosphate cement

812. Patient with proximal caries and while you do proximal box you didn't found
gingival step what will you do:
1- Extend to root
2- Make step with amalgam
3- Make step with composite
4- Make step with glass ionomer

813. what is the principles of GTR in wound healing:

814. Old patient take nasal discharge for long time good oral hygiene suffer from
gingival hyperplasia, In upper anterior maxilla what is your diagnosis:
a. Rhinoscleroma
B...


815. Class III occlusion means:
A. lower distal surface of first molar distal to distal surface of upper first molar
B. lower first molar distal surface mesial to distal surface of upper first molar ***
C. at same level
D. none of above

816. Second canal in lower canine:
A. mesial
B. distal
C. labial
D. lingual ***

817. Age 51 female came to u with upper complete denture 2 years ago. Now
suffering from altered test and burning sensation, u check the denture there was perfect
seal and occlusal rest in a position the burning sensation from:
OR
51 year old woman symptoms of burning mouth syndrome. With laboratory data below
What is the cause:
A. viral infection
B. menopause ***
C. Vit D12
Textbook for complete denture, page 414

818. child came to u after anterior trauma on clinical examination the 4 anterior
was unalignment:
A. luxation ***
B. subluxation
C. dentoalviolar fracture

819. Normal pattern of tooth eruption
1. The crown before the root
2. The crown and root together in opposite way
3. The crown and root in the same way
4. the root before crown ***
820. The acceptable amount of bone resorption around an implant is:
a. 1-1.5 mm ***
b. 0.5 mm
c. 3 mm
d. 2.4 mm
821. Dentist after finishing preparation for full veneered crown put a groove on
buccal surface as a final step...why:
A. indicat for seating of crown
B. antirotational in cementation ***
C. to prevent mesiodistal movement
822. New born the mandible is separated in the midline by:
a. Synovial joint
b. Fibrous joint
c. Catrilagenous tissue
d. Fibrous tissue ***
823. The best way to detect the pulp health and integrity is:
A. thermal test ***
B. electric test
C. percussion
D .palpation
824. The most reliable to take prophylactic dose before dental surgery:
A. pregnant
B. rheumatic fever ***
C. hypertension
D. hemophilic
http://emedicine.medscape.com/article/1672902-overview#aw2aab6b4


825. Patient has Removal Partial Denture and complain of Ageing appearance:

A. Extensive decreased vertical dimension ***
B. insufficient freeway space
C. oblique occlusal plane

826. Ptrygoid process has two origin ...
1. From temporal
2. sphenoid ***

827. Which chromosome cause mongolims:
A. disomy 21
B. trisomy 21 ***
the extra chromosome that cause mongolism is trismoy21

828. Least common place of squamous carcinoma:
A. nasopharnyx ***
829. the best cavity liner of Ca (oh)2:
1. Light activated resin type.
2. Regular aqueous or methylcellulose. ***
3. Ledermix.
4. All of the above.

830. The trauma that has less effect in pulp:
A. concusion
B. infraction ***
C. complicated crown root fracture
D. Non complicated crown root fracture
Enamel infraction is an incomplete fracture (crack) of the enamel without loss of tooth
structure so if think it has minimal effects on the pulpal tissue but concussion is also
confusing it is defined as an injury to the supporting tissue only

831. Which material used in one-visit pulpectomy:
a. MTA *** (Mineral Trioxide Aggregate)
b. Ca (OH) 2 + CMCP
C. Formocresol
D. Zn oxide
832. Pt. came to u with dark color left lateral incisor , the pt. history trauma and did
RCT for the left lateral incisor, what is the cause of this coloration:
1- Bacteria in the coronal part
2- Microleakge of the restoration
3- Blood spot in dentinal tubules which cuz discoloration ***

Pulp extirpation can cause hemorrhage in the pulp chamber caused by rupture of blood
vessels. Blood components subsequently flow into the dentinal tubules, causing a
discoloration of the surrounding dentin (41, 42) initially, a temporary color
Change of the crown to pink can be observed. This is followed by hemolysis of red blood
cells. The released heme then combines with the putrefying pulpal tissue to form iron
(26, 43) The iron in turn can be converted by hydrogen sulfates that are produced by
bacteria to dark colored iron sulfates, which discolor the tooth grey. These products can
Penetrate deep into the dentinal tubules and can cause discoloration of the entire tooth

833. Normal pattern of tooth eruption
A. the crown before the root
b. the crowen and root together in oposite way
C. the crown and root in the same way
D. the root before the crown ***

834. Patient has complete denture complains from loose of denture retention now
for 2 weeks at first difficult of wearing of denture have 2 implant in lower jaw cause
1. Bone loss
2. Non parallel implant
3. Loss of rubber part of implant
4. Lose

835. Function of clasp in denture:
A. to hold abutment
B. to distribute stress
C. prevent dislodgment of denture ***
D. produce flexibility to denture

836. The junction between primary and secondary dentine is:
A. A reversal line
B. Sharp curvature
C. A resting line ***
D. A reduction in the number of tubules
837. The placement of a retentive pin in the proximal regions of posterior teeth
would MOST likely result in periodontal ligament perforation in the
A. mesial of a mandibular first premolar.
B. distal of a mandibular first premolar.
C. distal of a mandibular first molar.
D. mesial of a mandibular first molar ***

838. Early squamous cell carcinoma of oral cavity presents as:
a. Vesicle.
b. Sessile mass.
c. An ulcer ***
d. Red plaque.
e. A white cauliflower like lesion
http://www.oralcancerfoundation.org/cdc/cdc_chapter5.htm

839. primary teeth had trauma , tooth change in color become white yellowish ,
what should u tell the parents
a- pulp is dead
b- Inflammation in pulp
c- Calcification of dentin
d- B & c ***

840. C I Kennedy lower partial denture when u put ur finger on both distal ends
anterior portion lifts (elevator):
1. Relining
2. Rebasing
3. Make new. ***
4. Alveoplasty
It says both distal end ...that means the patient has
Kennedy class I = bilateral distal extension
Class II= unilateral distal extension
And because there's no retention in the denture...u can make relining but after some
years and bone get resorption failure happen surly
Rebasing: always used with broken denture base and rebasing always be in acrylic
denture base not metal base, also when increasing in vertical dimension
841. false negative response of an electric pulp test given
a- after trauma ***
B-periodontal disease
C-in teenager

842. isolated pocket in:
A-vertical root fracture ***
B-palato gingival groove
C-endo origin lesion
d- All
www.priory.com/dentistry/Vertical_Root_Fracture.htm

843. What is the color complex of calculus present in 18 y pt.:
A. black complex color ***
B. brown complex color
C. yellow complex color

844. Pt 30 y has accident discoloration upper 1 no bleeding is wide space between
two spaces:
1. splint
2. exteaction 2 part
3. Do endo for root and extract crown

845. Bionator is function appliance what is the first view in orthodontic
1. Sagittal **
2. Ant post
3. transvers

846. Maxillary growth beginning with:
A. apposition of nasal floor
B apposition of maxillary tuberosity ***

847. injecting without vasoconstrictor, the maximum safe
848. Dose of 2% lignocaine solution for 70Kg adult is:
A. 2.2ml **
B. 22ml
http://lifeinthefastlane.com/education/procedures/local-anaesthetic/

849. Child with fracture sub condyle in right said with class 1 malocclusion in left
side and class 2 in right side ttt:
1. Remove of sub condyle and bone graft
2. Open redaction
3. Close reduction
4. No ttt ***

850. basal cranial bone formed from:
1. Frontal and occipital
2. Occipital and sphenoid ***
3. Cranial and sphenoid
851. We want to construct upper denture with palatal strap. Which act as indirect
retainer... what the type of Kennedy class:
A. cl 1 ***
B. cl2
C. cl 3 ***
D. cl 4
Palatal strap used in class 3 not in class 1
As anterior posterior palatal strap
852. Which is correct regarding smear layer
A. Can be removed by MTA
B. Can be enhanced by 37% EDTA
C. Can be removed by MTAD ***
D. Can be removed by mta and Naoh
He effectiveness of MTAD as the final irrigant to remove the smear layer
http://www.ncbi.nlm.nih.gov/pubmed/19452675
http://www.ncbi.nlm.nih.gov/m/pubmed/21273715/
http://www.ncbi.nlm.nih.gov/m/pubmed/19452675/


853. why z plasty performed for frenum detachment:
1. Less scar conyraction ***
2. Easily internes of the tooth
3. less incition need and no suture need
4. Schuchardt Z-plasty
The main advantage of this method over the V plasty:
method was minimal scar tissue formation.
The method requires a skilled operator as it is tedious to perform.
Z-Frenuloplasty helps to release scar contracture and relieve soft tissue tension
854. Which of the following in prepare curved canal is false:
1. Pre flaring tech
2. Remoring action tech
3. Anticurved tech ***
4. Precurved tech
If do against it will broken
855. Which of the following is not contraindication of implant
1. Smoking ***
2. History of radiotherapy
3. Pregnant
4. Diabetic
http://www.isi-clinic.co.uk/Dental_implants/contraindication.html

856. Least concentration of lidocaine for pediatric:
4mg
9mg
2.2 mg ***
It is difficult to recommend a maximum dose of any drug for children, since this varies as
a function of age and weight. For children over 3 years of age who have a normal lean
body mass and normal body development, the maximum dose is determined by the
child's age and weight. For example, in a child of 5 years weighing 50 lbs the dose of
lidocaine HCl should not exceed 75100 mg (1.5 to 2 mg/lb). The use of even more dilute
solutions (ie, 0.25 to 0.5%) and total dosages not to exceed 3 mg/kg (1.4 mg/lb) are
recommended for induction of intravenous regional anesthesia in children

857. ultra sonic in endo has advantage than provisional method
A. coronal portion preparation
B. has torqueing engine
C. more fast ***

858. What is name of laboratory test of HIV positive patients:
A. Elisa *** (Western blot)
Detect antibodies by ELISA then by use western blot procedure
859. What is name of laboratory test for grading HIV patient:
A. CD4 ***
CD4 = T helper cell

860. The most bad prognosis is:
1. gingivitis
2. Periodontitis
3. Moderate periodontitis
4. juvenile periodontitis ***
861. Nikolsky sign is not seen in:
a. pemphigus
b. Hailey diseases
c. epidermolysis bullosa
d. lichen palnus. ***
http://archderm.jamanetwork.com/article.aspx?articleid=543566

862. occlusal splint device:
1.increase vertical dimension
2.help alleviate the increase muscle activity
3.regesteiring occlusal plane
4. all of the above ***
863. Place of retentive arm:
a. Between middle and cervical ***
b. Middle third
c. Cervical third
864. Restoration make chemical bond with tooth structure:
A. Zn phosphate
B.ZOE
C. caoh
D. Zn Polycarboxilate ***
865. Child requires graft in his alveolus what is the best graft:
A. Autogenous cancellous from iliac crest ***
b. Autogenous corticocancellous
c. Freeze dried

866. When we can extract permanent 6..if we need to extract it:
A. after eruption of 7
B. after eruption of 5
C. after formation of 7 furcation ***
D. after formation of third molar
After formation of furcation of 7, at this stage the root will be developed and these will help the
tooth to move to the 6 space

867. Dental material is content from four main categories :
1. Polymer metal ceramic composite ***
2. Polymer metal ceramic stone
3. Polymer metal ceramic cement
4. polymer metal ceramic alginate
http://books.google.com.lb/books?id=iCm1SJBDZwkC&pg=PA423&dq=dental+material+
four+main+categories+polymer&hl=en&sa=X&ei=3y0jU9HPL4rJ0QXKqIDoBg&ved=0CC8
Q6AEwAA#v=onepage&q=dental%20material%20four%20main%20categories%20polym
er&f=false

868. Separate the tooth from the middle (buccolingualty & mesiodestally) to
remove the affected part and preserve the rest of the tooth this procedure called:
A- Hemisection ***
B- Bisection

869. What most age of child trauma:
A. 7-8 yrs.
B. 5-6 yrs.
C. 2-3 yrs. ***
http://www.cosmicsmile.com.au/dentist/dental-trauma-in-children-2.html
http://books.google.com.qa/books?id=JEydZuZjHewC&pg=PA60&dq=child+trauma+age
&hl=en&sa=X&ei=im4jU7yCIKiq4AS6y4CIDA&redir_esc=y#v=onepage&q=child%20trau
ma%20age&f=false

870. When lower 2
nd
molar erupt:
a. 7
b. 10
c. 11 ***
d. 14
.
All 2
nd
molars (upper & lower) erupt 11-13 yrs.

871. What is GG#1 file length means:
1. 20mm ***
2. 30mm
3. 50mm
4. 60mm
GG#1 = Gate Glidden No.1
Gate Glidden is 6 files only
Diameter start from 0.5mm to 1.5 mm
The full file length is 32mm
The shank length is 19mm



872. Slightly displacement of tooth coronal from socket:
a. Extrusion ***
b. intrusion
c. subluxation
Extrusive luxation: partial displacement of tooth from its socket
Subluxation: commonly used to describe loosening of a tooth without displacement.
Oxford handbook page 104

873. Patient with pain on 15 and this tooth undergo with RCT but he still has pain
on percussion, what u suspect?
1. Primary Apical Periodontitis.
2. Secondary Apical Periodontitis. ***
3. over instrumentation.
4. Over medicate

874. Periodontal involved root surface must be root planned to:
1. Remove the attached plaque and calculus.
2. Remove necrotic cementum.
3. Change the root surface so it becomes biocompatible.
4. A and B are correct ***

875. Patient has facial asymmetry what type of x-ray
1- Anterior posterior object***
2- Orthotomograthy
3- CBCT

876. Definition of Auto graft:
A. a tissue graft transferred from one part of the patient's body to another part
877. Question About you making studying of specific group this study mean called:
A. Cohort
B. Convenience
C. Longitudinal
About cholera disease
878. Drugs which should not be given by I.V. Route:
1. paracetamol.
2. Diclofenac. ***
3. choloroquine.
4. Adrenaline.
5. ALL.
Paracetamol = giving IV in dulited concentration in emergincies cases to relief pain
Diclofenac (Voltarein) = shouldn't giving IV only IM (strong antibiotic used in hospital to
relief symptoms
Choloroquine = Anti-malaria giving IV in some cases always make prefer to
give after meals.
Adernaline = giving IV in case of CPR to
879. What is the maximum cartridges can you give for a child: (lidocaine)
A. 1
B. 3 ***
C. 8
D. 13
The maximum cartridges can give for adult is (13) carpule equal 477mg
\\\\\\\\\\\\\\\\\\\\\\\\\\\\\\\\\\\\\\\\\\\\\\\\\\\\\\\\\\\\\\\\\\\\\\\\\\\\\\\\\\\\\
\\\\\\\\\\\\\\\\\ \\\\\\\\\\\\\\\\\\\\\\\\\\\\\\\\\\\\\\\\\\\\\\\\\\
880. What carat gold foil used for direct filling restorations?
1. 16
2. 18
3. 21
4. 24***

881. If a maxillary 1
st
molar is affected by periodontal disease it is expected that the
furcation which will be involved first is the:
1. Distal furcation
2. Buccal furcation***
3. Mesial furcation
4. All furcations will be involved at the same time

882. Thromboplastin and prothrombin deficiency caused by a lack of factor No:
1. 7
2. 5***
3. 8
4. 9
883. Which factor present by decrease in prothrombin time and partial
prothrombin time:
1. IV
2 .X ***
3. IX
4. Xlll

884. Endogenous morphine like substance which can control pain is known as:
1. Bradykinins
2. Peptides
3. Prostaglandins
4. Enkephalins***

885. Melanotic lesion in the buccal mucosa may be associated with:
1. Addison disease***
2. Cushing disease
3. Hypothyroidism
4. Acromegaly

886. Casted post and core indicated for:
1. Flared canals ***
2. Wide canals
3. Avoid canals

887. Geographic tongue also named:
1. Erythema multiform
2. Erythema migrans*** (benign migratory glossitis)
3. Erythema nodosum
4. Erythromycin

888. Inlay:
1. smaller than amalgam
2. Bigger than amalgam ***
3. Depend on the caries.

889. When take biopsy for healthy gingival mucosal from 38 yrs. pt. we harm
hemodesmons detect from:
1. Lamina dura
2. Lamina propra ***
3. Lamina lucice

890. During try in of crown Pt. complain of stud between the teeth what is the
reason:
1. Open contact
2. Tight contact ***
3. Occlusal interference

891. Soldering has all except:
1. Free flowing
2. Pitting ***
3. Lower fusing temperature
4. Corrosion resistance

892. In which stage of orthodontics treatment, the Pt. with heart disease need to
antibiotic prophylaxis:
1. Banding ***
2. debonding
3. Bonding separate

893. Anterior bite plan results in
1. Active extrusion.
2. Passive extrusion.
3. Active intrusion.
4. Passive intrusion

894. Which one of the following is true about oral hairy leukoplakia:
1. Associated with HIV virus infection and is commonly seen on the dorsal of the tongue
2. Associated with HIV virus infection and is commonly seen on the lateral side of the
tongue***
3. Usually caused by Candida species
4. Always associated with trauma to the lateral side of the tongue
5. Always associated with pernicious anemia
Tyldesley's Oral Medicine 5Ed 2003, Page 46

895. Which of the following lesions Cannot be classified as an intra-epithelial
lesion:
1. Herpes simplex infections ***
2. Pemphigus vulgaris
3. Herpangina
4. Lichen planus
5. Hand, foot and mouth disease

896. Gutta percha cones can be disinfected by immersing them for 20 minutes in:
1. Eugenol
2. Isopropyl alcohol ***
3. Acetone
4. Chloroform

897. Pontic with illusion to tooth, all or nearly convex .allow for cleaning area:
1. Modified ridge lap***
2. Ovate.
3. Stationary
4. Ridge lap

898. What is the last step you make in the Try-in:
1. Cheek Periphery
2. Centric relation
3. Protrusive ***

899. Which side shift (bennete) excrete movement:
1. anterior
2. Posterior
3. Lateral ***
4. medial

900. Inferior alveolar block anesthesia in children is given:
1. below occlusal plane
2. At occlusal plane ***
3. above occlusal plane

901. Pleomorphic adenoma... what is the best treatment:
A. enculation
B. excision with epithelium and connective tissue. ***
C. marsupialization
902. Imaging showing recording movement of condyle:
a. MRI
b. CT
c. Arthrography ***
D. plain radiograph.
E. plain tomography

903. Which the following projection is best for examination of fractures of the
zygomatic arch:
1. Waters projection
2. Submentovertex projection ***
3. Reverse Towne projection
4. Lateral cephalometric projection

904. When esthetic is important, posterior class I composite is done in:
1. Subgingival box.
2. Bad oral hygiene.
3. Contact free area.
4. Class I without central contact. ***

905. The following Bacteria causes Pericoronitis except:
1. streptococus meutans
906. Critical PH at which enamel start to demineralization is:
1. 5 ***
2. 6.9
3. 4
4. 2
907. Habit breaking appliance should be left at the end of treatment for:
OR
Child suffering from habit thumb sucking what time by month need to treat this habit:
1. 3
2. 4
3. 5
4. 6 months***
908. 3 years old child with only deciduous canine & molars, the child has light fine
hair and overall appearance of an old person, what is your diagnosis in this case:
1. Hereditary ectodermal dysplasia***
2. Ostiogenesis imperfect
3. Clidocranial dysostosis
4. Gouzon's disease

909. 7 yrs. old child with early missing of D,E in both side in maxilla best space
maintainer is:
1. transpalatal
2. Nancy appliance ***
3. Band and loop
4. Lingual arch
910. Taste in posterior 1/3 of tongue by:
1. Lingual nerve
2. Glossopharyngeal nerve***
3. Chorda tympani nerve
911. Antibiotic prophylaxis recommendations in:
1. Pregnancy
2. Hemophilia
3. Hypertension
4. Rheumatic fever***
912. Residual cyst related to:
1. Vital
2. Non vital ***
913. Which of the following least affect the wound healing in the oral cavity:
1. Infection
2. Blood supply to the area
3. Size of sutures ***
4. Tension in the suture
914. Method of Detecting pulp vitality :
1. CO2
2. Doppler flowmenrty ***
3. Oxygen
4. Heat

915. Pt. 14 yrs. with mamelon in upper centrals:
1. Malocclusion ***
2. Fluorosis
3. Enamel apposition

916. The extraction of maxillary deciduous molar in 5 yrs old child, you should use:
1. Mostly towards the apex pressure and some movement
2. Rotation
3. Distal pressure and movement
4. Labial-lingual movement
917. Remanent of endochondrium make:
1. Periodontum
2. Alveolar bone
3. Synchondrosis ***
4. Max suture
918. Patient came to the clinic with discolored upper central with history of RCT
and trauma the cause of discoloration:
1. Bleeding inside dentinal tubules
2. Microorganism in dentinal tubules
3. Incomplete removal of debris ***
919. Component that prevent inhibiting of gypsum cast is:
1. Sodium sulphate
2. Sodium phosphate
3. Calcium sulfate***
4. Calcium phosphate

920. Mixing of alginate should done:
1. Slowly
2. Vigorous ***
3. Rapid
4. Adequate
921. Elastic recovery of alginate is:
1. 97.5%
2. 97.4%
3. 97.3% ***
4. 97%

922. Pt 17 years complain from lesion like white band in cheek, he was in exam has
history of hepatitis C and adrenal disease in childhood, saw tooth:
1. Lichen planus
2. Lichenoid reaction***

923. Purpose of posterior bite plate as active appliance:
1. ttt of crowding of ant. Teeth ***
2. ttt of crowding of posterior teeth
924. Shape of rest seat in R.P.D :
1. Spoon shape and square ***
2. Concave
3. Convex

925. Which material the denture base made from it and can reline later:
1. Metal
2. Acrylic ***
3. Combined
926. Contraindication of rubber dam in children:
A. mentally retarded pt.
b. nasal obstructive pt. ***
c. pt. with orthodontic

927. Submandibular gland empty at:
1. In the floor of mouth ***
2. At 2 side of lingual frenum
3. Upp. Opposite to 2
nd
molar
4. Posterior near to tonsil
The Submandibular gland is located medial to the angle of the mandible, and it drains its
mixture of serous and mucous saliva via the submandibular duct (Wharton duct) into
the mouth, usually opening in a punctum located in the floor of mouth
928. CATAR:
1. One wall defect
2. Two***
3. Three
929. How much time to take restore normal plaque after vigorous tooth brushing:
1. 1-2 hr ***
2. 4-5 hr
3. 10-11hr
4. 15-20hr
930. Cheek swelling is due to this muscle:
1. Buccinator
2. Masseter ***
931. 25 year old Female in her First Trimester of Pregnancy Presents with an Acute
dental infection, Which of the following is CONTRAINDICATED for this patient:
1. Prescription of a radiograph
2. Prescription of penicillin V.
3. Extraction using 2% xylocaine with 1:100,000 epinephrine.
4. Acetylsalicylic acid for pain management.
5. Aspirin cause teratogenic effects in first trimester***
932. Universal scaller angle:
A. 60 to 80
B. 90 to 100 ***
C. 30 to 40
933. Important in acid etching for pit and fissure sealer is :
1. Remove debris and clean enamel
2. Expose more surface area and provide retention ***
934. criteria of irreversible pulpitis:
1. Severe pain on cold
2. Pain on hot
3. Severe pain on cold and pinging ***
4. Moderate pain on percussion
935. Child with thump sucking and different dental problems the first treatment:
1. Early appliance***
2. Psychological
3. Rewarding therapy
4. Punishment
936. Most retentive abutment:
1. molar with diverge root ***
2. Molar with conical root
3. Molar with little interseptal bone
937. Xerostomia cause:
1. Increase dental caries ***
2. Decrease dental caries
3. Increase cementum caries
938. Width of functional cusp in amalgam building:
1. 1mm
2. 2mm ***
3. 10 mm
4. 20 mm



939. A tooth with grade II mobility & type II recession with moderate bone support
its prognosis:
1. Fair ***
2. Bad
3. Good
4. Moderate
grade II mobility, grade II furcation & moderate bone loss = Fair
grade III mobility, grade II furcation & moderate bone loss = poor or bad
940. Perforation at the middle of the root occur your material of choice to be used:
1. Mineral trioxide aggregate (MTA) ***
2. Amalgam
3. Glass ionomer
4. Calcium hydroxide
941. Young patient with large caries on the lower 6 unrestorable when the dr
should extract this tooth:
1. after complete eruption of the 2
nd
premolar
2. after complete eruption on 7
3. after 6 root reach bifurcation area ***
4. after complete eruption of 8
942. Von Willebrand disease is:
1. Hemophilic disease***
2. Bacterial Endocarditis
3. Congenital cardiac disease
4. Rheumatic fever

943. 3 Walls osseous defect the best graft to be used :
1. Intraoral cancellous bone mixed with patient blood.
2. Intraoral compact bone mixed with patient blood
3. Mixed intraoral cancellous & compact bone mixed with patient blood ***
4. Extra oral compact bone mixed with patient blood
http://books.google.com.lb/books?id=_4002SyMTgUC&pg=PA137&lpg=PA137&dq=Oss
eous+Coagulum+and+Bone+Blend&source=bl&ots=Y6-
wLBGxQ0&sig=9fLjfQOiVdn080jyfx5e-
RcORoU&hl=en&sa=X&ei=JjE0U_CKJeeN0AWK0oC4BQ&ved=0CEAQ6AEwBA#v=onepag
e&q=Osseous%20Coagulum%20and%20Bone%20Blend&f=false
944. Patient has deep caries he want to restore it with esthetic restoration what
suitable base give STRENGTH to restoration:
1. Calcium hydroxide
2. Zinc phosphate
3. GI***
945. In order to decrease gastric secretion:
1. Histamine
2. Adrenal steroids
3. Anticholinergic *** (decrease)
4. Cholinergic. (Increase)
946. Access cavity for upper 4 & 5 premolar has two roots and two canals:
1. Ovoid ***
2. Square
3. Triangular

947. How do you remove a deep carious lesion:
1. Center to periphery with a large round bur
2. Center to periphery with a small round bur
3. Periphery to center with a small round bur
4. Periphery to center with a large round bur ***

948. Psedo. Class 3 malocclusion management:
1. Ant advantage of upper ant teeth
2. Restriction of lower arch ***
3. Lip bumper
949. Which of the following of clasps has more number of component:
1. Reverse back action
2. Jackson clasp
3. Butterfly clasp*** (embrasure clasp)
4. Extended arm clasp

950. lining cells of the maxillary sinus:
1. pseudo stratified ciliated columnar with goblet cells ***

951. Long question about TMJ pain referred to ear and on x ray
1. myofacial pain dysfunction syndrome
2. Synovial chondriomatosis
3. Rheumatic arthritis ***
4. Osteoarthritis

952. Which medical condition require antibiotic prophylaxis:
1. Atrial infiltration
2. Myocardial infarction
3. Artificial ventricular ***
4. Post coronary by pass
953. The distance between two implant:
1. 1mm
2. 2mm
3. 3mm***
4. 4mm
Minimum Distance btw implant - tooth 1.5 mm
Minimum distance btw implant -implant 3 mm
Minimum distance btw implant and max sinus 1mm
Minimum distance implant and IAC 2 mm

954. A 22 year old woman has acute gingival hypertrophy, spontaneous bleeding
from the gingiva and complains of weakness and anorexia. Her blood analysis was as
follows:
HB=12gm, Neutrophils=90%, Monocytes=1%,
Platelets=250000, WBC=100000, Lymphocytes=9%,
Eosinophils=0%
the most likely diagnosis is:
1. Myelogenous leukemia ***
2. Infectious mononucleosis /glandular fever
3. Thrombocytopenic purpura
4. Gingivitis of local aetiological origin
5. Pernicious anemia / Vitamin B12 deficiency
955. Tooth that lose fissure sealant
1. Highly susceptible to caries than tooth not treated by fissure sealant
2. Less susceptible ***
3. Same as normal tooth
http://www.dentalhealth.ie/.../fs_guideline_online_final
956. Shrinkage of co-cr alloy is:
1. 1.7%
2. 3%
3. 2.3% ***
4. More than 3
957. Pulpitis, you decide to make RCT and make access opening from palatal side
What is the most appropriate filling to do will not disturb the crown cementation:
1. Reinforced cement and any appropriate filling ***
2. Towel
3. Pin amalgam seal
958. The best and the most effective way to remove stained mottled enamel
1. Home bleaching
2. Micro abrasion technique***
3. Office bleaching
4. Walking bleaching
959. Cleft lip and palate patient often requires expansion, Appliance of choice in
such cases is:
1. Hyrax appliance.
2. Hass appliance.
3. Cap splint type of expansion appliance. ***
4. SARPE.
5. Spring jet

960. Electrocyte disturbances during massive and rapid blood transfusion:
1. Hypernatremia
2. Hyperkalemia***
3. Hypercalcemia
4. Hyponatremia

961. Difference between acrylic teeth and porcelain teeth for complete denture:
962. Pt. 18 yrs. class 11 has badly decayed centrals need to be restore and there
minimal overlap, the best rest:
1. Metal crown
2. Porcelain jacket
3. Veneer
4. Metal ceramic
963. The best time to treat the pregnancy is:
1. Fist month
2. Second month
3. Forth to sixth months ***
4. Seventh month
964. Theory for defect of residual ridge coronogigivally at Kennedy classification:
1. Cl I
2. Cl II ***
3. Cl III
4. Cl IV
965. Patient came to you complain from discoloration in proximal of upper first
premolar the clinical examination show tooth is intact and also in radiograph no-
cavitation what your decision:
1. No tx.
2. Fluoride application for comprised hydroxyapatite ***
3. Composite restoration
4. Amalgam restoration
966. What kind of suture used under the increase clot:
1. Horizontal matter suture***
2. Vertical matter suture.
3. Interrupted suture.
4. Continuous locked suture
5. 8 shape suture
967. Color choice for cervical third for full coverage:
1. Highest Chroma***
2. Thick enamel
3. Highest value
4. Lowest hue
968. Periodontal abscess occur in:
1. centrals
2. Laterals
3. Canine
4. Molars ***
969. The Ideal palatal-buccal ratio of a maxillary canine be impacted:
1. 6:1
2. 2:1***
3. 3.2
970. We do..during extraction:
1. Expanding socket wall ***
2. Wedging
3. Lever and fulcrum
4. Wheel and axel
971. Patient with gold inlay started to have severe pain on biting:
1. High thermal conductivity of gold
2. Occlusal in discrepancy ***
3. Galvanic
972. X-ray in child not accurate:
1. Sharps, distortion, Contrast, in density
2. Sharps, contrast
3. in density, distortion, contrast

973. Clinical diagnosis of RCT tooth success of treatment is by:
1. Reduction of PA radiolucency
2. No symptoms of palpation and percussion ***
3. Disappearance of inflammatory cells
4. All of above
974. Instrument holds flap edges during suturing:
1. Adson forceps ***

975. In ceram and all ceramic composition:
1. Silica
2. Zirconium ***

976. case of young patient with fever and has sore in hard palate, he had them
once before one year:
1. Recurrent herpes
2. Recurrent aphthous
977. Pt. make for him a long dental ttt come after 2 day with many ulcer of his
mouth:
1. Herpes simplex
2. aphthus ulcer
3. Recurrent aphthus ulcer
4. Stomatitis

978. Glass ionomer cements compared to composite has a higher:
1. Modulus of elasticity
2. Solubility ***
4. Stiffness
5. Linear coefficient of thermal expansion
979. Patient with bilateral lower fixed bridge, what type of occlusion he should has:
1. Mutually protected occlusion
2. Reverse occlusion
3. Unilateral balanced occlusion
4. Bilateral balanced occlusion ***
980. Balanced occlusion and non-balanced occlusion in fixed bridge?

Anda mungkin juga menyukai